Вы находитесь на странице: 1из 262

161  ‫ ﺻﻔﺣﻪﮫ‬ ‫ ﺍاﻟﻰ‬ ‫ﻣﺣﻠﻭوﻝل‬

You receive a call from a school about a boy who is your patient and asthmatic. They are
immediately  about  to  leave  for  a  camp  and  wants  to  know  if  the  child’s  asthma  management  plan  
can be given as they could not reach his parents. What will be ur next step?
a. Give the teacher the prescription for management plan
b. Ask them to bring the child to your practice
c. Try contacting his parents

The fiduciary duty of the state.

In common law a fiduciary relationship exists in any relationship which gives one of the parties
an opportunity to exercise power to the detriment of the other party who depends on that
person  to  act  in  their  interest.  That  is,  a  ‘fiduciary  relationship’  is  one  where  the  ‘fiduciary’  
undertakes to act on behalf of and in the interest of another person.

A fiduciary duty is therefore an obligation to act in good faith toward someone for whom
responsibility has been accepted in such a way as to confer a benefit on that person. This means
the power of the stronger party ought not be exercised to the detriment of the weaker party. The
notion underlying fiduciary obligations is inherent in the relationship between the state, workers
and the young people.

This can be seen for example in educational institutions. The state legally compels all young
people aged 5 to 16 to attend school. The young people whose parents/guardians have placed
them in the care of the state trust workers to act in loco parentis. This is a fiduciary relationship.
The same applies to young people who are under state guardianship, who are placed in care,
protection or in some form of custody. The fiduciary (ie, the state and/or its representative staff)
is obliged to act in the interest of the young person to the  exclusion  of  the  fiduciary’s  own  
interest. Further, persons subject to this duty are not expected to profit from the relationship, or
to put themselves in a position where the fiduciary obligation and personal interest may conflict
(Butterworth’s  Australian Legal Dictionary, 1997: 471). Fiduciary obligations recognise the
position of disadvantage or vulnerability on the part of the weaker party which causes that
person to rely on the other and requires the protection of equity acting upon the conscience of
the other (Batley in PIAC 1997, pp. 50 - 55).

mother came for followup of child. child normal however mother is silent and withdrawn.
wat to ask mother?
-mood
-financial strain
-maritial status
Neonate with skin lesion (described only) with pus .. Organism asked: - Staph - HSV
2) molluscum pic ... No exclusion
3) man from Ethiopia with hx of hCG vaccination now mtx 12 mm no s/s asked next .... Chest Xray
4) swollen inflammed scrotum with black tissue in DMII patient after Antibiotics and IVF what to do:
- blood culture - wound debridement
4) CT IMAGE subdural hge
5) CT IMAGE ICH white rounded lesion next Inv. - CT angio - MRI
6) complete heart block with heart rate of 36 .. Pt on digoxin enalpril n metoprolol .. Mx: pace maker
7) ECG Rapid AF asking Dx
8) ECG mobitz type II what to stop? - Digoxin - ACEI - Apirin
9) 5 sentences about CTG to select which one is the right one !
10) Chest trauma with bilateral # ribs .. Hemothorax is there too .. Patient is vitally stable n maintaing
oxygen saturation .. Not breathing well because of pain .. Next step : morphine
Old man with Fx one leg he will use the walking cane (crutches) where he will put it?!!
same side of affected leg and move it with the healthy leg
The side of the healthy leg and move it with the affected leg
Risk factor for osteoporosis in a 60 year old : early menopause at 35
Patient on treatment for AIDS . Developed Hematuria and dysuria with flank pain : non contrast CT
Ribavirin side effect is hyperuricemia in 38% of its users uric acid stones non contract ct

Noncontrast CT scanning (spiral CT scanning) is the most sensitive and specific study to search for uric acid stone
MEDSCAPE
But there is another scenario when the patient is on indinavir for HIV presented with the same
symptoms ,, choose ct with contrast as the stones here is radiolucent
least 5 yrs survival rate Cx : - testis - lung - pancreas - stomach
Q about staging of colon Cx ?????
MI from 8 months , known AF on warfarin . New X Ray of Cx colon, patient was stable, no
distressing symptoms what to do: - urgent operation - postpone until warfarin effect reversed
If apple core x ray so its not emergency , no need to rush ,,, if x ray or signs of IO then urgent
op. and give FFP
18) Another similar Q but patient with prosthetic valve
apple core x ray so its not emergency , no need to rush ,,, if x ray or signs of IO then urgent
op. but without giving anything

19) Mother brought her daughter claiming that she is behaving differently and some troubles with
father , sitting alone , lack of interest asking which Q (interviewing the daughter alone) will help you
know if she has depression ? - ideas about suicide - does she feel satisfied about herself - lack of
sleep
there is a missing option for mood and appetite
20) recurrent candidal vaginitis treated every time with clotrimazole this time when you swab the
surface bleed asking what to do ? - cervical swab for culture - vaginal swab for culture - HIV test
The next step is a pelvic exam. Your doctor will examine your vagina and the surrounding area to see if there
are external signs of infection. They will also examine your vaginal walls and cervix. Depending on what your
doctor discovers, they will take a vaginal culture to send to the lab for confirmation. Tests are usually ordered
only  for  women  that  have  yeast  infections  on  a  regular  basis  or  for  infections  that  won’t  go  away.
3 or 4 milestones Q one of them 12 months and another 18 months

long scenario post operative case of diverticulitis.surgeon removed the


the affected part of colon and made a colostomy.. Now oozing of
serosanguinous discharge from the operated site,but no fever or
tenderness. Only presence of erythema on the operated site..What
would be the cause?

a) staphylococcal infection

b) streptococcal infection

c) wound dehiscence

Pregnant female with genital herpes how to differentiate 1ry From


recurrent herpes simplex:

 •  HSV Igm now

 •  HSV specific serology now

 •  PCR

 41years old man has headache and is accusing neighbors as they use
insecticide excessively. He mentioned he had frequent trouble with
them as they are very noisy.he changed his living place two times
before as he was unlucky with his neighbors

Thinking that they hate him and want to harm him .other persecution
thinking was there  .

a delusion

b.depression

c. schizophrenia
d. Paranoid personality disorder

 45years  old  man  who’s  working  in  real  estate.  He  used  to  dress well
but recently he looked in bad shape, shouted on customers and fired
from  his  job.  What’s  the  cause?

A. Depression

B. Schizophrenia

C. Bipolar

D. Borderline personality disorder

E. Fronto-temporal dementia

BMI 14 in female, excessive exercise, you will do all inv, except?  

A- ECG

B- Electrolyte

C- Gonadotropin (non routine, others include amylase, CT or MRI) (if


amenorrhea, gonadotropins are routinely done)

D- LFT

E- Bone Density (it is routinely done when low bmi for 6 months or
more, otherwise, it is nonroutine)
Pt 65 yrs ,Xray of chest AP and lateral view given. Looked like right
middle zone consolidation. The lady presented with 5 days history of
fever, cough normal spo2 What will be the most appropriate antibiotic?

a. Amoxicillin clavulanate

b. Fluclox..

c. Penicillin
d. Roxithromycin

e. Erythromycin

pat with renal failure got utitx

Gentam

Nitrofurantion

Trimethop/sulpha

Amoxicillin (if pyelonephritis, go with amoxicillin or ceftriaxone, not


voncomycin in renal failure!)

which antibiotic combination is nephrotoxic:

a.gentamicin plus chloramphenicol

b.gentamicin plus cephalothin

c.gentamicin plus cloxacillin

d.tetracycline plus erythromycin.

A man with peptic ulcer who is taking triple therapy ( PPI ,Amoxicllin,
metronidazole )for 6 weeks but urease breath test was positive for
H.pylori what is the reason?  

A) resistance to metronidazole
B) resistance to amoxcillin

C) unreliable urea breath test

A case of severe UTI. There was chills, high fever and pain . GFR is low
(20 ml/min(

what of the following would you prescribe to the patient

a.trimethoprim

b.vancomycin

c. nitrofurantoin

d. amoxicillin

e. gentamycin

:
Case of stone 9 mm at ureteric orifice in bladder, prepared for surgery,
which investigation must be done before going to surgical table?

a. abdominal xray

b. u/s

c.others
Pt diagnosed with AIDS before 6 months on ribavirin developed severe
loin pain with hematuria..his urine showed +++RBC asking for
investigations

-­triple phase ct scan

-­non contrast ct scan

-­abdominal x ray

Us
Ribavirin side effect is hyperuricemia in 38% of its users uric acid stones non contract ct

Noncontrast CT scanning (spiral CT scanning) is the most sensitive and specific study to search for uric acid stone
MEDSCAPE
But there is another scenario when the patient is on indinavir for HIV presented with the same
symptoms ,, choose ct with contrast as the stones here is radiolucent
initial side effect of resperidone?  

a.early night sedation

b.weight gain

c. hyperglycemia

pregnant lady(G1)come to u with family h/o of mother with DVT at


50age, her sister has h/o abortion.during antenatal period what inx will
be most benefit for her current pregnancy??

FBE
Thrombophilia screen (family history)

AntiphospholipidAb (if the patient HERSELF had history of recurrent


abortion, go with this one)

Coagulation profile

patient with chest trauma +open flail chest what to do at accident


scene

A. Opioid analgesic

B.chest tube

C. Mouth to mouth resuscitation

D. Cover open chest wound with a dressing

 •MVA patient with 96% saturation on room air, finding it hard to


breath b/c of intractable pain and taking shallow breaths due to pain,
multiple bilateral rib injuries, ct provided showing small hemothorax on
rt side what is the immediate treatment to help him breath

Intubation

Morphine

Chest strapping

Underwater seal
 9years old child, his BMI 20 came with sore throat. All other family
members are overweight. What will you tell his parents?

A- He will grow and become normal

B- He should reduce hours of watching TV

C- He should start on weight reduction regimen

D- replace juices with fruits

E does he take adequte salt

Mahmoud Hamdalla:

Female on Clozapine , devloped tachycardia and flattened T wave, Inv ?  

A- Echo

B- Troponin

C- Holter ECG

An old age pt brought to you with complaint of weakness of lower half


of face and right upper limb,she has flaccid paralysis for 1 hour.she is
on aspirin for some cardiac issue.CT scan is normal.what is most
appropriate next step?
a.aspirin and dipyridamole

b.rtpa

Taxi driver, Symptom of night side weakness for 10 min, resolved, CT


normal, carotid duplex also normanl, he is insisting on discharge to
work as driver again  .

A. Discharge but sign consent

B. Refer to neurologist to decide about driving clearance

C. Tell him you can drive in 6 month

D. We need to assess driving after more Imaging

old man coming from travel presents with increasing chest pain and
sweating with hypotension and ECG done it showed ST elevation in lead
2 3 and avf with rbbb (rsR pattern) what is the diagnosis?  

a- Acute inferior MI

b- PE

C lbbb

D pericarditis
Which study to do to find causality between HTN and MI  ?

a. Case control

b. Cohort

c. Cross sectional

d. Randomised control trial

e. Case series

COPD with severe dyspnea, tachypnea , on 8L O2 , spo2 86%, bilat good


air entry, whats the most imp Inv. To be done

A spirometry

B ABG

pregnant in first trim. Her last pap 6 months normal but no


endocervical cells ..what to do

A)repeat pap now

B)repeat in two weeks

C)repeat in 18 months

:
A question asking why to screen for chlamydia?

 •  Chlamydia causes infertility

 •  Chlamydia is asymptomatic

 45years old woman came to you complaining that her 16 years old son
is a drug addict and he is beating her

a. call child protection authority

b. provide shelter or refuge

c.call the police

d. psychological assessment for the boy

pregnant lady who her husband abuse her and make sure not to leave
any marks what is your next action:

-­urgent admission

-­urge her to call police

-­reassure

 -­make escape plans with her


A baby cries a lot ,10 days old,on examination he has systolic murmur
otherwise well and growing well.a

A.reassure because it is normal

B.admit hospital

C.immediateecg

D.refer to paediatrician
Innocent murmurs

Many normal children have heart murmurs, but most children do not have heart disease. An
appropriate history and a properly conducted physical examination can identify children at
increased risk for significant heart disease. Pathologic causes of systolic murmurs include
atrial and ventricular septal defects, pulmonary or aortic outflow tract abnormalities, and
patent ductus arteriosus. An atrial septal defect is often confused with a functional murmur,
but the conditions can usually be differentiated based on specific physical findings.
Characteristics of pathologic murmurs include a sound level of grade 3 or louder, a diastolic
murmur or an increase in intensity when the patient is standing. Most children with any of
these findings should be referred to a pediatric cardiologist

Any infant younger than one year should undergo echo or referred to a cardiologist once
murmurs were detected because of the high possibility of structural heart disease without
symptoms.

 15year female, seprated from boyfriend 6 weeks, now loose wt. , lazy
at work, not good sleep, anxiety attacks & stress?  

A. FLuoxetine

B. Female adolscence group

C. Start CBT

D refer to psychiatrist
A builder comes to you wd headache and dizziness.labs showed
platelets count double than normal.what is your diagnosis?

a.essential thrombocytosis

b.sun stroke  
These results lead us to conclude that: (1) platelet activation is a frequent feature of
heatstroke; (2) in heatstroke altered aggregation responses, whether hyperaggregable or
depressed, occur simultaneously with a consumption coagulopathy.

 23yr female, palpable spleen, pic of thrombocytopenia and leukopenia


normal hb, rash on her legs Dx?  

A. SLE

B. ITP

C. HSP

D aplastic anemia

E acute myeloid leukaemia

 40yr female tonsilitis, took amoxicillin, after 2 weeks she


devlopedpurpuric rash, not blenching, painful joint, fever 38?  

A. Hypersensitivity vasculitis

B. HSP

C. I.Mononucleosis
Purpura and vasculitis does not blanch (extravascular). Erythma usually
does blanch (intravascular lesions)

 .A patient has nausea vomiting, epigastric pain or tenderness (with no


signs of peritonitis). From the history, you knew that the patient had
gastric ligation surgery before, what is you initial management?

a. barium

b. urgent surgery

c. CT scan

a 75 years old man came with progressive constipation in last 3 days


with absent flatus .distention of abdomen is getting worse with mild
tenderness , he has history of 25 years ago cholecystectomy and
appendectomy .in rectal exam rectum is empty .what is the best
diagnosis?

adhesion obstruction

sig volvulus

colon carcinoma
Pt with hx of hemicoloctomy for sigmoid volvulus presented with
moderate to sever abdominal pain on the right side with distention, x
ray was typical for cecal volvulus whats mot app treatment

-­colonoscopy

baruim enema

-­immediate surgery

Gastrograffin swallow

Small intestine serial films

Pilonidal Sinus pic , asked Rx?

A- Fistulectomy

B- Marsupialization

C- Antibiotic

Abscess needs to be drained, the PN needs to be excised entirely with


removal of granulation tissue and the whole track of PN.
Marsupialization should be done to leave the wound healing by
secondary intention and avoiud abscess formation.

 27femal pt. with infertility for 3 yrs , midcycle sex, 1 child before,
semen 15 million,60%,40% ,tube problem in 1 side only, normal
hormones level, with mild pain during sex, wht is the cause of her
inferti;lity

a. Semen abn.

b. Tube abn  .

c. Endometriosis

d. Ovulation problem

 14years old female with body mass index 32, complaining of tiredness
a laziness at work. She had menstruation at age of 12 and menses are
regular. She has dark pigmentation on back of her neck and axilla (this
sentence was repeated twice at the start and end of the scenario). Her
grandmother has type 2 DM. diagnosis?

A- Cushing disease

B- Addisson disease

C- Hypothyroidism

D- PCO

E- Metabolic syndrome

XRAy of hilar LN with chest symptoms like hematuria. what is th next


investigation?

Ca level

CT Abdomen
Xray abdomen

X-ray  picture  of  colls  fracture…management:

Plaster cast in partial extension

Plaster cast in full extension above elbow

Plaster cast in partial flexion and below elbow

Plaster case in complete flexion below elbow

Pregnant lady with 39 weeks of gestation presents with bleeding


200ml, now stopped , clinical examinations reveals 3 cm dilation and
visible membrane, rest is normal vitally, what next:

a. admission and observe

b. AROM

c. C-section

pregnant at 37 wks..sprained ankle badly..u applied crepe bandage and


and advised to keep foot elevated..what is the best advice u can give
regarding pain management..?

a. avoid codein
b. avoid all pain medicines unless it is intolerable

c.Avoid NSAIDS

d. use topical medicine

patient with polycystic kidney disease his GFR is 20 ,,, he is on rimapril


,metformin,statin,aspirin  ……  went  outside  country  then  after  2  years  
he returned to his doctor his investigation was just the same GFR 20

WHICH DRUG SHOULD BE STOPPED NOW??  

A-RIMAPRIL

B-ASPIRIN

C-STATIN

D-METFORMIN

:
 23yrs old female with very weird symptoms. Says she can't remember
whats happening for full day at a time. Complaints of hallucinations
when she tries to sleep. History of 3 such episodes in last 3 months.

Temporal lobe epilepsy

Meningitis

Migraine

Cannabis intoxication

A girl brought an old man who has dysphasia,dyscalculia,Right left


disorientation,he is heavy smoker(30 pack year) and drinker.what is
diagnosis

a.temporal metastasis

b.vermis carcinoma

c.frontal meningioma

d.left parietal glioblastomamultiforme

Female diagnosed with cancer,her daughter came for BRCA 1

,whom she can pass her outcome


A-Life insurance company

B- Superannuation fund.

C-First degree relative

Old female with breast cancer her son doesn't want her to know as she
will be depressed

A Insist to meet the patient alone

B Tell her son to bring all family for a meeting except her

C Give an appointment for the son to discuss this issue

D Tell the patient to bring a family member with her next appointment

Knee pain with swelling in patient with low eGFR

Colchicine

Allopurinol

Indomethacin

Probenecid
xray of bamboo spine not relieved by paracetamol asking about the
most important management

a.methotrexate

b.physiotherapy and hydrotherapy

c.HLA B27

D.esr

SCENARIO OF SECONDARY AMENORRHEA all her investigations was


normal (FSH AND LH are within normal range and no increase in
LH/FSH ratio ).. her WEIGHT IS 45kg  .…  

Us shows normal ovaries with 3- 4 cysts Whats the most likely cause

-­PCOS

-­POF

-­idiopathic hypothalameic dysfunction

aborignal female 3 month dyspnea, on examination b/l basal


crepitation and diastolic murmur, not smoker but binge drinker, next
step?  

A. ECG

B. Echo

C. alcohol counsel about intake


:
Old patient c/o tiredness and leg ede

ma on spironolactone thiazide and analapril no dyspnea on


examination high jvp and lower limb edema with bilateral basal
cterpitation ,ecg shows sinus tachycardia ,echo shows mitral regurge
and ejection fraction 40 what to do

A metoprolol

B warfarin

C digoxin

 40yo female , history of migraine , polycystic kidney ,sudden occipital


headache , no neck stiffeness nor
photophopia , ct normal and 2 LP unsuccessful , next appropriate inv?  .

-­repeat ct

-­MRI

-­cerebral angio

-­Repeat LP

 30year female , 2 babies, now infertility 2 years 28 days regular cycles,


have sex on 10-14 day, cause

A. Luteal phase defect

B. Tubal damage

C. Infrequent sex

D sperms problem
   24yr female had 2 babies before, amenorrhea 2 years, all normal
hormones (prolactin, testosterone,FSH,LH) , estrogen 70, US 3-4
multiple cyst, Dx  ?    

A. PCOS

B. Hypothalamic dysfn  .

C. Pituitary Adenoma

D. POF

37  .yr old woman with secondary amenorrhea normal BMI ,FHS- high
OESTRADIOL low prolactin normal .USG-3 to 4 cyst in ovary.WANT TO
CONCEIVE.Wt is best rx

A)POP

B)Menopause hormone therapy

C)OCP

D)Metformin

I  think  he  means  don’t  want  to  !!

Man previously presented with loin pain radiating to his groin. Non
contrast CT revealed 9mm renal stones. Patient now presents with
worsening pain. What is the investigation you must do before taking
the patient to surgery:
A. Contrast CT

B. Plain x-ray

C. MRI

D. Ultrasound

A young woman with persistant pain on the lower right side of the face
with redness asking about most probable diagnosis

a.varicella

b. trigeminal neuralgia

C herpes simplex

Ctg showed reduced baseline variability and variable deceleration (ctg


not given(

the mother had 4 cm cervical dilation within 14 hours , pethidine was


given to her 3 times I guess

the fetal head is in the left occiput posterior. What is the cause of ctg
abnormality

a. prolonged labour

b. sedative drugs

c.elevated resting uterine pressure


d. molding of fetal head

 6wk boy was ok and increase weight,3 days ago he has increasing
vomiting but he is well hydrated but not increase weight since 1 week
all examintions are normal

A-GERD

B-PS

C-uti

Hand book photo of old pt. who is heavy smoker for many years , had
swelling behind mandibular angle , swelling is fixed , Not changing in
last 12 months, Dx?  

a) Metastatic spread

b) Oropharyngealca.

c) Lipoma

d) Lymphoma

E) parotid tumor

Pic of an old female in nursing home with NG tubes and sick looking
and has a left sided parotid swelling and redness asking about cause

a. Parotid gland tumor


b. Parotid stone

c. Parotid infection

d. Poor oral hygiene

Question about ctg in low risk pregnancy

 1assessment of fetal previous oxygenation

 2assessment of fetal oxygenation now

 3is superior to intermittent fetal auscultation

 4CTG monitoring can sometimes lead to medical interventions which


are not necessarily needed

 5assess fetal reservoir

Female on venlafaxine?  

A-BP

B-LFT

 60years old man feels pain when he walks only two blocks and stops to
rest after that. He has a recent history of left buttock and back of thigh
pain, but he has three month history of ischaemia in left leg and foot. A
bruit is present over femoral artery and His limb is pulseless and cold.
Next step?

A- Thrombolytic

B- Angioplasty

C- Bypass surgery

D- Walking program

E- Nefidepine

Case of chronic limb ischemia : 100m claudication & *it go away at rest,
on examination bruits on femoral artery?  .

A. Angioplasty

B. Femoral bypass graft

C. Intensive walking program

sudanese child what will check for?  

a vit D

b.CBC

c.Hb electrophoresis
Picture of molluscumcontagiosum . exclusion?

.1no exclusion

.2untillscb over

.3untill full recovery

.4after commencement of treatment

pic of pyogenic granuloma

A excision with 1 cm safety margin

B diathermy

C cryot

herapy

D local excision

E inscional biopsy
 .Old pt with alzheimer on antihypertensives, controlled. Wat to give

 1donepezil 2 rivastigmine 3 memantine

 3years old child comes with complain of cough and mother had hay
fever and father is smoker, what drug is used for prevention of asthma
case here:

a.SCG

b.Fluticasone

c.budesonide

d.salbutamol

 5yr old, symptoms of asthma , admitted to hospital, nebulized by


salbutamol & oral steroid, now what on discharge?  

A.Fluticasone

B.Oral Steroid

C.LABA (formeterol(
 5years old boy with cough from 12 months before, was on Salbutamol
and also taken oral Prednisolone. He has an eczema history, both
parents are smokers,( typical asthma scenario) which one is the best
medication  for  prevention  which  is  used  in  “INHALATION  MODE  ?”

 •  Chromoglycate

 •  Salbutamol

 •  Fluticasone

 •  Salmetrol

Child - 2yrs old Harsh Cough, stridor Temp 38 degrees cel

Admitted at 11 pm

What do u administer first.

A- prednisolone IV

B- adrenaline neb

C-salbutamol neb

D-oral steroid
A pregnant lady 15 weeks was exposed to parvovirus.serology test
revealed positive IgM,positiveIgG.WHAT TO DO?

.1repeat serology within weeks

.2viral culture of amniotic fluid

.3USD

.4fetal haemoglobin
Mass in anterior leg, firm regular, attached to gastrocnemius muscle,
most appropriate inv?  

A. MRI

B. U/S

C. CT

D. Biopsy

A boy ate a pizza before few days when he developed diarrhea and
now presented with subconjunctival hemorrhage and epistaxis and rash

what is the test that will lead you to diagnosis

a. blood culture
b. stool culture

LLS 21ST JUNE 2016

HI  guys  these  are  some  recalls  ….I  think  moslty  from  last  6  months  but  
with different opionsso  it  will  change  answer  too  so  don’t  rely  only  on  
recall  ans…I  also  got  some  new  questions  too  .I  Will  update  later.Be  
carefull about time Mx most of the questions with long scenario ,few
only one line quest

1. FEMALE PATIENT AGE 60 YEARS CAME TO CLINIC FOR


POSTMENUPAUSAL OSTEOPOROSIS ASSESMENT AND WORRIED
ABOUT FOR THIS AS HER MOTHER HAS ALSO HAVE HISTORY OF
SUSTAIND # OF HIP IN OLD AGE .WHAT WILL BE THE MOST IMP
FACTOR OR HISTORY FOR DOING BONE SCANE IMMEDIATELY?
A. EARLY MENUPAUSE AT AT AGE 35YEARS
B. SMOKING HISTORY BEFORE AGE 25 YEARS
C. WAS ETHELTES AT AGE 18-20 YEARS AGE
D. FAMILY HISTORY OF OSTEOPOROSIS
2. 2 YEAR OR 18 MOTNTHS ?CHILD BRING TO ER UNWELL SINCE
LAST 36 HRS ,IRRITABLE, LETHERGIC, NOT FEEDING WELL,COUGH,
FEVER 38,DYSPNOEC ,RR 40-60 ,SPO2 88, ON CHEST REDUCED AIR
ENTERY TO LEFT SIDE OF BASE, TRACHEA SHIFTED SLIGHTLY TO
RIGHT SIDE.WHAT IS THE INITIAL MANAGEMENT?
A. STEROID
B. BLOOD CULTURE
C. IV FLUXACILLENE
D. ? ONE MORE ANTIBIOTIC
Pneumonia with effusion

(NO OPTION FOR O2,NEBULIZER,INTUBATION ,IV FLUIDS)

3. CHILD WITH ASTHMA HISTORY SIMILER TO OLD RECALL ON SABA


4 HRLY, AND ORAL PREDNISOLONE TAKING SABA FOR AS A
EXERSISE PREVENTER,LONG SENARIO ASKING FOR MANAGEMENT
FOR ONGOING SYPMTOMS?
A. SABA
B. ORAL PREDNISOLON
C. CICLESONIDE (ICS)
D. BUDESONIDE (ICS)
Plz  don’t  forget to check the age ,, LABA are not
recommended for 5 or less
4. SAME RECALLS 9 MONTHS BABY BRONCHEOLITIS …RSV CAUSE
5. MMR AND EGG ALLERGY SENARIO
6. 25 yr old female complain of Raynaud's phenomenon in
cold,also having edema of bl feet,,bibasal crepitation on
lung bases ,heart on CXR silhute appearance .Ana+, ENA
neg..cz
A-primary Raynaud's
B-cold agglitonin antibody
C-sle
D-limited sclerosis
patient female with raynaudes phenomena with lung infection ana 1/80 and ena negative dx ??

a.primary raynaud b.scleroderma c.sle

7.ECG of HB Mobits type 2 pt on many medications


,indapamide,  verapamil,  prendopril  ,  aspirin…..  presentwth  
light headedness , wt to do first?
a-valsalvamanover
b-cease verapamil
c-temporary pace maker
d. cease aspirin

8.adult male with pic of U/s-there is complex mass, swelling


of scrotum,what to do
a.biopsy
b.CEA
c.alphafeto protein
d.surgery.
ccccccccccccccccccccccccccccc

10.68 year old female have menopause at 55 , present with


purulent brown-greenish vaginal discharge for 6 day , she
is sexually active, what is the cause , previous pap smear
normal
a. Chlamydia
b.endometrial Ca
c.gonnorea
d.ovarrain ca

11.Pic of pyogenic granules ma WD snerio


Trauma to finger 6weks ago aftr that gvn painless lesion is
increasing in size,tx asked
Cautry
Surgecal excision
Cryo therapy
Incisional biopsy

12.Pic of middle lobe pneumonia in 50 yr old male with


fever,cough and malaise from 5 days, tx asked
Augmenton
Benzyl penicillin
Flucloxacillin

13Mother comes with 10 months infant with height 45 head


size 25th at birth then now increase to 75 percentile now ..
On examination child is mild hypotonic with open anterior
fontanelle ,other examination normal,feeding well,what to
do??
Ct head
b. Check cmv
C.tsh
D. Usg head
14.a female had a normal delivary of a normal weight baby ,
there was perianal tear that was small and didn't needed
suturing , after the delivary she noticed that her locia didn't
flow well in the following days , and at Day 4 she presented
with heavy bright red bleed with fever of 37.5 what is the
reason for fever
A)endometritis if fever choose it
B)cervical tear
C)infection with perianal tear
D)DVT
BBBBBBBBBBBBB...........the mom is NOT feverish and tear
is the only option here that can cause bleeding

15.Male 60 0r 65 ?withho backache had sudden vertebral


# at T8/T10 …after  lifting  of  heavy  things..(.no  history  of  ca)
ESR 104, HB 10.4.,low plt.144 wbc normal., ca .normal
2.4..What will do for to rule out diagnosis?
A)bone marrow
B)PSA
C)testosteron
Bbbbbbbbbbbbbbbbbb,,,,,,,final

http://emedicine.medscape.com/article/454114-overview#a3

16.A man with rheumatoid arthritis and HTN is on thiazide


,chloroquine ,aspirin ,enalapril, feeling lethargic and fatigued.
C/F pale, HR ~80bpm BP normal
Lab inv: hb-low, leukocyte-low, platelets-low
What is the cause:
A. Chloroquine
B. Thiazide
C. Aspirin
D. Enalapril
AAAAAAAAAAAAAAAAAAAAAAAAA

17.bamboo spine. Pic pt on paracetamol+ physiotherapy


for  ….long  time  but  not  getting    better  .what  will  next  step  
Mx?
A.physio
B.mtx
C.sulfasalazine if naproxen given, go for it
D.steroids
E.infilixamb
EEEEEEEEEEEEEEEEEEEE.....but NSAIDs should be first
A patient comes in because of difficulty breathing. He reports that he is a very heavy drinker,
and on physical exam there are basal crepitations on both lungs, with no cardiac murmurs
and no shift of the apex beat. What is the most likely cause?
a. MI
b. Alcoholic cardiomyopathy
c. Pulmonary embolism
d. Pneumonia
e.calcified aortic stenosi

18.female ,pg 18 wks , sudden frontal headache, bp 80/60,


pulse 90, on exam: confused and drowsy, heart normal what
inv:
A.Mri
B.Us
C. Echo
D.CTG
19.ECG bradycardia with hyperacuteTwave only one
strip….Middle  age  pt  c/o  light  headach  and  syncopal  attack  3  
episode in one month, bp 100/60 0r 90/60 ??he is on
multiple drugs.which drug comination will be tha cause of his
problem? Actually confusing scenario not that  simple….
A.Amlodipin +Aspirin
B.frusimide+Aspirin
C.Amlodipin+Amiodaron
D.Aspirin+ Nitrtriglycerite
AAAAAAAAAAAAAAAAAAAAA.......hyperkalemia

20.young pt 2weaks pain in shoulder with limited abduction


and  flexion  of  shoulder    after  lifting  of  heavy  things  …what  is  
the initial Mx?
a- paracetmol
b-MRI
c- intrarticular Cortisone injection
AAAAAAAAAAAAAAAAAAAAAAAAAAAAA

 With the exception of red flag conditions, fractures, dislocations and


inflammatory arthropathies investigations are not indicated and will not alter
management.

http://www.racgp.org.au/afp/2012/april/shoulder-injuries/

21. parents brought 3 yrs old son with c/o room spinning
around with no loss of consciousnesses and lasting for 1-
2minutes every month for last 3 months and subside
spontaneously.No h/o headache or vomiting. Ear
examination normal .What will you do ?
A.EEG and CT scan head
B.Audiology and ENT referral
C.Reassure due to BPPV that it will resolve later. (can’t  say  
BPPV without EEG first).

Peripheral vertigo presents with the following:

 Associated nausea
 Vomiting
 Auditory complaints
 Abrupt onset

Central vertigo often produces other neurologic symptoms, although this generalization has many exceptions.
The symptoms are characterized as follows:

 Gradual onset
 Tend to be much less intense than those associated with peripheral vertigo

For a doctor to diagnose Benign Paroxysmal Vertigo of Childhood, all of these things must be true:[3]

 The child has had at least five attacks of vertigo.


 The child gets vertigo a lot; the dizziness happens all of a sudden, and goes away after a few
minutes to a few hours.
 The child had a normal neurological examination (which tests whether there are problems with
how the brain is working)
 The child had normal audiometric and vestibular functions between attacks (this means that
between attacks, the child had normal hearing, and there were no obvious problems with the
inner ears)
 The child had normal electroencephalography (EEG) (this is a test that looks for unusual
electrical activity in the brain).
 Most common causes of vertigo in children are due to migraine or BPPV

22.pyloric stenosis pic from anthology. Pt uop 50ml/hr


asking what to give with iv fluid in 24 hours?
a.20mmolkcl
b. 50 mmmolkcl
c. more than 50 mmolkcl
d. no kcl required!
cccccccccccccccccccccccccccc

23. 60 yr male pt with 2 days ho intense photophobia with


blurring of vision. .vision also reduced on left side
asking diagnosis?
A.conjuctivits
B.gluacoma    if pain
C.keratitis

24..Pic of BCC asking diagnosis


.
25. ECG AF, Brady , complete HB, VF,
With different senario on multiple drugs .

26.old recall old man found collapsed in garden with temp


33.4 + hypothyroidism + heart block asking cause of
collapse?

27.Pt on chemotherapy receiving treatment from cvs line ,


then she developed neck and facial swelling gradually .
What is the next imp mx?
A.Ct chest
b. CT head & neck
C. Limb angiography
AAAAAAAAAAAAAAAAAAAAAAAAAAAAAA
28.ECG. VF….  Pt  brought  in  ER  unconsiuos  ,CPR  on  
going….what  will  do  immediately?
A. Defib
B.Intubation
AAAAAAAAAAAAAAAAAAAAAAAA..........the first step in VF
is defibrillator .....every minute delay in applying the
defibrillation decrease the chance of succseeful defibrillation
by 10%

29. 60 yr male pt with 2 days ho intense photophobia with


blurring  of  vision.  Pain  behind    left  eye….vision  also  reduced  
on left side asking for next investigation ?
A.CT
B.MRI
C.Carotid Doppler
AAA........BUT this case is mostly temporal arteritis so right
option is missing
85 old male pain and tender joints with effusion and no fever cause
asked?
RA
OA
Reactive arthritis
BBBBBBBBBBBBBBBBBBBBBBBBBBB

. Daughter death has made him depressed on sertraline and now


confused Na 124 ,K dec bicarbonate increase rft normal
Stop sertraline
Iv fluids
Na tablets
BBBBBBBBBBBBBBBBBBBBBBBB

15 yr homeless drug addict homosexual came to emergency with


scalp wound what to do?
Inform some authority
Inform his parents
Inform police
Check for std
DDDDDDDDDDDDDDDDD

Patient with vomiting nausea abd pain, ECG given


1) Hyperkalemia
2) V Fib 3
) A.Fib
4) Sinus tacchycardia
AAAAAAAAAAAAAAAAAAAAAAAAAAAAAA........mostly DKA

12 month baby with abdominal mass discovered suddenly next


invst
Us
c.t
initial..........AAAAAAAAAAA
best ............BBBBBBBBBB

80 yr female with vaginal bleeding on examination vaginal


atrophy next tx
S& c
Hysteroacopy
Local estrogen cream
BBBBBBBBBBBBBBBBBB..........exclude cancer 1st

Young girl with pain and menses ocp and NSAIDs fail next?
Usg abdomen
TVS
CT
D&C
Laparoscopy best

65 Pt with abd. Pain with ct done suprarenal mass 5 cm wt to do


1 biopsy
2 partial nephrectomy
3 total nephrectomy
4 mibg

Famous recall of young couple with infertity, regular menses, mid


cycle sex. Cause of infertility?
A- Tubal abnormalities
B- B- Short luteal phase
C- C- Sperm antibodies
D- D- Bad timing

E- ====== most common feature of calcular Cholecystitis:


F- 1 filling defect
G- 2 intrahepatic biliary dilatation
H- 3 thickned wall
I- 4 fluid inside the bladder
J- cccccccccccccccccccccccccccccccc
ECGs of 3degree block,MX Insert temporary pacemaker all other
were drugs in option ,
ecg nf. Wall m.I presented in 2 hrs,mx
angioplasty

Ecg bradycardia in pt. On multiple drugs wt to do


1. Stop metoprolol
2.stop meto gv verapamil
3. Stop prindopril
AAAAAAAAAAAAAAAAAAA

ECG of ventricular fibrillation ,managmnt asked Defibrilation


Pic of pyogenic granules ma WD snerio Trauma to finger 6weks ago
aftr that gvn painless lesion is increasing in size,tx asked
Cautry
Surgecal excision
Cryo therapy
Incisional biopsy
BBBBBBBBBBBBBBBBBBBBBB

2.Pic of middle lobe pneumonia in 50 yr old male with fever,cough


and malaise from 5 days, tx asked
Augmenton
Benzyl penicillin
Flucloxacillin
BBBBBBBBBBBBBBBBBBB

A diabetic pt. Had left leg amputation 2 yr ago, now his hbaic is 9,
FBS 7.8, cholesterol 6, hdl,1.1. What shd he do to prevent right leg
amputation
1. Tight glycemic control
2. Start statin
AAAAAAAAAAAAAAAAAA

45 yr male with malaise 3weks, microscopic hematuria,dysuria, o/e


all abdomen normal,on microscopy pus cells positive,no organism
on culture, Dx
1.t.b nephritis
2. C.a bladed
3.rcc
4.bladed stones

Old patient with macroscopic hematuria ,cystoscopy done but no


finding was mentioned, also has intermittent loin pain wch
radiate to groin ,next most apropriate investigation
1.USG
2.c.t abdomino pelvis
3. Urine microscopy
222222222222222222222222222222………..RENAL  STONES

Some New Recalls (June) Quick Answers:


==============================
85 old male pain and tender joints with effusion and no fever cause asked?
RA
OA
Reactive arthritis
Answer: B: OA
OA in very old age + monoarthritis + Asymmetric + No signs of Inflammation
=====
3 . Daughter death has made him depressed on sertraline and now confused
Na 124 ,K dec bicarbonate increase rft normal
Stop sertraline
Iv fluids
Na tablets
Answer: B
Metabolic Alkalosis .. if associated with Hypernatremia or HTN ythen we think in Renal stenosis, Cushing, or Conn's ... if hyponatremia or
hypotension then this is due to Diahrea or Diuretics
=====
15 yr homeless drug addict homosexual came to emergency with scalp wound what to do?
Inform some authority
Inform his parents
Inform police
Check for std
Answer : D
he is homeless not living with his parents and have his own sexual life .. 1st u check sexual Disease and educate him about Safe sex then
any thing would come after .. but u will not inform his parents or police
=====
Patient with vomiting nausea abd pain, ECG given
1) A.Fib
2) V Fib
3) Hyperkalemia
4) Sinus tacchycardia
as there is no ECG we can not Answer but most likely it is A or D .. so plz give a look at these 4 ECGs to pick the right Answer
=====
12 month baby with abdominka mass discovered suddenly next invst us c.t ===> US
=====
80 yr female with vaginal bleeding on examination vaginal atrophy next tx
S& c
Hysteroacopy
Local estrogen cream
Answer: C
====
Young girl with pain and menses ocp and NSAIDs fail next?
Usg abdomen
TVS
CT
D&C
Laparoscopy
====
65 Pt with abd. Pain with ct done suprarenal mass 5 cm wt to do
1 biopsy
2 partial nephrectomy
3 total nephrectomy
4 mibg
Answer: Partial nephrectomy
=====
Famous recall of young couple with infertity, regular menses, mid cycle sex. Cause of infertility?
A- Tubal abnormalities
B- Short luteal phase
C- Sperm antibodies
D- Bad timing

i am with Bad timing due to Mid coitus ..as it is not accurate for me .. but some gynecologists think that this thinking is applicable in reality but
not for exam ,,, and mid coitus is given to let us exclude bad timing not to allow exaggerated analysis of the word .. and tubal more common !
so this is open for ur discussion
======
most common feature of calcular Cholecystitis:
1 filling defect
2 intrahepatic biliary dilatation
3 thickned wall
4 fluid inside the bladder
Answer: A
Some recalls of june
ECGs of 3degree block,MX
Insert temporary pacemaker all other were drugs in option
, ecg nf. Wall m.I presented in 2 hrs,mx angioplasty
Ecg
bradycardia in pt. On multiple drugs wt to do
1. Stop metoprolol
2.stop meto gv verapamil
3. Stop prindopril
ECG of ventricular fibrillation ,managmnt asked
Defibrilation
Pic of pyogenic granules ma WD snerio
Trauma to finger 6weks ago aftr that gvn painless lesion is increasing in size,tx asked
Cautry
Surgecal excision
Cryo therapy
Incisional biopsy
2.Pic of middle lobe pneumonia in 50 yr old male with fever,cough and malaise from 5 days, tx asked
Augmenton
Benzyl penicillin
Flucloxacillin
A diabetic pt. Had left leg amputation 2 yr ago, now his hbaic is 9, FBS 7.8, cholesterol 6, hdl,1.1. What shd he do to prevent right leg
amputation
1. Tight glycemic control
2. Start statin
45 yr male with malaise 3weks, microscopic hematuria,dysuria, o/e all abdomen normal,on microscopy pus cells positive,no organism on
culture, Dx
1.t.b nephritis
2. C.a bladed
3.rcc
4.bladed stones
Old patient with macroscopic hematuria ,cystoscopy done but no finding was mentioned, also has intermittent loin pain wch radiate to groin
,next most apropriate investigation
1.USG
2.c.t abdomino pelvis
3. Urine microscopy
Obstructive sleep apnea scenario obese with waist n neck circumstances given.asking for long term treatment?
A.wt loss (ans)
B.metformin
C.Cpap
D.surgery
Endoscopic pic of metaplasia but no dysplasia, previous patient had GERDtaken ppi with immediate relief whats next step
Review in 12months(ans) should be 24 months
Endoscopy in 6months
Double the ppi
whats is most imp screening in women above 75 years
a.mamography
b.pap smear
c.FBS(ans)
d.chest x ray
A 44 year old man comes because he is concerned about his health. His father had diabetes at 56.? Next?
A)  OGTT  2  yearly…..
B) 6 monthly RBS
C) FBS yearly(ans)

 #  MAY    
A child is cyanotic, apex beat on left axillary line, loud murmur on the left sternal border.
a) ASD
b) VSD
c) TOF(ans)
d) TGA
Husband and wife travelled to Jakarta, stay in CITY HOTELS, DID NOT travel to rural areas. Wife found husband developed jaundice,
malaise, fever and headache.
a) Flavi
b) Malaria(ans) we need inc. period if
c) Hepatitis
d) Japanese encephalitis
man was with his wife inJAKRTA he had a short stay of 3 days then 3 days ago he developed symptoms of fever , RUQ pain , jaundice ,
what should you do to reach diagnosis?
a- Blood culture
b- Thick and thin blood smear films
c- PCR antibodies to rule out HIV
d- Do Japanese encephalitis antibodies
e- Do PCR to diagnose flavi virus (??)
Pt. after doing gastric bypass surgery .now complaint of lightheadedness , diarrhea , bloating 30 min. after meal , this picture is due to ?
A) Hypoglycemia late dumping diarrhea abd cramps
B) hypovolemia(ans)
C) vagal stimulation
A girl comes to you cz she has some abdominal pain and diarrhea and asks you for certificate for her illness but she is ok now..she has been
in the emergency 3,4 times before for suicidal attempts.what is your diagnosis?
a.malingering(ans)
b.borderline
c. schizoph
‫ ﺑﻭوﺭرﺩدﺭرﻻﻳﯾﻥن‬ ‫ ﺍاﻛﻳﯾﺩد‬ ‫ ﻑف‬ ‫ ﺍاﻟﺳﻳﯾﻧﺎﺭرﻳﯾﻭو‬ ‫ ﻧﻔﺱس‬ ‫ ﺍاﺫذﺍا‬ ..  ‫ ﻋﻠﻳﯾﻙك‬ ‫ ﺍاﺷﺗﻛﻲ‬ ‫ ﺭرﺍاﺡح‬ ‫ ﻭوﺗﻛﻭوﻟﻙك‬ ‫ ﻋﺻﺑﻳﯾﻪﮫ‬ ‫ ﺗﺻﻳﯾﺭر‬ ‫ ﺷﻲ‬ ‫ ﻣﻠﻛﻳﯾﺕت‬ ‫ ﺍاﻧﻪﮫ‬ ‫ ﺗﻭوﺍاﺟﻬﮭﻪﮫ‬ ‫ ﻣﻥن‬ ‫ ﺍاﻧﻪﮫ‬ ‫ ﺗﻛﻣﻠﺔ‬ ‫ ﺑﻲ‬ ‫ ﺳﺅؤﺍاﻝل‬ ‫ﻳﯾﺷﺑﻪﮫ‬
Q.5 An 18 year old case Anorexia Nervosa, which of the following will u find?
A) Tachycardia
B) Low grade pyrexia
C) Delayed relaxation phase of ankle jerk
D) Shivering in a cool environment
E) Loss Of breast development(ans)
Female BMI 18 with repeated treadmill exercise and looks herself every hour in mirror and changes clothes three times per day, dx:
a. Body dismorphic disorder
b. Anorexia Nervosa
c. Obsessive compulsive disorder
A pic of a man with really bad herpes zoster on left flank area. What is the management?
1. Acyclovir
2. Oral antibiotics
3. Oral analgesicsif with in 72 hrs consider acyclovir(ans)
4. Topical antibiotic
http://l.facebook.com/l.php…
41- Patient with hypothyroidism . Ischemic heart disease and hypertension on levothyroxine 150 mcg daily she developed signs of
hyperthyroidism and labs confirmed it so they decreased her dose to 100 mcg today , when should you follow up TSH level ?
•  twice  daily
•  after  1  day
•  after  7  days
•  after  2  weeks
•  after 4 weeks(ans)
#
 #  ENDO    23 year old young lady accountant presented with weight loss, nervousness, irritability, frequent palpitations and excessive sweating
of 2 month duration. She describes her appetite as excellent. She is planning to come off her OCP to have a baby. On examination she has
a mild diffusely enlarged thyroid gland, pulse rate was 110, sweaty moist palms, and peripheral tremor. Her thyroid function tests show a
suppressed TSH and raised T4,T3 levels. What is the best management plan you can offer to her?
1. Commence propranolol and carbimazole.
2. Offer anti-thyroid medication and reassure that the drugs are very safe in pregnancy
3. Strongly recommend surgery as immediate therapy.
4. Offer radio iodine treatment and defer pregnancy for 6 months(ans??)
http://l.facebook.com/l.php…
- schizo p.t on antipscychtc have weight gain, most is the next investigation?
a. TSH
b. FBS
c. Lipid profile(ans)
man with rt sided lower chest pain radiating to rt hypochodrium..febrile..tenderness over rt hypochondrium bt no rigidity..a flu epidemic is
going on..
a. rt empyema
b. cholecystitis
c. sub phrenic abscess(ans)
Patient after influenza epidemic came with resp difficulty and pleuritic chest pain in his right lower lung referred to his upper abdomen A)
Empyema B) subphrenic abscess C) cholecystitis D) pancreatitis
Patient with a trauma you want to give him blood transfusion what will you choose ?
•  O  group  blood
•  O  negative  low  titer  whole  blood
•  O  negative  low  titer  Packed  RBC  units(ans)
•  Give  blood  without  cross  matching
Pic of frost bite in great toe after camping trip Wat to do
Resurrance it's just frost bite
Remove the toe
Review in 4 weeks (ans)
Remove only the nail
30 years old lady came to you because of 8 weeks ago her marriage was broken, 1 year ago she was referred for psychotherapy from work
(Shouting at colleagues) . After finishing her work she cant able to relax at home. What is the diagnosis?
A) Bipolar-II with depression
B) Cyclothymic disorder
C) GAD
D) Major depression
E) Borderline personality disorder(ans)
5.Pancreatic pseudocyst scenario post Pancreatitis episode fullness and stuff
Asking Ix most app
CT (ans)
EUS (endo US)
6.Most earliest complication post pancreatitis :
RF (ans)
Pseudocyst
#
Pancreatic pseudocyst scenario
Asking Ix most app
ERCP e` biliary drugs
endoscopic cystography
open cystography
http://l.facebook.com/l.php…
man with nasal blockage especially at night & can't sleep well.he has periorbital darkness.allergy test (+).what is ur initial step??
Normal saline wash
Oral cetrizine at night
Oral(another antihistamine) in the morning
Oral steriod at night
Nasal steriod spray at night(ans)
woman  her  pap  smear  didn’t  show  abnormal  cells  but  the  endocervical  cells  were  absent.  Her  previous  pap  was normal
what would you do?
Reassure that no test are required any more(ans)
b. repeat pap smear now
Woman with pap smear showing LSIL.Previous pap smear 3 yrs back was negative.
A. Repeat pap smear in 6 months
B. Repeat pap smear in 12 months(ans)
C. Colposcopy
Mother brought her 16 year old ballet dancer girl..breast bud stage 2, tsh fsh and prl was NL...amenorrhea present, next?
USG(??)
reffer to adolescence unit or something like
reassure
pap smear
16yr ballet dancer, present with amenorrhoea, BMI 15 and breast bud present, what is
Next management?
FSH
Review in 3-6 months
Refer to adolescent health (??)
most common feature of calcular Cholecystitis:
1 filling defect
2 intrahepatic biliary dilatation
3 thickned wall (ans)
4 fluid inside the bladder
A 64 years old man with incontinence of the urine after a long trip. He have to strain strongly in order to evacuate his bladder.You did rectal
examination, you felt enlargered irregular prostate.
Prostate specific antigen(ans)
CT prostate
MRI
US (TRUS)
Cystoscopy
1.Young man 30-35 yo complains on 2 years history of problem with finish his urination. He states it's feels like smth in urethra. And after
finishing void he still feels his bladder is not empty. No fever, no pain. Just incomplete voiding. What is the best investigation in his case?
1- ct
2- mri
3- cystoscopy
4- uretroscopy(??)
2.young man has difficulty with initiating urination and drippling sometimes how to initially diagnose?
a) venous pyelography
b) transrectal sonography(ans)
c) CT
d) MRI...
49.What the most common cause of infection of gall bladder?
A. staphyloauriusb.E Coli(ans)

Boy when sleep on his back stridor, otherwise ok, stressed tachypnea and cyanosis?
A. Foreign body inhalation
B. Epiglotittis
C. Broncholitis
D. tracheomalacia

Tracheomalacia (from trachea and the Greek μαλακία, softening) is a condition characterized by
flaccidity of the tracheal supportcartilage which leads to tracheal collapse especially when increased
airflow is demanded.

The trachea normally dilates slightly during inspiration and narrows slightly during expiration. These
processes are exaggerated in tracheomalacia, leading to airway collapse on expiration. The usual
symptom of tracheomalacia is expiratory stridor or laryngeal crow.

Old age female with IHD on thyroxine, 75 mcg


Lab showed hypothyroidism, highTSH and low T3 and T4
What is next?
Increase thyroxine to 100
Keep the same dose
Decrease thyroxine
Man on eltroxine and lab showed hyperthyroidism,
Low TSH and high T3 and T4, you increased the dose, when to follow?
After 1 week
After 2 weeks,
After 3 week
‫ ﺳﻲ‬ ‫ ﻧﺧﺗﺎﺭر‬ ‫ ﻣﺎﻛﻭو‬ ‫ ﺍاﺫذﺍا‬ ‫ ﺷﻬﮭﺭرﻱي‬ ‫ ﺍاﺏب‬ ‫ ﺍاﻟﻔﻠﻭو‬ ‫ ﻋﺎﻣﻪﮫ‬ ‫ ﺑﺻﻭوﺭرﻩه‬ ‫ ﺑﺱس‬ ‫ ﺍاﻟﺳﺗﻳﯾﻡم‬ ‫ ﺧﻁطﺄ‬ ‫ ﺑﻲ‬ ‫ ﺍاﺗﻭوﻗﻊ‬ ‫ﻫﮬﮪھﺫذﺍا‬
12 month baby, the mother noticed mass in the Rt hypochondrium
What is the next
CT
MRI
Us abdomen
X ray
X ray abdomen for adult male picture of toxic megacolon, rectal tube inserted, no improvement
What is next
Colonoscopic decompression
Surgery
NGT insertion
You are in the ER, a young female came with Rt abdominal pain, sweaty and dizzy, BP 90/60, tachycardia 120.
You are concerned about perforated appendix, you called the surgery oncall, he was narcissist on you and told you call the Oby/ Gyne to
exclude ectopic pregnancy and hang up the phone.
What to do
Call him again
Do pregnancy test
Give Iv fluids      could be septicemia
Call Oby/ Gyne
Male patient after tranurethral resection of prostate, became pallor and hypotensive
Give Blood
IV fluid 1 liter over 1 hour
Iv fluid 1 liter over 2 hours
Old pain taking many painkillers for cancer pain
He is unresponsive and received naloxone and high flow oxygen after which he opened his eyes and then he became drowsy again.
ABG Po2 120, PCo2 55, Hco3 26
What to do.
Decrease oxygen
Give naloxone
Intubation
Give morphine
Another recall of COPD patient with ABG of PO2 60, PCO 58, HCO3 32
What is the cause of his low conscious level
Hypercapnia
Hypoxia
Normal PO2 more than 60, below 60 is hypoxia
Normal PCO2 is 45, hypercapnia is more than 50
Man came with shoulder pain after lifting heavy object.
What is next
X ray
MRI
CT
Bone scan
Female patient came for conceive, she was on OCP for 10 years
On examination you found the cervix is bulky and uterus is retroverted.
What to do
Pregnancy test
US pelvis
He in the stem she is amenorrhic so A .. if not then us
Old man with x ray showed white lesion on the left side of the lung.
What to do next
CT
MRI
Bronchoscope
Man with red face , congested neck veins, and picture given showed of mass in the middle of the neck, what is next.
US neck ‫ ﺍاﻻﻧﺛﺭرﻭوﻟﻭوﺟﻲ‬ ‫ ﺻﻭوﺭرﻩه‬ ‫ ﻧﻔﺱس‬ ‫ﺍاﺫذﺍا‬
MRI
CT
CXR
Lady is pregnant and she has picture of Pulmonary embolism.
V/Q scan
CPTA
CXR

Adult male came from England, scenario of Pulmonary embolism


V/Q scan
CPTA
CXR
Memories wells score
39 weeks vid contractions every 3 mins , cervix posterior n 2cm dilated n head 3 cm above IS. After 4 hours cervix mid position, fully effaced,
4 cm dilated, intact membranes, head 1 cm above IS. Dx ?
A. Spurious labour
B. Incordinate labour
C. Normal labour
D. Obstructed labour
Spurious labour =false labour False labor: Intermittent non-productive muscular contractions of the womb (uterus) during
pregnancy, most commonly in the last two months before full term. These contractions are non-productive in the sense that
they do not produce any flattening (effacement) or dilation (opening up) of the cervix.
Incordinate labour
An abnormality of the first stage of labor in which uterine contractions are too weak or too ineffective to dilate the cerv
ix.
Clear scenario of imperforate hymen
Female delivered a baby 3800gm at 37 weeks without any complications. Placenta out after 5 mins, syntocinon given. After 30 mins of
deliver, 1000ml of bleeding vidin10 mins. Dx ?
A. Birth canal lacerations
B. Atony
There  is  another  stem  where  he  tells  u  if  the  uterus  is  contracted  or  atony  ,  any  why  if  didn’t  mention look for causes of atony
(prolong  labour  ,  big  baby  ,  twin  ,  polyhydromenios  ,  exesive  use  of  oxytocin  ….etc  )

A 25yo woman with T1DM has delivered a baby weighing 4.5kg. Her uterus is well contracted.
Choose the single most likely predisposing factor for PPH from the options?
a. Atonic uterus LAX
b. Cervical/vaginal trauma
c. Retained POC LAX
d. Large placental site LAX
e. Rupture uterus LAX

55 yr female with breast lump in outer quad. Mammo done , found nothing. Usg done, found hypoechoeic mass. What next ?
A. Core biopsy
B. Reassure
C. Excision biopsy

HERE D
A
C
C
B
IV Ca gluconate
Indapemide
A
RSV
CT
Endometrioses
CT
Pacemaker

3- Woman, came back from a trip to south east Asia 2 months ago , at that time she had
lower right Quadrant pain and tenderness then relieved now she has fever, Pain in right
upper abdomen, with high bilirubin and fever , Diagnosis? A) cholangitis B) Hydatid
disease C) Clonorchiasis D) liver abscess

20.a soldier with proteins+ on unrine exan wat woud u do next

a.24  hrs  urine  …to check if the proteinuria is glomerular in origin

b.albumin creatine ration…to check if it is transient

A child is cyanotic, apex beat on left axillary line, loud murmur on the left sternal border.
a) ASD b) VSD c) TOF d) TGA
TOF is usually presented at the age of 3-6 months when the pulm vasculature starts to
be more resistant with increasing age. Child usually in compensation squatting position.
It is presented with cyanosis, especially on feeding or excertion, with systolic thrill on the
left sternal border with ejection systolic murmurs in pulm area. The condition is
aggrevated by age, infection or feeding.
11- 35 yr woman with amenorrhea, have 2 children at age 31, 34. herbmi 16.past h/o
amenorrhea at age 18-25yr.she is anxious about her menopause.whatinx will be benefit
for her? A) FSH B) BMD C) US D) Oestrogen Leve

Young boy with intermittent pain in flank and radiating to thigh for two years, now pain is
continuos with vomiting, a mass felt below left hypochondrium, dx
I) Wilmstumour
J) Nephroblastoma
K) Ureteropelvic junction obstruction

Middle aged woman who collapsed two times in a mall within a week, with diastolic
murmur best heard on right (or the left not sure), and systolic murmur best heared on
the left (or the right not sure). Whats the cause? A) Asd B) Vsd C) viral cardiomyopathy
D)1ry pulmonary hypertension

There is another stem where mixed aourtic lesion is an option ,, it all depend on
the site of the murmur
Pt came with suspected cancer caecum,wt is in the history made u suspect A. Fatigue
B. Dark stool C. Change in bowel habit
Vit  C  effect  on  iron  tablets”  which  study  you  do  ?  a)  Cohort  b)  Case  Control  c)  Case  
series d) Double blind
Any efficacy choose RCT ,, the option here is RCT with Double blind

.65 3 year old boy with chronic hematuria and proteinuria. NO other abnormality found.
Dx
A)Chronic glomerulonephritis
B) Nephrotic Syndrome
C) thin basement disease
‫ ﻭوﺍاﺣﺩد‬ ‫ ﺍاﺭرﻫﮬﮪھﻡم‬ ‫ ﻫﮬﮪھﺫذﺍا‬ ‫ ﺑﺱس‬ ‫ ﺫذﻭوﻟﻪﮫ‬ ‫ ﻣﻥن‬ ‫ ﻭوﺍاﺣﺩد‬ ‫ﻭوﻻ‬
Dr.EslamJune Recalls

1. Mass in anterior leg, firm regular, attached to gastrocnemius muscle, most appropriate inv ?
A. MRI
B. U/S
C. CT
D. Biopsy

2. 24 yr female had 2 babies before, amenorrhea 2 years, all normal hormones (prolactin,
testosterone,FSH,LH) , estrogen 70(normal from 75 to something), US 3-4 multiple cyst, Dx ?
A. PCOS
B. Hypothalamic dysfn.
C. Pituitary Adenoma
D. POF

3. Same above Scenario but ↑↑↑ FSH and ↓↓↓estrogen, Dx?


A. PCOS
B. Hypothalamic dysfn.
C. Pituitary Adenoma
D. POF

4. Molluscum C. pic exclusion ?

5. Herps pic in arm of kid

6. Pic of parotid mass, poor oral hygiene

7. Pic Xray middle lobe pneumonia

8. 24 yr female cough, bloody sputum, hematuria, rash on ant.tibia, ACE ↑, Inv ?


A. Skin Biopsy
B. ANA
C. ANCA

9. Xray hilar L.N &


chest symptom &hematuria , Next inv. ?
A. Ca level
B. CT Abd.
C. XrayAbd.
But again remember if the compliant if acute hematuria we do ct to the abdomen , if the history of hematuria is
chronic or he want the dx go for ca level
10. 22 year female, seprated from boyfriend 6 weeks, now loose wt. , lazy at work, not good
sleep, anxiety attacks & stress ?
A. FLuoxetine
B. Female adolscence group
C. Start CBT

11. Boy when sleep on his back stridor, otherwise ok, stressed tachypnea and cyanosis?
A. Foreign body inhalation if history suggestive go for it
B. Epiglotittis
C. Broncholitis
D. Trachiomalacia
Infants present after a few weeks of life with expiratory stridor (also called laryngeal crow). Expiratory stridor may worsen with supine position, crying,
and respiratory infections. Feeding difficulties are reported sometimes. Hoarseness, aphonia, and breathing also may be reported.
Obtain history of an acquired etiology, such as prolonged intubation, tracheostomy, chest trauma, recurrent tracheobronchitis, cartilage disorder
(relapsing polychondritis), or lung resection.

12. Old female , H/O appendic. 30 years ago, abd distention & vomiting 24 hrs, by
Hx 1 month increase constipation ..
A.sigmoid valvoulus
B.fecal impaction
c.small bowel ob.
D.ca sigmoid
13. farmer U/S liver multiple hypoechoic lesion, had hemicolectomy for ca. colon before 2 year,
most app inv ?
A. Hydatid cyst serology
B. Triphasic CT

14. Bedridden old male, constipation and dribbling of stool, Dx ?


A. Fecal Impaction

15. 80 yr male xray, constipation and vomiting for 3 days, history of appendix 30 years ,Dx?
A. Ca
B. Smal bowel Obst.

16. 38 week preg., cervical dialt. 3cm, 200ml bleed, U/S normal baby, Post. placent ?
A. ARM
B. Obervation
-female with bleeding 200 ml in her 39 weeks of gestation ,, she had stable vital signs ,, stable baby ,, uterus is
soft ,, on speculum exam there was no blood but the cervix was dilated 3 cm and membranes are clearly visible
,Ultrasound was done revealed placenta in fundus in posterior part of uterus whats is your management :-

-admission for observation

-send her home

-urgent C/S

-Artificial rupture of membrane

17. 24 female amenorrhea 12 months, pain during sex, on examination uterus size normal, all
examin. normal, what will you suspect?
A. Pregnancy
B. Endometriosis

18. Preg. measles on week 16, IgM +ve after 24 hrs of symptoms rash, rhinorrhea ? (What to do
)
A. MMR Vaccine contraindicated
B. IVIG she is already infected
C. Serology of baby
D. Search doctors and patient who contact her
The story of measles in pregnancy
If contact with someone with the disease within 6 days do serology and give IVIG after the results(except if
immunocompromised  don’t  wait)
Administer NHIG to measles antibody negative women who are immuno-compromised following the CDNA guidelines
for dosages. If antibody testing is not available for 72 hours (weekend), administer before results are received

If contact with someone with the disease within 6 days and she is 20 weeks or more do serology and give IVIG after the
results(except  if  immunocompromised  don’t  wait)    and  give  IVIG  to  the  baby  after  birth
If already have symptoms or infected (igm+ve ) or beyond 6 days then trace contact

19. Travelling to endemic typhoid, took vaccine already, what to advice ?


A. Wash hands
B. Avoid all fruits n veg.
C. Bottled water

20. 30 year female , 2 babies, now infertility 2 years, did histosalphingography: one tube
problem, did U/S, normal uterus thickness, regular cycles, have sex on 10-14 day, cause ?
A. Luteal phase defect
B. Tubal damage
C. Time of sex

21. old POF scenario: she dosen't want to be preg. & Don't have sex ?
A. OCP
B. HRT
C. POP
D. Estrogen

22. 45 year female, urge incontinence, nocturia, uterus prolapse,on ex no abnormality in pelvic
floor, bladder full then did cough test, no dribbling ,Dx?
A. Urge incontinence
B. Stress Incontinence
C. UTI

23. 49 year female, nocturis, urgency, affect physical life, 1st inv ?
A. Urine culture
B. Urodynamic study
URINARY infection should be excluded; any microscopic haematuria is an indication for investigation of localised bladder
pathology Urodynamics provides the most definitive assessment of lower urinary tract function and is generally well tolerated by
the elderly. Indications include: When a conservative management
regimen has been unsuccessful. When the clinical picture is unclear. Before considering continence surgery. In some patients
with a neurogenic cause of incontinence.
24. Old male present with constipation, have ca with bone metastasis, took 20mg morphine for
pain last week, Inv ?
A. Morphine level
B. AbdXray
C. Serum Ca.

25. 9 month boy chest Xray normal, grunting & 39' fever, weez, given O2 n IV fluid, ?? (same old
recall)
A. RSV
B. Bacterial pneumonia
C. Asthma

26. 23 yr female epistaxis, headache, malaise, dizziness, LABS: Hb-8.5 Ca-1.9 INR-1.5 Ferritin
↓ , Rx?
A- Vit. K
B. Iv Ca
C. Plasma
D. Blood transfusion

27. 35 year female , her mother has breast ca at 60 year, she is afraid to have ca ? (most
appropriate next)
A- Breast screening program
B. Self examin.
C. Reassure
D. Come when you are 49 to start screening program

28. 47 year female afraid from osteoporosis, what is risk factor of Osteopo. in history?
A. Family Hx at 80 years
B. Early menopause at 35

29. Open flial chest, pain, distressed, cyanosed ? (most appropriate next)
A. Morphine if not then negative pressure
B. Needle thoracotomy
C. Cover wound

30. Male AF, whitish lesion in Ct brain ?


A. Echo
B. CT Angio
C. MRI

31. Rupture membrane & meconium, baby PR 144 bpm ,iariability 5 , no decelartion, no
accelaration ?
A. Normal
B. Abnormal & need sampling
32. ECG wide complex , slow af biventricular failure, basal crepitation, E.F=40% on
digoxin,lisnopril,verapamil ?
A. increase digoxin
B. stop all drug and check in 12 hours
C. decrease verapamil
D. Add lasix 20ml/day

33. aborignal female 3 month dyspnea, on examination b/l basal crepitation and diastolic
murmur, not smoker but binge drinker, next step ?
A. ECG
B. Echo
C. alcohol counsel about intake

34. Recall of gradual walking program

35. Case of chronic limb ischemia : 100m claudication & it go away at rest, on examination
bruits on femoral artery. ?
A. Angioplasty
B. Femoral bypass graft
C. Intensive walking program

36. 24 yr female, BMI 32, stria on abd, dark pigmentation on neck & axilla, normal cycle, H/O
mother early DM ?
A. Hypothyroid
B. Metabolic Synd.
C. Cushing Synd.

37. 2 yr old have pertussis, have 5 month sister, after Rx ?


A. DPTa for sister
B. DPTa for adult of unknown hx of vacc.
C. Erythromycin
D. Parents & Grandparents Vaccine

38. 80 yrfemale , her daughter will have baby in 2 days, what to take?
--> DTP vacc.

39.Pic of Baby born with mass in testes &Transillumination test +ve ---> Hydrocele

40. Baby in 7th day of age jaundice, Total Bil. 240, InDirect 120, palpable spleen 2cm below c.m
?
A. Haemolytic Anemia
B. Biliary Atresia
C. Breast Milk Jaundice
It is characterized by indirect hyperbilirubinemia in a breastfed newborn that develops after the first 4-7
days of life

I think he chose this because was not in the options


41. 23 yr female, palpable spleen, pic of pancytopenia, rash on her legs Dx ?
A. SLE
B. ITP
C. HSP

42. 40 yr female tonsilitis, took amoxicillin, after 3 weeks she devloped purpuric rash, not
blenching, painful joint, fever ?
A. Hypersensitivity vasculitis
B. HSP
C. I.Mononucleosis

43. 17 male ate pizza, had diarrhea, s.conjuctivits, hematuria, fever, what to do to dx ?
--> stool culture

44. Female present to you.. her son hit her n he is a drug dealer ?
A. Admission
B. Inform Police
C. Boy to rehabilation
D. Arrange escape plan

45. 30 yr female present to you, her husband hit her but dont leave marks (old one) ?
A. Police
B. Refugee Center
C. Talk to husband

46. Alturism Recall


47. Derailment recall
you  are  examining  a  women.  You  asked  her  about  something,  she  answered  “I  woke  up  in  the  morning,  I  will  tell  you  about  the  
whales,  the  baby  is  crying”  what  is  the definition of her thought disorder?
a. derailment
b. pressure of thoughts
c. flight of idea

d. circumstantiality

48. female don't want to eat because she think her bowel are rotten, she is dehydrated ?
A. NS until urine output 2ml/kg/hr
B. D5 1L/hr
C. 1/5 NS/ 4hr
D. NS until urine output 1ml/kg/hr

49. Old age , fever sign of pyelonephritis, BP 90/60, confused and dysuris, pus in urine in
urinanalysis, u send for culture, GFR<20 *managment ?
A. Amox
B. Trimeth. &Sulfa
C. Gentamycin
D. Ciprofloxacin

50. Old age male, UTI, culture done, E.Coli was there, he took antibiotic, 4 days after, he
devlopedchill,fever and abd.pain, what is the best inv of choice ?
A- U/S Renal next
B. CT Abd.
C. Retrograde Urethrogram
D- IVP

51. CT given (Ischemic stroke), Old female, Rt side hemiplegia, dysarthia came after 1 hour from
symptom, best Mx ?
A. rtPA
B. Clopidogrel& Aspirin
C. Clopidogrel
D. Warfarin

52. 24 female, sudden loss of conscious attacks, at bed time she have visual and auditory halluc.
, weakness in rt. side ?
A. Rt. temporal lobe
B. Lt. temporal lobe
C. Partial lobe
D. Occipital lobe
Temporal lobe epilepsy. It is a combination of:
1. memory  impairment,  so  ask  a  witness  when  the  patient  can’t  remember
2. Aura; could be characterized by auditory hallucinations, visual hallucinations,
emotional flashbacks, de ja vu, or even psychic phenomenon.
Diagnosis by clinical history, EEG and high resolution MRI to show the
calcification of hippocampus.
53. Acalculia, agraphia, dysathria, weakness on the left side
A. Medulloblastoma in Rt. partial lobee
B. Medulloblastoma in Lt. partial lobe
C. Cerebellar tumor

54. Female on haloperidol for a while , now develop ExtrapyramidalSymptoms, cogwheel


rigidity, repetative fall, what is most imp sign to confirm dx ?
A. Dysdiadochokinesia
B. Bradykinesia extrapyramidal=parkinsonism
C. Sialorrhea

55. 15 yr Drug abuser, shout to his parents, decrease in school performance. Most thing that let
that boy have good psychiatric therapy.
A) To show him similar cases like him and tell him that those case are improved
B) Talk to him about his life
c)to make him confident that all what he says will kept as a secret

56. ***Female insomnia and irritability, psychosis symptoms, she was on some weird drug (
trichloro ----) for 2 years and became good but she stopped itbecause of stiffness, even when
she stopped symptoms came back ?
A. Quitiapine
B. Clozapine
C. Same drug she stopped

57. Old male on antidepressant for 8 year, MMSE score- 23 , now irritable and insomnia, not
socially  active,  lazy  all  the  time,  don’t  talk  to  anyone  ?
A. increase antidepressant
B. change antidepressant
C. Add Antipsych with antidepressant

58. You r in family party, your cousin have exam next week, want BZD ?
A. Give BZD
B. Give melatonin
C. Physiotherapy & exercise
D. tell his parents
E. Tell him to see formal doctor for advice

59. Male, MMSE 18, HTN &IHD ?


A. Memantine
B. Rivastigmine

60. Male, pancreatitis two months have tenderness and fullness in Abd. , On CT 10cm
Pseudocyst ?
A. Endoscopic Gastrectomy
B. Laprotomy
C. Percutaenous Gastrostomy

61. Middle age 47 or 48 yr non to have IHD, have RUQ pain, put stent from 2 months, on
clopidogrel, pain resolved, but after 9 week the pain comes back. on U/S stone was in neck of
gallbladder, there was fever n chills n tenderness, no jaundice ?
A. TranscutaenousColycectectomy
B. Antibiotics
C. Stop Clopi n Make operation in 5 day
D. Give hip platlets& Do operation

62.Xray Colles #, cast was done, when to do Xrayagain ? --> 2 weeks

63. A woman on prosthetic valve, she is takingMarevan( warfarin) and you are checking her INR
every week.
Date INR WARFARIN dose in mg
2nd April she was on 4 mg her INR is 1.5
9th April she was on 5 mg her INR is 1.8
16th April she was on 6 mg her INR is 2.1
23 April 2.1 ?
what is the dose of warfarin will you give for this week?
a. 5 mg
b. 6 mg
c. 7 mg
d. 8 mg
e. 9 mg

64. PE recall, female came from england, after flight low grade fever, dyspnea, chest pain? --->
CTPA

65. Sudden pain in smoker, IHD Hx, PVD Hx, sudden pain and pallor in left Leg, on examination
felt femoral pulse but no popliteal pulse, best Mx ?
A. IV heparin
B. Unfrac. Heparin
C. Warfarin
D. Verapamil

66. Old DVT Hx, now low grade fever, calf muscle redness and tender, ankle redness, best Inv ?
A. Blood culture
B. Lower Limb Duplex

67. Best preventer in 4 year old boy, asthma relieved by salbutamol


A. FLuticasone
B. SCG
C. LABA
D. Oral Montelukast

68. 5 yr , symptom of severe asthma, admitted to hospital, nebulized by salbutamol & took oral
steroid, Now want to discharge ? (what to give)
A. Fluticasone
B. Oral Steroid
C. LABA

69. Male immigrant, Chest symp. ,Xray done . its apical tb, sputum +ve bacilli
A. INH 9 months
B. Isolation

70. Periorbital cellulitis, eyelid swelling, orbital redness, fever, he can move eyeball, what is Rx ?
A. IV fluxacill.
B. Oral AMox
C. IV Ceftriaxone
periorbital = preseptal cellulitis
mx:
1.orbital….admission  with  iv  flucloxacillin+3rd generation antibiotics
2. Periorbital cellulitis
a.  mild  (no  fever)….oral  antibiotics  (see  below)
b.moderate  (with  fever)…  admission  and  iv  flucloxacillin
c.  severe….  Admission  with  iv  flucloxacillin  and  3rd generation cephalosporin

71. UterovesicalJn stone 9mm, Mx ?


A. ESWL
B. PCNL

72. 24 yr female, no GI symptoms, anemia, IDA pic, Upper GI endo free, next ?
A. HB Electrophoresis if iron normal
B. Colonoscopy
C. Small bowel Biopsy if anemia

73. Capsule endoscopy recall

74. Male 70 yr want to do check up for insurance , Plasma Protien Increase, s.albumin 3 (n: 3.5-
5.5) , No symptoms ? (what to do next)
A. Serum immunoglobulin
B. Plasma Protein Electrophoresis
C. Bone Marrow Biopsy
MGUS might develop to MM ,, follow up yearly with electrophoresis
74. Young boy have asperger on sertaline , now irritability , cause ?
A. S/E of sertaline
B. Asperger Itself

75. Old Male, unconscious in bathroom, all labs normal, but blood sugar 9, Hx of prostatic
enlargment ?
A. Discharge & Let him see outpatient follow up
B. Admission
C. Insulin

76. Taxi driver, Sumptom of right side weakness for 10 min, resolved, CT normal, carotid duplex
also normanl, he is insisting on discharge to work as driver again.
A. Discharge but sign consent
B. Refer to neurologist to decide about driving
C. Tell him you can drive in 6 month
D. We need to assess driving after more Inv
E.tell him he will be ok and drive after investigation
77. Flu like Sympt. 2 weeks ago, now weakness in lower limb &dyspnea& sensory loss,
whichInv needed ?
A. CT
B. LP
C. MRI spine
D. Vit. B12

78. Spine X-ray bamboo sign, back pain, Now Can't bend, took PCM but not relieved?
A- Physiotherapy &Hydrotherapy
B - Rest
C – Methotrexate if sulfasalazine its right

79. Old male, backpain, Xray done, Little osteoarthritic changes in spine without compression
on spinal cord, he want opoids ?
A. GIve
B. Rest
C. Exercise
D. Tell him morphine is taken only in malignant pain.

80. 8 yr boy, fever 39, Hx of tonsilitis 1 week ago, Now fever n rash, toxic looks pale
1. Blood culture
2. Amox
Should be benzylpencilline or ceftriaxone (meningococcemia)
81. 80 yr female sudden loss of vision, tenderness over scalp, best Ix ?
A. Temporal Artery Biopsy
B. ESR
C. ANA
D. MRI

82. Girl fall, now fever and swelling? (what is next step and best step here )
A- Knee Joint Aspiration
B- Blood Culture
C- Xray Knee
D- Antibiotic

83. Boy 11 yr, BMI 22 , painful limp, impaired internal rotation, your Ix? -->Xray Joint (hip)

84. BMI 25 in boy, came for tonsillitis, you find that parents r obese too (old scenario)
A. Decrease watching tv
B.Wt. Reduction Schedule
C. Reduce Caloric intake

85. male attack of angina and hypertensive, 170/? ,s.cholesterol 5.5, most risk factor for IHD?
A-Angina
B-BP
C-Cholesterol

86. Peripheral vascualar disease & heavy smoker, S.Cholesterol 7.5 (increase), BP increase and
diabetic, after advice to stop smoking, what will decrease peripheral vascular dsz ?
A- BP control
B- Control Cholesterol
C- Control DM

87. old man back pain, no hypercalcemia, difficult in micturation, initial Inv ?
A. Bone Marrow Biopsy
B. PSA
C. Transrectal U/S
D. Serum Plasma Electrophoresis

88. Irreducible mass pain , no impulse cough, lateral to pubic tubercle?


A- Surgical
B- AbdXray
C- US

89. 6 wk boy was ok and increase weight, 1 wk H/O vomiting but he is well hydrated but not
increase weight since vomit started ?
A-GERD
B-PS if increasing vomiting
90. Middle age man ,↑↑ ca, his father 75 yr have ca ↑↑ but didnt take Rx, PTH ↑↑, what is
your Inv ?
A. Urinary ca
B Parath. scan

91. Pic of GOUT but renal impairment


A. Stop Allopurinol and give colchine
B. ↑ allopurinol and give colchine
C. Give allopurinol and give steroid
D. Stop allopurinol and give steroid

Q. Middle age lady,obese,bp normal, pigmentation on abd. ?

92. egg allergy Q

93. Preg, Pap 6 month ago was normal -->


A. repeat in 18 month

94. Dumping recall : Pt had ca stomach, did partial gastrectomy, after meal by 1 hour have
tachycardia, sweating and hypotension, Dx?
A- Early dumping
B- Late Dumping

95. Bariatric surgery history 6 weeks, now abdominal pain and vomiting
A barium

96. Pilonidal Sinus pic , asked Rx?


A- Fistulectomy
B- Marsupialization
C- Antibiotic

97. Builder confused, dehydrated, no fever or fever not sure, thrombocytopenia


A. Sunstroke

98. Boy, Mouth ulcer, rash in hand n foot, school exclusion ?


A. Until blister dry
Hands, food, mouth disease, coxakie virus, school exclusion until blisters dry
99. Boy was watching tv then attack of convulsion what to ask ?
A. First attack
B. Hx of cynosis during attack
C. Family Hx of convulsion

100. Female on Clozapine ,devloped tachycardia and flattened T wave, Inv ?


A- Echo
B- Troponin
C- Holter ECG

101. Female on venlafaxine ?


A-BP
B-LFT

102. BMI 14 in female, excessive exercise, you will do all inv, except ?
A- ECG
B- Electrolyte
C- Gonadotropin
D- LFT
E- Bone Density

103. Initial adverse effec after taking resperidone ?


A. Wt. gain
B. Hyperglycemia

104. Menopause on HRT, Initial beneficial effect ?


A- Insomnia
B- Depression
C- Cognitive function & Dementia

A child brought by his mother .she complains that he is always obese in the class. On
examination ht. 96 percentile and wt 97 percentile. What will you find in this child
a.increased bone age b. hypothyroidism c . hyper parathyroidism d increased cortisol.
Q.24 Lithium toxicity , levels of lithium given. Tx
A) Glucose
dialysis
C) NaHCO3
Hydration is the first line
The mainstay of treatment is fluid therapy. Give NS to enhance lithium clearance. First step, intubate, to
avoid aspiration as vomiting is very common in lithium toxicity. Next, give IV fluid and maintain that to
achive normal GFR and clear lithium. If lithium level is above 4 mEq/L in acute toxicity or 2.5 mEq/L in
chronic toxicity, initiate hemodialysis. Charcoal is NOT effective at all. If within first hour, whole intestine
irrigation may be helpful.

Q.26 Old lady admitted in hospital, wishes to change her will to donate all her wealth to
animal charity and asks u to sign as witness. What do u do?
A) Sign
refuse
C) Refer to guardianship
D) Assess mental state
Q.36 Retinal detachment diagnosed already in question with blurring of vision.Mx
A) Pilocarpine
Head tilt this is the management after attempting pneumatic retinopexy to keep the
gas buble in con tact with retinal hole.
C) Acetazolamide
Q.45 Old man with Parkinson started haloperidol one month ago now has tremors.
Treatemnt?
A) Decrease Haloperidol better if stoped
Increase Haloperidol
C) Start Sertaline
Q.55 Teenager attends a party last night comes to you with fever tachycardia high bp.
Cause?
A) LSD
MDMA amphetamine
C) Cocain

Q.72 A lady 40 years old present with sever occipital headache and examination
reveals high BP . What next?
A) MIBG
24 hour urine catecholamines next CT abdomen
.103 Chronic diarrhea with foul smelling feaces. Cause
A) Shigella watry, severe diarrhea, abdom pain, fever, tachycardia
Giardiasis
C) Amebiasis Bloody diarrhea
Q.106 A man hits his car with a van complains of chest pain. All blood tests and ecg
normal. What will u give him?
A) Sublingual nifedipine
GTN patch
C) Metoprolol
Q.108 A patient had CABG and on clopidegerl. Diagnosed of pharyngeal pouch.What
will you do?
A) Put him on waiting list its not emergency
Suregry now
C) Surgery now with FFP
D) Surgery now with platelets

an xray of small intestinal obestruction of 60 yrs old pt complain from constipation since 3
days , now he develops nausea without vomiting, abdominal pain , no flatus has hx of
appednictomy 15 yrs ago what is the dx?
A. ca colon
b. sigmoid volovulus
c. intestinal adhesions
Ureteral lithotripsy done 8 hrsback ,pt was given amoxicillin as prophylaxis , developed
fever and unwell , how to manage ?
a. Ceftriaxone
b. amoxicillin clavu
c. genta

Yasser abdelhamid recalls Mumbai 23/6/2016

1- An old postmenopausal pt. with bloody discharge from one nipple, most probable dx?

-­Benign duct papilloma

-­Ductal ca

-­Fat necrosis

-­abcess/Cyst

-­Paget’s  dis

2) 45years old man who had a hip replacement 2 weeks ago, present with sudden chest pain and
Hypoxia. O2saturation is 89%. He has aHx of renal disease. What's the appropriate investigation in
this pt

CTPA

V/S scan

ECG

CX-Ray

D-dimer

3- Most important measure for obesity regarding health aspect and risk Assessment

Bmi

Waist circumference

Waist hip ratio


-­4  9yrs with head trauma with swelling at site of trauma and vomiting once, headache until now..
mx

Discharge

Observe 4 hrs

Observe 24 hrs

CT head

35)year old boy with recurrent asthma attacks. What would you reccomend

Budesonide

Fluticasone

Sodium Cromoglycate SCG

Salmeterol/Fluticasone

6)patient with bradykinesia and postural instability. He is taking risperidone for visual hallucinations.
What history will b asked to get Dx?

A previous antipsychotic toxicity

B recurrent forgetfulness

A7) patient with ischaemic stroke from 3 days , with cholesterol level of 7 and BP 180/100. He is on a
spring tt ... Most appropriate management?

Start statin

Perindopril

Mannitol

8)child with signs and symptoms of meningitis develops seizures, generalised lasting for 5 minutes.
His lab values are normal except for sodium of 120mmol/l. What to do next?
IV mannitol

0.9% normal saline

3% hypertonic saline

Fluid restriction

Diuretics

9)ECGbradycardia with hyperacuteTwave  only  one  strip….Middle  age  pt  c/o  light  headach  and  
syncopal attack 3 episode in one month, bp 100/60 0r 90/60 ??he is on multiple drugs.which drug
comination will be tha cause of his problem? Actually confusing scenario not that simple.…

A.Amiodarone +Aspirin

B.frusimide+Aspirin

C.Amlodipin+Amiodaron

D.Aspirin+ Nitrtriglycerite

Hyperkalemia is caused by Aspirin plus metoprolol, aspirin plus ACEI or ARB,

10)  45years male in good healtWhat is the protocol?

A – FBS yearly

b – FBS 3 yearly

C – OGTT now

11)PREGNANT LADY 8 WEEKS WITH A FAMILY H/O GDM IN HER MOTHER which become diabetic
later on ... ASKING ABOUT MOST APPROPRIATE NEXT

A OGTT IN 20 WEEKS

B OGTT 26-28 WKS

C FASTING BLOOD INSULIN NOW

MISSING OPTION OGTT NOW…  


.80Post MI after 4 years came to your clinic for follow up visit, the patient is doing well. Still on diet
and regular exercise with complete normal lab work provided. He is on ace and aspirin only

reinforce diet and exercise

stop aspirin

add statin

complete in regular follow up

till him he don't need any more follow up

Adult patient is under dexamphetamine treatment. If stops TTT abruptly which could be the
symptoms

Aggressive

Anhadenia (=dysphoria=dissatisfaction)

Auditory hallucinations

Paraanoia

Homicidal ideation

Patient known case of schizophrenia presents with akanthia, he takes marijuana and is on drugs too.
He want to stop all drugs, what to do now:

tell him, if he stops then will have relapse in 2 yrs


givebenzotripne (anticholinergic) or propanolol

stop taking marijuana

Mesenteric  ischaemia…most  important  identifying  point-­

atrial fibrillation

,bloody diarrhoea,

abdominal pain

 .45pt come with bloody diarrhoea and lt iliac fossa tenderness on examination. he took antibiotic 1
wks ago. wt the dx?

a. pseudomembranous colitis b. diverticulitis (if old, go with this one)

A pic of Ct scan something in liver scenerio was pt went to outside country & then presents with
abdominal pain fever after some days he return wts the dx.

a.hydatid cyst

b.hepatocellularca

c.amoebic liver abscess

Picture of ctbrain , scenario of RTA with no significant neurological symptoms initially, then
developed headache and confusion, asking Dx

-­Extradural hge

-­Subdural hge

-­Intracerebral bleed

-­Brain contusion

But Depends on the CT

http://radiopaedia.org/articles/extradural-haematoma-vs-subdural-haematoma

Aboriginal mother brings baby for immunisation and leaves quickly no eye contact what will u do?
cultural and socio

Tell her to bring family member next time

Send nurse home visit

Call child protection

A pregnant lady sits on a needle in train came for advice?

Give antiviral drugs

Test serum HIV now

Serology test of needle

Serial check for HIV 3. 6. 12. Month later

A 26 yr old lady came at 18th week gestation with clear vaginal discharge, however she told that the
pregnancy was unplanned, fetal movement normal heart rate 144/min, her last pap smear was one
year ago. she

had no abd pain, what inv will you do

a.Fetalfibronectin test usually indicated between 22-35 weeks when there are signs of premature
labur

b. Low vaginal swab (usually to test for GBS at week 36 to know to give penicillin during labour)

High cervical swab

Urine microscopy and culture

e.Repeat Pap

Old age patient male with anemia, raised urea and raised serum creatinine with constipation. What
investigation needed for diagnosis: ( long scenario-I mentioned only abdominal one(

CT abdomen if other malignancies suspected.

Serum electrophoresis if MM

Thyroid function tests


Young female, primigravida came at 38 weeks pregnancy. LOP fetus position. Head is not engaged.
What is the cause?

Contracted pelvis

LOP position

Placenta previa

Cephalo-pelvic disproportion

55y old lady known with insulin dependent DM, has had right leg amputation 5yrs previously, now
blood pressure 175\90, normal regular pulse, BMI 32, LDL 2.8, s. triglyceride 4.5,hba1c was
8.5,fasting glucose was 9.5.Which of the following is important to keep her other left leg from
amputation?

Meticulous foot care

Control her blood pressure

Tight glycaemic control

Reduce LDL

Reduce her body weight

A  girl  with  recurrent  dizziness  vertigo  every  month.  What’s  the  most  important  step  in  
management?

EEG

Ct if child

Reassure it will be gone if teen should be dip-hallpike to diagnose it first


before reassurance!

ENT referral

manual worker with swelling of whole rtul compared to the other side after heavy work

by saw machine:
Muscle sprain

Subclavian vein thrombosis

Pic of a femoral hernia ..Mx

usg

surgical exploration

c.wait for 6weeks and do the surgery

child found by his parents wandering in the garden, he's disoriented, urine examination shows +++
ketones.  what’s  the  aetiology

a.DKA

ketotic hypoglycemia

.28a univ.student asking for citificate to  repeat  her  exam  as  she  couldn’t  sleep  the  night  before  due  
to anxity

a.refuse

b.refer to univ. medical unit

C.give to her if she is on anexitytt

Business man got bankrupt, many financial problems, complaining of late insomnia, lack of pleasure,
loss 15 kg from his weight through 3 months, any many other symptoms of depression, his family
concern  about  him  and  the  patient  refused  to  take  any  medication  as  he  doesn’t  believe  he  is  sick,  
he  admit  he  is  tired  and  exhausted  but  not  depressed,  what’s  this  called?

A Denial

B Reaction formation

C Depersonalisation

D Rationalisation
(78One of ur inpatient woman hv told her brother secretly that she has a plan for suicide, he doesnt
want her to know that he called u

inform police,

b)request bro to ask more abt his sister plan, by ian

c)ask the patient

.20obese lady comes to you with acne and some other symptoms,she has some visual
problems.what is contraceptive for her.

2 options for ocps

b.DMPA

c.Barrier

A child with signs and symptoms of meningitis develops seizures, generalised lasting for 5 minutes.
His lab values are normal except for sodium of 120mmol/l. What to do next?

IV mannitol

0.9% normal saline

3% hypertonic saline

Fluid restriction

Diuretics

#SCREENING

 42years Patient with colonoscopy negative, father when 55,maternal aunt when 62, what to do?

Fobt 2 yearly
colonoscopy yearly

colonoscopy 2 yearly

colonoscopy 5 yearly

mother of 1 ur child refuses vaccination...

.1follow her wish

.2child authority

.3persuade her abt benefits of vaccine

.4vaccinate without approval

 .5Send her for community health education

women 3 months after menuause has hot flushes. wants HRT. but doesn't want unpredictable
bleeding. what to give.

cyclical HRT

continuos HRT

Female BMI 18 with repeated treadmill exercise and looks herself every hour in mirror and changes
clothes three times per day, dx:

Body dismorphic disorder

Anorexia Nervosa

Obsessive compulsive disorder

34w pregnant female after MVA the itrus tender abd at 37 w ,all the vital signs is good ,even baby
CTG, What's next

US

blood match

Ct urgently

Fluid
 58years old man with painless slowly grieving mass in front of his left ear. He can't close his eye
properly . It's firm and seems fixe to the muscle.whats the appropriate Next investigation?  

USG

CT head & neck.

laryngoscopy

x-Ray

FNA cytolog

wat will u do to find out the association btw melanoma and nevus?

cohort

Rct

case control

case series

E-cross sectional

.case of a ptpersent with foul smelling cough,high fever with rigor.there was no xray but they give
the finding ,it was opacity in middle zone with air fluid level,after giving flucloxacilin what next

.1transpleural drainage

.2waterseal drainage

.3aspiration

 -­20Old age with manifestation of pneumonia, o2 saturation is 92%, not irritable

Ttt?

Amoxicillin clavulinic

Penicillin

Xray of pa and lateral chest with consolidation between right middle and lower lobe , with crepts on
lower lobe extending to middle of lung , dx?  

Middle lobe pneumonia

b. lower lobe pneumonia


c. lung abscess

d. irrelevant

seems to me lower lobe

#SURGRY

.after knee replacement surgery creatinin level increase plus oliguria.. next

IVP

B.retrogradepylogram

c.bladder us

d.other option with contrast

player injured in knee...initially there was pain..pain subside rapidly..btthr was effusion..after 1
mon..all goes nor...bt sometimes he feels locking his knee in ful extension & sometimes his leg gives
away..wts the dx

-­only ant cruciate ligament injury

-­med meniscus injury

-­lateral collateral liginj

-­med collateral lig+med meniscus injury

18)player injured in knee...initially there was pain..pain subside rapidly..bt


thr was effusion..after 1 mon..all goes nor...bt sometimes he feels locking
his knee in ful extension & sometimes his leg gives away..wts the dx -
only ant cruciate ligament injury
-med meniscus injury
-lateral collateral lig inj
-med collateral lig+med meniscus injury
The recall about the football player and knee giving away.

2) History of soccer player injured his Rt Knee .Sudden pain and swelling and locking of
Rt knee
. WOF is the cause.
A Ant. Cruciate Ligament.
B Lateral Collateral Ligament .
C Tear of medial meniscus.
D Medial collateral ligament.
the answer is AAAAA if locking before swelling subsides
1) Football player suddenly feel acute moderate pain in his rt knee while playing after hours
the pain subsides but effusion expanded after some day when the swelling and pain settled
sometimes he felt his leg is giving away some time his knee is locked
Dx
Med meninscial injury
Fracture patella
Med meninscial plus ligamentous lesion
Ant cruciate ligament...
AAAAAAAAAA
Football player has an injury while playing after that there was effusion n he feels his leg
is giving away n some other time his knee is locked dx??
Ant cruciate lig injury
Med meniscal lig
Med meniscal+ ligament lesion
Fracture patella
*if effusion was gradual…the answer is bbbb
*if effusion was sudden and the locking is after
effusion subsides….the answer is CCCC

Engineer 3cm lacerated foot injury, received tt just from one week for another injury. What to give now

Tt

Tig

Tt+tig

No treatment need

55year old patient with low folic acid, howel jolly body, stomatitis, normalbowel, weight loss, inv?

.1Small bowl biopsy

.2colonoscopy

.3vitb12

55y old lady known with insulin dependent DM, has had right leg amputation 5yrs previously, now
blood pressure 175\90, normal regular pulse, BMI 32, LDL 2.8, s. triglyceride 4.5,hba1c was
8.5,fasting glucose was 9.5.Which of the following is important to keep her other left leg from
amputation?

Meticulous foot care

Control her blood pressure

Tight glycaemic control

Reduce LDL

Reduce her body weight


 72    .52y old female with severe dyspnea and chest pain on climbing the stairs she is pale fatigue
HB=7.2 what most apropriarte for management decision

A-Oral iron

B-Parentral iron

C-Packed RBC

D-Fecal occult blood

female 14wks gestation come to ER because she can't void for some hours.she had USG at 10wks &
confirm gestation dates.Ut size 14wks.Abd exam tender mass midway below umbilicus.Dx??

Ectopic preg

Incarcerated ut

Ovarian cyst

Fibroid

A lady came 6 days before for routine PAP smear. After that first 4 days she had no problem. Last 2
days she is having lower abdominal pain, tenderness, fever. What to do for diagnosis?

High vaginal swab

Low vaginal swab

Blood culture

Endocervical swab

Investigations
A definitive diagnosis is difficult since routine
specimen collection has limitations in assessing the
organisms. Definitive diagnosis is by laparoscopy but
this is not practical in all cases of suspected PID.
•  Cervical swab for Gram stain and culture, PCR
( N. gonorrhoeae )
•  Cervical swab and special techniques (e.g. PCR
for C. trachomatis

old man who has 3 months history of dysphasia,dyscalculia,Right left disorientation,he is heavy
smoker and drinker.what is diagnosis

a.Multi infarct dementia


b.Alzheimer disease

C.brain abscess

D.gliblastoma

girl developed soreness down below 2 weeks ago after 2 times intercourse with her new boy friend,
she had mild dischrge and dysuria. now she is fine and her bf go away. wt the dx? (nothing
mentioned about weather itchy or not(

A.HSV

B.Candida

C.Chlamydia

Old pt with known bilateral chronic osteoarthritis, develop parasthesia and numbness in the lateral
side of leg below knee condition worse at night and improve after walking for 10 min, wt is the
important thing to examine in this pt?  

a Straight legs raising test

b Any tenderness in lateral condyle

c Any loss of sensation in the lateral side of leg

Woman with 2 days history of vison problem on left eye and pain behind
the eye,right eye is normal.Left eye with 6/12 and right with totally normal
vision, normal papillary reflex and normal movements of eye particularly
mentioned.  What’s  the  next  investigation?

CT head

Visual evoked potential

LP

In MS, pain is aggrevated by eyemovemet, and papillary reflex is decreased,


vision is decreased
Primigravida with cervix fully effaced and 2 cm dilated, the head is 2 cm
above ischial spine. When to do C/S?  

Dilatation 3 cm-head same position-intact membrane

Dilatation 5 cm-head 1 cm above spine-intact membrane

Dilatation 8 cm-head 1 cm above spine-intact membrane

Fully dilated- head 2 cm above spine-ruptured membrane

No change from the previous, the same

 .32Female with PMD (given the diagnosis not scenario(

Relaxation technique

Reassure

Clozapine

Fluxotine

Some lab

 -­20An X-ray of bamboo spine and mild pain asking about initial treatment?

Codeine

Paracetamol

Naproxen

Infliximab

 3year old boy with recurrent asthma attacks. What would you reccomend

Budesonide

Fluticasone

Sodium Cromoglycate SCG

Salmeterol/Fluticasone
young male who felll suddenly in the field of a match , without being touched,after a few MINUTES
he regained consciousness without any intervention and continued playing what is the case?  

a)- vasovagal attack

B)-Jacksonian seizures

C)- arrhythmia

 .22scenario of varicose vein and mi and on clopidegral started 7 days before.pt is on stent and
clopidegral..

refer surgery after 12 months and stop clopidegral 10 days before surgery

do surgery now

c.give platelets and do surgery

Pt on antisychotic and diazepam got some side effects which i felt mild , no fever, no rigidity but
asked why he had them:

A.neuroleptic malignant syndrome

B.Antipscyhotic

C.benzodiazepam toxicity
Mother came with her 9 years old boy complaints of progressive decline in his school records up on
which mother started to beat him by wooden spoonful and isolate him for hours for
punishment...mother has pat history if depression...what will you do

A.treat mother depression

B.talk with the child

C.children protection authority

D.send her for a course of right parenting

patient complaints of dysurea and blood in urine, but no fever, urine analysis shows ++blood, ++ pus
cells, bacterial growth is -ve,  what’s  the  aetiology:

TB nephritis

perinephric abscess

renal cell carcinoma

cancer bladder

 13Mother comes with 10 months infant with height 45 head size 25th at birth then now increase to
75 percentile now .. On examination child is mild hypotonic with open anterior fontanelle ,other
examination normal,feeding well, what to do??

Ct head

Check cmv

C.tsh

Usg head

Patient on sertaline , now abdominal pain , cause?  

Change to another ssri

Increase the dose

Another antibpsycotic
Postoperative second day agitated and confused what is next

pulseoximetry

ABG

CXR

electrolytes

Man with urinary retention mass above pubic symphysis, DRE done enlarged prostate, median
sulcus papable, catheter inserted, what next

Transrectal US

PSA

Abdominal CT

A 27 yrs. old lady with pelvic abscess scenario, breast feeding her baby and has allergy to penicillin,
what to give?

Vancomycin +gentamycin +metronidazole

Ampicillin +metronidazole

Doxy+ metronidazole

Ceftriaxone

Cephazolin

scenario of coal mine worker 55 yr old man has nocturnal cough and difficulty in swallowing with
long hx of smoking .. what is the next apprpraiteinv

endoscopy

boronchoscopy

ct chest
A patient had chronic limb ischemia. What is the sign most consistent with this?

Pain subsides on resting leg on a pillow.

Dependent rubor

Pain subsides on activity

Child with 48 hours rhinorrhoea and lots of absence among his classmates due to influenza like
illnesses, asking best to do? Check influenza 7 days off school (exclusion usually for 2 daysL) Wear
a face mask Oseltamivir (within first 48 hours only) Give vaccine (no prophylaxis for influenza)

Female with fracture in femoral neck, has written consent not to transfer to hospital if seriously ill.
You give morphine and she becomes drowsy, what next to do now:

transfer to local emergency unit

wait till she get more aware

familymetting

regular analgesics

Pt had colon cancer dukes c operated 1 yr ago and had chemo. What ix to do now. Pt is well. CEA
sent already.

CT colonoscopy

Colonoscopy

Abdct
 25years old woman presented with painful mensturation (no menorrhagia mentioned) she is taking
30 mcg OCP and wants to conceive in 12 months

continue OCP

NSAID and condom


increase the dose of estrogen to 50 mcg

mirena

depoprovera

 .5Man with swelling 1.5 cm in scrotum testes & cord palpated normally , no pain or urinary
symptoms , whats is the definitive treatment ?epididymal cyst

A)trans scrotal surgery

B)aspiration by needle

C)Reassure now & check it after 3 mo.

D)trans inguinal surgery

https://www.andrologyaustralia.org/testes-problems/scrotal-lumps-and-inflammation/

Surgical Therapy
Spermatocelectomy via a transscrotal approach is the primary operative intervention for spermatocele, and it may be offered
to any reasonable surgical candidate. Systemic anticoagulation and desire to father children are relative contraindications.

 65year old lady nulliparous comes with painless bleed from the nipple on 9ne side..

what is the diagnosis?  

Intraductal papilloma

Ductal ectasia

Invasive cancer

Paget disease

A 2 year old boy comes with history of harsh unproductive cough for 2 weeks. He is afebrile and all
physical examination is normal. What is to be done for diagnosis?

Chest X-ray

Nasopharyngeal aspirate

Bronchoscopy

Ct chest
E Serology.  

ECG of a 6 year old boy complaining of palpitations since four hours.Ecg was of wide complex
tachycardia  for  sure..it  wasn’t  like  typical  SVT.  He  looked  well  and  respiratory  rate  was  24/min.  What  
is the management?

IV sotalol

Immerse head in cold water and carotid sinus massage

Iv adenosine
#psych

a  patient  with  some  psychotic  illness  on  Dothiepin…asking  about  the  most  common  side  effect  of  
dothiepin (TCA)?

cardiacarryrhmia

hallucinations
tremor

insomnia

nausea and vomiting

 .859Famous recall of child born befor 37 week due to maternal problem, he is now 11 month and
can’t  sit  with  HC,  weight  and  height  in  10th  percentile

Due to prematurity

CP

Hypothyroidism

Autism

Familial

A patient on peritoneal dialysis who was going well suddenly declined to have dialysis. On
examination she was having slight temperature and mild tenderness in upper abdomen. What
finding you will most likely find in this patient?

A Disorientation

B Depressed mood

C Blunted affect

Pic of cleft lip baby born for 41 years old mother using carbamazepine and alcoholic and ask what is
the cause in this case

Genetic

Alcohol

Carbamazepine

Maternal age
Most teratogenic in last trimester?

a. anticonvulsant b. antipsycotic

case of old woman with vaginal bleeding, she didn't have sex 1 year ago, her menstraution stopped
11 m ago, what is the cause of her bleeding?

-­cancer endometrium

-­vaginal atrophy

-­follicular stimulation

-­cancer cervix

A boy came to you with history of ear discharge 2 weeks ago, his tympanic membrane looks dull and
retracted  and  there’s  yellow  discharge  from  his  ear.  What’s  the  nest  step  in  management?

Amoxicillin

Ear toilet

Pt on melatonin tt, police has arrested him from 2 days drunk, they take from him urine sample for
drug screening ...he comes today for you to prescribe for him melatonin again..what to do

Prescribe for him by Australian GP

Delay prescription after the drug screening result

 6years girl presents with urti and urine analysis has protienuria and rbcs, previously she has
episodes of urinary incontinence, what next to do to her

.1repeat urine analysis after urti subsides

Urine culture

Us

patient was brought to emergency room after a fight causing a stab with a knife in the chest
patientbut O2sat is decreased he is lobular ed speak, he has dyspnea , on examination he has
dullness to percussion , decreased air entry on both sides..

what should you do?  

Give O2 Next

Chest tube Best

Thoracocentesis usually indicated for pleural effusion when


diagnostic tests are required.

The treatment should ideally be:

- Give O2
- Rescucetate intravascular volume by giving fluid
- Chest tube
- Then after stabilizing the patient, assess the injuries.
- If massive pneumothorax is suspected, proceed with thoracotomy.

P.S for tension pneumothorax, (trachea shifting to the contralateral side, and resonance on the affected
side), place 3 sided dressing to prevent air sucking in, and insert needle in separate space to deflate the
pneumothorax.

Female who did not have sex before ,gonorhea is +ve, what is next

Repeat pcrgonorhea

Give Ceftriaxone single dose

#OPTHAL

 (17dacrocystitis image of rx asked

Warm compression

i.vantiobiotics

incision drain

#DERMA
Q.37 Melanoma skin care screening. 4o years old IT guys come for screening. What would be the risk
for  him?  That’s  all  given  in  exam.  Nothing  mentioned.

A)recent diagnosis of melanoma in cousin

B) outdoor working in early twenties

C)history of sunburn in childhood

family history of BCC

Midline post operative scar defect, what is the proper exam

.1stand and cough (for inguinal hernia)

Lie down and coup

.3actively to move his arm above the bed

Passively move his arm above bed

Passively raise his leg

INCISIONAL HERNIAS EXAMINATION. Lay the patient down, and put your
hand through the weakened area in his abdominal wall to feel the size and
shape of his hernia. It may be elliptical, or irregular, and he may have
more than one. Ask him to raise his head and shoulders off the couch
without using his arms. This will fill the sac and show you its true size.
If the hernia followed Caesarean Section, ask the patient to contract her abdominal
muscles. If her recti become taut, almost meet in the midline, and grip your fingers,
you should be able to repair her hernia without too much difficulty
Patient with palpitation and fear with idea that he will got HIV from stairs of the mole in spite that
his  GP  told  him  that  it  can’t  be  transmitted  by  this  way

GAD

Schizo

OCD
patient travelling to endemic area of TYPHOID,, what is ur advice

A Bottled water

B avoid vegtables and fruits

C wash hands

Pt has ra she is on chronic steroid tt 10m predinisolonedaily , now came with chest infection with
distress, with iv fluid what you will give

IV hydrochortisone

.2another options did not remember

 2years old boy found roaming last night in the hose with altered level of conscious and till now he
don’t  react  with  his  parent

DKA

Hypoglycemic ketoacidosis

Meningitis

Cerebral insult

 .16pt with neck stiffness, csf findings: lymphocyte 2000,(forget about glucose and protein) rbc 200,
temperature 37''c . wt next?

analgesia and observation.

iv a

1) Pyromania Recall :

Q. In australia almost every year there are sevral indication of bush fire usually it is usually by youn
people which of the following is true ?
A. Junenil pyromania
B. Due to accidant fire caused fire by yongster who get scard after the fire start to speed
uncontrolled
C. It has serious consequences
D. Yongsters like to ingnite and play with fire
E. It is done to hide crimes
Q. pyromania is seen mostly in youth generation. What leads them to do this?
1) to show heroism
2) childhood instinct to fire and fire fighting objects
3) as a display of revenge and anger

1. Diagnostic criteria for 312.33 Pyromania


2. These criteria are obsolete.
3. DSM IV - TR
4. (cautionary statement)
5. A. Deliberate and purposeful fire setting on more than one occasion.
6. B. Tension or affective arousal before the act.
7. C. Fascination with, interest in, curiosity about, or attraction to fire and its situational contexts (e.g.,
paraphernalia, uses, consequences).
8. D. Pleasure, gratification, or relief when setting fires, or when witnessing or participating in their
aftermath.
9. E. The fire setting is not done for monetary gain, as an expression of sociopolitical ideology, to conceal
criminal activity, to express anger or vengeance, to improve one's living circumstances, in response to
a delusion or a hallucination, or as a result of impaired judgment (e.g., in Dementia, Mental
Retardation, Substance Intoxication).
10. F. The fire setting is not better accounted for by Conduct Disorder, a Manic Episode, or Antisocial
Personality Disorder.

2.FTD –I forgot what I choose Both minus 7 and verbal fluency was there 

Q.wife brings husband with strange behaviour , recently thrown out of job and he makes silly jokes
and dresses badly etc what will help in detereming the cause
A- minus 7 test
B-copying pentagon intersection
C-disorientation to time
D-verbal fluency test

3.sarcoidosis q-skin biopsy

Q.A scenario which seemed like sarcoidosis young girl with painful nodules on both shins
from 6 weeks,had severe ankle pain and swelling from two weeks,she has dry cough and
exertional dyspnoea and her ace levels are high.Most appropriate investigation
a)respiratory function tests
b)skin biopsy
c)ankle aspirate
4.another sarcoid-loin pain hematuria-CT chest

Q. one chest xray given with reticulonodular shadow..pt complains of 3 months progressive
dyspnea..now intermittent loin pain ndhematuria..calcium raised..next
a. serum ACE
b. CT chest
c. urine microscopy and culture

5.EcG-inf  MI,two  “I  dunnowT  hell  is  it  ‘  ecg..pt  confused  on  multiple drugs I choose stop all and
reascess

Q.ECG Not clear one with many ectopics, the old one with biventricular failure & many drugs asking next
?
A. Stop all and reassess in 12 hours
B. Increase digoxin dose
C. Give furosemide

6.SBO,it was clear-adhesion obs

Q. SbOxray with nausea vomiting abd distension and constipation. No medical history, history of
appendicectomy 15 yrs.
a. adhesion obst
b. pseudo
c. ca caecum

7. old man,chronic constipation for 1 month only now with LBO features-no xray-I choose
Volvuluscz (I probably wrong ,Ca was there )

Q.old age pt with hx of constipation for 1 month , now presented for 24hr with colic abdominal pain
with  distention…Tympanic  sound  on  abdominal  percussion.  No  previous  H/O  surgery
No x ray given ….  Whats  most  likely  cause  ?
-sigmoid volvulus
-ca sigmoid
-adhesion
8. algerian lady with CA cecum

Q.70years Algerian lady with weakness and anaemia and is vegetarian lab inv given typical of
microcytic anaemia.cause
1.hook warm infestation√
2.ca cecum√
3.vit B12 deficiency
4.thalassaemia minor
5.diet

9. Young boy with celiac disease=Small bowel biopsy

Q.Young age boy with diarrhea, stool feel greasy, anemia was given , anti glidin and others were
normal, what next invest
a.ctabd
b. small bowel biopsy

Q.(related Q) A young girl comes with history of diarrhea , no blood. There is perianal excoriation.
What will lead you to diagnosis?
a) small intestine biopsy
b) biopsy shows flat atopy of intestine
c) biopsy shows granumatous

10. Two trauma scenario, One open flail chest ,at the scene –cover the open wound =opioid

PnemohemothoraxXray,really reallyconfusing,after 02 supplementation what u will bnxt most


appropiate step=
I ended up in morphine again,he seems to be in real pain

Q.Open flial chest, pain, distressed, cyanosed ? (most appropriate next)


A. Morphine (Most appropriate)
B. Needle thoracotomy
C. Cover wound (initial)

11. 2 CTG scene pathedine one


Q. preg and labour primi for 14 hrs cervic 4 cm given pethidine 300 mg 3 doses ctg showing hr
90(not sure but most prob )dec variability and early dec or something same as old recall asked
cause
1. prolonged labour
2. due to pethidine
3. caput moulding

Q. Ctg showed reduced baseline variability and variable deceleration (ctgnot given)
the mother had 4 cm cervical dilation within 14 hours , pethidine was given to her 3 times I guess
the fetal head is in the left occiput posterior. What is the cause of ctgabnormality
a. prolonged labour
b. sedative drugs
c.elevated resting uterine pressure
d. molding of fetal head

12.admit and observe one

Q.female ,39 weeks of gestation,200 ml painless bleeding. on palpation ,uterus is soft ,nontender. on
examination-cervix is 3 cms ,dialated with membranes clearly visible, what to do next-
a. amniotomy
b. admit and observe
c.urgent
d. c.s section

13.renal mass qs, 55 years man with hematuria ,mass 3 cm on ct, no partial nephrectomy I
choose left nephrectomy

Q. (Similar Q) Patint presented with hematuria.On USG there is 3 cm mass on the upper pole of right
kidney.whats your next step of MX
1.Percutaneous biopsy
2.Right nephrectomy
3.Review in 6 month

14.another one at 60-partial nephrectomy


Q. (Similar Q) 75 year pt with painless hematuriact abdomen showed 1cm solid mass in lower pole of
kidney.initial appropriate management
A percutaneous biopsy
B.partial nephrectomy
C.neprectomy
D.cystoscopy
Q. (Similar Q)Pt with abd. Pain with ct done suprarenal mass 5 cm wt to do
biopsy
partial nephrectomy
total nephrectomy
mibg

15.PKD with LUT obstruction,thr was no retroh\grade urethrogram or cystoscopy I forgot wht I
choose

Q. (Similar Q) Case of polycystic kidney disease on annual follow up presented with hematuria and
GFR 20 asking what of the following would help you to find if there is lower urinary tract obstruction
a.renal us
b. CT abdomen (not pelvis)
c. IVP
d. retrograde urethrogram
e. another test with contrast

16.16.CKD onmetforminramipril ,what to stop now?

17.cecal volulushemicolectomy q

Q. pt with hx of hemicoloctomy for sigmoid volvulus presented with moderate to sever abdominal
pain , distention, x ray was typical for cecal volvulus whats mot app treatment
-colonoscopy
-baruim enema
-hemicoloctomy
18.pt has spinal stenosis and on statin,now comes with hip muscle weakenss
wht to do
CPK
Doppler
MRI

19.catatonic scizo diagnosis

Q. man wondering in the shopping ,mall whole day ,peoples house door to door,sometimes raising
his hands and stand or seat aimlessly,dx
1.hepatic pathology
2.schizophrenia
3.manic

20. Post accident  husband  with  anger  anxiety  having  said  his  wife  didn’t  die  again  say  he  can  
hear dead wifs spirit calling hin
temazepum

Q. A young man brought to you.he is distressed,teary and angry on a someone who has killed his
wife by passing a vehicle over her when she was crossing road 3 weeks ago.he says he can see his
wife or listen her voice,also he says his wife is not really dead.
1.temazepam
2.olanzapine

Please CORRECT me if any wrong. Thanks - Chintan


Management of Optic neuritis in MS: give high doses of IV doxypredinisolone
(Oral is contraindicated!). If not effective enough, do plasma exchange. Last
resort is interferon based biological treatment.
CT of the chest is the initial test for retrosternal goiter
EKASHMI RECALLS-24th JUNE
An old lady lives by herself mmse 28 had fracture femur .. Family wants eveything to be done she also
has copd and has to be admitted icu many times a day .. This is her 6th time .Doesnt want further
treatment .. Lady is competent to make her decisions A .PsychoneurologicalassessemnetB. Arrange
meeting with the family (Recheck)( ansi felt ) C . Inform the hospital ethic group D .
80 year old lady has had multiple falls and chronic patient of COPD is tired of
being admitted multiple times in hospital, 2 days back she was working in her
garden and had a syncope episode and was admitted again in hospital, she
manages her finiances herself and takes care of her house, her mmse score
was 28/30 and she requested next time she get hospitalized she does not want
to be resuscitated. Her family disagree with her and request everything be done
to save her. What is the next step?
A, tell her son that you treat her
B, family meeting
C.  don’t  resuscite

---------------------------------------------------------------------------------------------------------------------------

Xray  of  fecal  impaction  ..  No  flatus  n  cinstipated  for  3  days  ..  History  of  surgery  30  years  ago…I  think  this  
scenario

[[ Full stem- x ray old Woman 71 years old with constipation and didn’t pass flatus since three days
abdominal distention nausea without vomiting with hx of appendectomy when she was 30 years with
a reducible inguinal hernia , what is the cause
a. colon cancer (my answer)
b. adhesional obstruction
c. obstructed hernia
d.fecal impaction]]]
---------------------------------------------------------------------------------------------------------------------------

Xray of sigmoidvoluvulus once operated comes wid central abd pain .. No stool flatus .Xray image rif was
empty nihaustrationscoffe bean btkying in the lifMangennt next appro asked Surgery hemicolectomy (ans)

.{{ Full stem- A patient who had a history of sigmoid colectomy (or previous similar attack am not
sure) for a previous episode of sigmoid volvulous, now presents with severe abdominal pain and
distension, the X ray Abdomen was given and the typical coffee bean appearance was seen. It was
volvulus! What is the most
appropriate management?
a. Hemicolectomy
b. Enema
c. Colonoscopy
d. NG tube}}

---------------------------------------------------------------------------------------------------------------------------

Farmer wid hemicolectomy liver usg given Triphasicct ..

---------------------------------------------------------------------------------------------------------------------------

Ecg 1st degree heart block Cease meto (ans) Cease meto start verapamil

i think full stem is {{ Ecg of 65years old man on hypertensive treatment & history of MI 3yrs ago ,now
all examination is normal except of soft blowing precordial murmur on the left border of sternum.rate
around 50/min.what to do ?
A. cease digoxin
B. cease metaprolol& commence verapamil
C. add warfarin}}}
---------------------------------------------------------------------------------------------------------------------------

Ecg looked like bigemenybt dig was nirmal was mobitz type 2 hr 35 Temp pacing (ans)!

---------------------------------------------------------------------------------------------------------------------------
Ecg a fib was on dig 0.125 mg and multiple drugs for heart failure htn Option increase dig Other options
irrelevant cant remember nth related to a fib though Dc cardioversion was tehrebt the patient was stable
hr n bp

I  think  very  old  recall  2015  ..  Full  Stem  ……………  {{{    7 yrs old lady presents with chest pain. She was
on nitroglycerine, diltiazem 60mg, digoxin 0.125mg, enalapril xxx. On examination she is found with
biventricular enlargement. ECG: (seem to be grade II A-V block & T wave inversions, ventricular
entopic, HR: 50-65/ min). what is the best management?
Stop all medications and review in 12 hours (I think this)
Increase the dose of digoxin
Commence of colvexin
Infusion of potassium
Commence on frusemide }}}}
---------------------------------------------------------------------------------------------------------------------------

Fourniers gangrene pic immediate management Debridemnet (ans) Antibiotics Blood culture
---------------------------------------------------------------------------------------------------------------------------

Molluscum pic ( school exclusion ) Not needed

---------------------------------------------------------------------------------------------------------------------------

Xray sarcoidosis was quite confusion there wws diaphragmatic herniation seemd so sth over the heart
protuding like a arrow presnetwid hematuria loin pain 3 episodes in a week Asked invetsigationi went
widctabd S calcium

I thnkthis Q similar to this stem …….{{  Sarcoidosis xray( bilateral reticular shadowing) with dyspnea for
3 years and episodes of loin pain and hematuria . what is the most appropriate management
a. calcium level
b.ct abdomen }}

---------------------------------------------------------------------------------------------------------------------------

?????

21 yr old went trekking 2 days ago presnetswid painful swelling "over teh inguinal ligament" nonn
reducible no fevr nth Femoral hernia Inguinal hernia Inflammedlykph node i ticked this Saphen varix was
confused betnn two
---------------------------------------------------------------------------------------------------------------------------

Mgus scenario nth on bone xary slightly elevated plasma proteins Managembet asked Bone scam Bm
biopsy Annual review of plasma proteins (went widthis )
---------------------------------------------------------------------------------------------------------------------------
Women on trifluphenazine well controoled left drug when she started developing tremors one yr back
Now thinks has intestinal worms whole over her body initial treatment asked Halo
TrifluphenazineQuietapine (ticked this Depot risperidone conta
---------------------------------------------------------------------------------------------------------------------------

Ligament holding the uterus -Uterosacral ligament


---------------------------------------------------------------------------------------------------------------------------

Mother worried child 2 yrs non toilet trained but waked at 12 months says mum dad undresses rest
normal Normal child (ans )
---------------------------------------------------------------------------------------------------------------------------

11 months pre mature delivery ( cant sit unsupported had icu admission Diagnosis Dev dlay due to
prematurity CpansHypothyroid
---------------------------------------------------------------------------------------------------------------------------

Old man widgleason 4 rest normal Radical prostatectomy


---------------------------------------------------------------------------------------------------------------------------

Man widstmptoms of bph comes with acute retention dre reveals firm prostate wid clear medina sulcus
after catheterisationPsaBm biopsy Transrectalusg (ticked this)
---------------------------------------------------------------------------------------------------------------------------

Man from afghan bloody sputum chest crackels sputum postive for afb next step Ct chest PcrInh for 6
months Islolation in a negative pressure room( ticked this though negative pressure made me a lil
doubtful )
---------------------------------------------------------------------------------------------------------------------------

A female 3 montshwristbjoint comes widdiplopliahemsturiahtn wat inx to reach diagnosis Esr Ana Rf S
urate Ancai did this
---------------------------------------------------------------------------------------------------------------------------

Charcot marie investigation for diagnosis Child cant climb stairs mither had same wid arch feet reflexes
normal lost muscle bulk Nerve comduction (ticked this Ecg Ct
---------------------------------------------------------------------------------------------------------------------------

Accidnet many ribs farcture patient is severlydyspnei severely cyanoses nth else mentioned one liner ..
Wat will u do at accident site Cover the wound Needle thoracostomy( wentwid this since was severly
cyanosed nth mentioned abtbo pulse ) Morphine
---------------------------------------------------------------------------------------------------------------------------

Anitherct chest given pneumonwws there btni tracheal devuation maintaining spo2 96 innsevre pain inn
breathing Needle thoracistomyMorphinen(at accident site ) went wid this
---------------------------------------------------------------------------------------------------------------------------

Pilonidal sinus ..Recurrent Fustulectomy (ans  )Marsupialisation  Antibiotics  …HB 3.008 JM 382
---------------------------------------------------------------------------------------------------------------------------

Cleft palate pic Cause alcoholic mother on carbanazepineGenteic (answer)


---------------------------------------------------------------------------------------------------------------------------

Mmr n egg allergy

---------------------------------------------------------------------------------------------------------------------------

Ligament holding uterus Uterosacral ligament

---------------------------------------------------------------------------------------------------------------------------

Antenatal women at 39 weeks wid 3 cn cervix 200 ml bleeding uterus softbnon tender vitals stable fhr -
110 .. Wat will u do ??

Admit n observe(old scenario)

Send home

Cs

Amniotomy

---------------------------------------------------------------------------------------------------------------------------

Alcoholic abo lady seldom sees doctor brought by her friend has features of dcm(dilated
cardiomyopathy)

Mr diastolic murmur is dyspneic wat is the most appropriate u can do for her managemnt??

Estsblish a sustainable relatuonwid her inchose this

Echo (my answer)

Ecg

Xray

---------------------------------------------------------------------------------------------------------------------------

Asthmatic child history of eczema motehr ha shay fever father smokes best preventor

Chromoglycates chose this

Fluticasone

Budesonide
Oral predni

I think this stem{{ 5 years old boy with cough from 12 months before, was on salbutamole n also tkn
oral prednisolone.. he has an
eczema history, both parents are smokers,( typical asthma scenario) which one is the best
medication for prevention
which  is  used  in  “INHALATION  MODE”?
a.  monteleucast0……oral
b. Chromoglycate
c. Salbutamol
d. Fluticasone
e. Salmetrol }}}}}

---------------------------------------------------------------------------------------------------------------------------

6 yrntrelievd by 6 puffs salbutamol by spacer

Wat next 12 puffs

Nebu salbutamol

Oral predni

Inhaler fluti

Intubation

---------------------------------------------------------------------------------------------------------------------------

Child asthma has nocturnal n exrcise inducesymptoms . Kept in hopital 1 day treatment with oral predni
n nebu salbutamol .. Wants to discharge tomorrw .. Most appro in the ongoingbtreatment??

Inhaler fluti

Oral prednisolobe

Montelukast

---------------------------------------------------------------------------------------------------------------------------

Driver alwys late for works wahses all keys nncounts them .. Thrown from job

Obsessional behaviour

Compulsary rituals
Mannerism

Full stem……{{  A truck driver was fired from his job because of always being late. He makes sure that
he wash all the keys one by one so that nothing bad will happen on the road.
What does his behavior denote?
A. Obsession
B. Compulsion
C. Mannerism
D sterotype }}}

---------------------------------------------------------------------------------------------------------------------------

Pregnant lady husband beats twice n doent leave marks tells u abt it next

Urgent admisson

Call police

Send her to police

---------------------------------------------------------------------------------------------------------------------------

A man wife died when hit by a car 2 weeks ago tearful n angry at times cant forget the event cant sleep
properly presents to u wat mext

Temazepam (ans )

Olanzapine

Haloperidol

---------------------------------------------------------------------------------------------------------------------------

Old day post cystoscopy wid epidural agitated accusing of sexual assault on her

Olanzapine

Risperidone

Lorazepam

Diazepam

---------------------------------------------------------------------------------------------------------------------------

Old day presents wid painless hematuria three episodes urine rme shows blood +++ wat next appro

Cystoscopy

Ct abd
Ivp

Xraykub

---------------------------------------------------------------------------------------------------------------------------

One man wid polycystic kidney disease on annual review wat investigationn to detect the lower urinary
trctobstructuon

Cystoscopy

Abdct

Contrast ct

Usg

Retrograde  pyelography…

Full StemCase of polycystic kidney disease on annual follow up presented with hematuria and GFR
20 asking what of the following would help you to find if there is lower urinary tract obstruction
a.renal us
b. CT abdomen (not pelvis)
c. IVP
d. retrograde urethrogram
e. another test with contrast

---------------------------------------------------------------------------------------------------------------------------

Young child urti comes with features of urti urine shows hematuria protein + cines fta two weeks urine
analysis unchanged shows blood dx ??

IgA nephropathy (ticked this )

Psgn

Transient benign hematuria

---------------------------------------------------------------------------------------------------------------------------

Man comes with progressive bluriing in the left eye fir 2 days no headache no ither features wat invxnxt
??
Mri

Ct head

Ct angio

Esr

Stem {{ 80 year old man present with 2 days h/o blurring of vision.there was no headache and
opthalsmoscopycant be done due to cataract .what is next inv?
1.esr
2.lp
3.carotid artery duplex
4.mri }}}

---------------------------------------------------------------------------------------------------------------------------

Boy post urti has bl leg oedema bp140/90

Hematuria protein + wat do u do fir mangement

Presnislone

Antibiotics

Iv fluids n Lasix (can’t  be  because  of  fluid  in  the  option)

Iv hydralazine (not the best, but the only possible option here)

Stem {{ Child with URTI and after two days presents with BP 140/90, edematous face and bilateral
ankle adema, urinalysis show rbc++,protein++, aso titer raised, what to give him:
a.Iv hydralazine
b.furosmide+iv fluid
c.steroid }}

Post strep GN is treated by 1. diuretics and fluid restriction 2. Ace inhibitors and CCB

---------------------------------------------------------------------------------------------------------------------------

Dm patient history of dvt 4 yers ago presnetswidsweeling red feet fever 38 c

Wat nxt investigation

Doppler usg

Blood culture

Xray ankle
---------------------------------------------------------------------------------------------------------------------------

15 gal came by herself doenst want to tell her oarents has been feeling low no suicidalmthoughts .. No
interest in stidies ..Mangembet

Cbt

Female adolescent group (ans )

Relaxationntherapy

Fluoxetine

Mirta

---------------------------------------------------------------------------------------------------------------------------

In venlafaxine wat wudmonitor to increase the drug BP

---------------------------------------------------------------------------------------------------------------------------

Man wid femur fracture ca normal vitamin d reduced

Testesteronethrapy

Calcutriol

Iv calcium

Alendronate 70 mg weekly (ticked this )

Similar Q: 1)An 80 year old man came to you following surgery for fracture neck offemur after a fall.
Routine laboratory tests done all normal. Morning testosterone was 8 ng/ml (8-50 is normal) and
calcium was normal, Vitamin D however was very very low (0.12). What will you give for Mx?
A) Alendronate 70 mg once weekly
B) Calcitriol 0.25 gm daily
C) Calcium 1000mg with Vitamin D 600 IU daily (I think, recheck)
d. testosterone replacement.

2)80 years old man with fracture femur. He takes long list of drugs amongst is oral steroid intake
every period to control his COPD. His investigations showed low vit D and calcium, low normal
testosterone. Next step?
Give alendronate
Give calcium
Give vit D and calcium
Give calcitrol
Give testosterone

---------------------------------------------------------------------------------------------------------------------------

Pre op for hystercetomy calcium is 1.8 otehrs normal afta one week still calcium low wat will u do

Iv calcium

Oral tabs

Injvit k

Stem ………a woman was admitted to the hospital for elective hysterectomy. the pre clinic test her
calcium level was 1.8 and now it is still low (nothing mentioned about symptoms)
what is your management
a. iv calcium
b. calcitriol
c.  ca  carbonate……………..    

SimialrQ : 40 yr woman came for precholecystectomy visit, she had thyroidectomy 5 yrs ago her ix
showed Low ca 1.9 Low vit d Tx?
IV ca
Oral ca
Calcitonin

---------------------------------------------------------------------------------------------------------------------------

Fractur femur old lady says has easy bruising bt platelet count n morphi normal has stent on clopidrogel

Urgent surgery

Give ffp n surgey

Give platelet n surgery

Wait one week

---------------------------------------------------------------------------------------------------------------------------

One man feras getting in train lost 5 kgs is depressed no interest cant sleep at night management initial
Realxationexrcise

Cbti went wid this since had sais initial bt confused

Fluoxetine

---------------------------------------------------------------------------------------------------------------------------

Old lady lost 10 kgs severe depression says her instestinerittendoenst eat brought by son .. After giving
iv fluids. Most appropriate management

Ect

Escitalopram

Fluoxetine

Cbt

---------------------------------------------------------------------------------------------------------------------------

Post partum mild depression baby 1 month n she is separated from her husband most imp un her
management??

Involve her husband

Cbt

Fluoxetine but could be this if sever

Send her to some meditation centre

---------------------------------------------------------------------------------------------------------------------------

Pediatric growth curve weight on 96th percentile

Mildly overweight (ans ) ?

Obse

Normal

Underweight
---------------------------------------------------------------------------------------------------------------------------

Cvs risk chart

---------------------------------------------------------------------------------------------------------------------------

9yr bmi 18 advice

Give only fruits

Stop tv watching

Weight reduction program (ans )

---------------------------------------------------------------------------------------------------------------------------

Went to papua guinea 3 weks back 10 days agi started diarrhoea .. Has weight loss n abdpain ??

Etec e coli

Giradiasi (ans )

Amoebiasis

Camplyo

Shigello

Need more detailes

---------------------------------------------------------------------------------------------------------------------------

This one was cifnusing lady 3 weeks gradual headache then she develiped mild neck stiffnes .. No other
features on csf

Monocytes increased

Protein decreased

Glucose decreased

Echo

Mycotb

Listeria

Cryptococcus neoformans( out of exclusion ticked this was confused widtb)


Ankylostomacanneberis or sth

Similar old scenario: Pt, T-37, mild neck stiffness,3 wkheadache. Lp reveals:
PMN 1*10^6(0)
Mononuclear cells 20*10^6(1-2)
Protein: Increased
Glucose:Decreased.What’s  the  most  likely  organism:
A.Echovirus
B.ListeriaMonocytogens
C.MycoplasmaPneumoniae
D.Mycobacterium Tuberculosis (no suggestive history)
E.Cryptococcus monocytogens (in HIV)

---------------------------------------------------------------------------------------------------------------------------

Young couple cant conceive for two years female ovulation usg all normal intercouse on evey alt day
..male semen count 15 million abnormal 75% motility 50% cause

Tubal cause

Semen abnormal

Short luteal phase

Bad timing

---------------------------------------------------------------------------------------------------------------------------

Had that pof scenario fshlh increased oestrogen reduced 3-4 cyst on usg-Dosent want to conceive
then OCP& Want to conceive then HRT

---------------------------------------------------------------------------------------------------------------------------

Another similar wid hypothalamic fshlh normal oestrogenskightkybreduced on usg 3-4 cyst..dysfunction
of unknown origin
Stem.{{ 24 yr female had 2 babies before, amenorrhea 2 years, all normal hormones (prolactin,
testosterone,FSH,LH) , estrogen 70, US 3-4 multiple cyst, Dx ?
A. PCOS
B. Hypothalamic dysfn.
C. Pituitary Adenoma
D. POF
Same above Scenario but ↑↑↑ FSH and ↓↓↓ estrogen, Dx?
A. PCOS
B. Hypothalamic dysfn.
C. Pituitary Adenoma
D. POF }}}}}}}}
---------------------------------------------------------------------------------------------------------------------------
Young gal nullipara wants to conceive in 12 months dysmennorhoes despite 30mcg ocp

Nsaids n condoms (ans ) HB Q

Increase ocp

Pop

Iucd

---------------------------------------------------------------------------------------------------------------------------

Pethideneinj slowing of heart rate n ctg shows reduced baseline variability n increased variable
decelrationsprimi 14 hrslabourwid 4cm dikatation

Cause

Prilongedlabour

Caput

Incordinate contraction

Pethidine (ans )

Please CORRECT me if any wrong. Thanks - Chintan


RECALLS 21ST JUNE 2016

HI  guys  these  are  some  recalls  ….I  think  moslty  from  last  6  months  but  with  different  opionsso  it  will  
change  answer  too  so  don’t  rely  only  on  recall  ans…I  also  got  some  new  questions  too  .I  Will  update  
later.Be carefull about time Mx most of the questions with long scenario ,few only one line quest

7. FEMALE PATIENT AGE 60 YEARS CAME TO CLINIC FOR POSTMENUPAUSAL OSTEOPOROSIS


ASSESMENT AND WORRIED ABOUT FOR THIS AS HER MOTHER HAS ALSO HAVE HISTORY OF
SUSTAIND # OF HIP IN OLD AGE .WHAT WILL BE THE MOST IMP FACTOR OR HISTORY FOR DOING
BONE SCANE IMMEDIATELY?
E. EARLY MENUPAUSE AT AT AGE 35YEARS
F. SMOKING HISTORY BEFORE AGE 25 YEARS
G. WAS ETHELTES AT AGE 18-20 YEARS AGE
H. FAMILY HISTORY OF OSTEOPOROSIS
8. 2 YEAR OR 18 MOTNTHS ?CHILD BRING TO ER UNWELL SINCE LAST 36 HRS ,IRRITABLE,
LETHERGIC, NOT FEEDING WELL,COUGH, FEVER 38,DYSPNOEC ,RR 40-60 ,SPO2 88, ON CHEST
REDUCED AIR ENTERY TO LEFT SIDE OF BASE, TRACHEA SHIFTED SLIGHTLY TO RIGHT SIDE.WHAT
IS THE INITIAL MANAGEMENT?
E. STEROID
F. BLOOD CULTURE
G. IV FLUXACILLENE
H. ? ONE MORE ANTIBIOTIC

(NO OPTION FOR O2,NEBULIZER,INTUBATION ,IV FLUIDS)

9. CHILD WITH ASTHMA HISTORY SIMILER TO OLD RECALL ON SABA 4 HRLY, AND ORAL
PREDNISOLONE TAKING SABA FOR AS A EXERSISE PREVENTER,LONG SENARIO ASKING FOR
MANAGEMENT FOR ONGOING SYPMTOMS?
E. SABA
F. ORAL PREDNISOLON
G. CICLESONIDE
H. BUDESONIDE/
10. SAME  RECALLS  9  MONTHS  BABY  BRONCHEOLITIS  …RSV  CAUSE
11. MMR AND EGG ALLERGY SENARIO

12. 25 yr old female complain of Raynaud's phenomenon in cold,also having edema of bl


feet,,bibasal crepitation on lung bases ,heart on CXR silhute appearance .ana+, ENA neg..cz
A-primary Raynaud's
B-cold agglitonin antibody
C-sle
D-limited sclerosis
7.ECG of HB Mobits type 2 pt on many medications ,indapamide, verapamil, prendopril ,
aspirin….. presentwth light headedness , wt to do first?
a-valsalvamanover
b-cease verapamil
c-temporary pace maker
d. cease aspirin

8.adult male with pic of U/s-there is complex mass, swelling of scrotum,what to do


a.biopsy
b.CEA
c.alphafeto protein
d.surgery.
9.mother present to you due to violence of her son which reported he is addict & drug dealer
,what to do
a.arrange admission of son to drug rehabilation
b.Insist her to inform police
c.inform social media
d.inform school

10.68 year old female have menopause at 55 , present with purulent brown-greenish vaginal
discharge for 6 day , she is sexually active, what is the cause , previous pap smear normal
a. Chlamydia
b.endometrial Ca
c.gonnorea
d.ovarrain ca

11.Pic of pyogenic granules ma WD snerio


Trauma to finger 6weks ago aftr that gvn painless lesion is increasing in size,tx asked
Cautry
Surgecal excision
Cryo therapy
Incisional biopsy

12.Pic of middle lobe pneumonia in 50 yr old male with fever,cough and malaise from 5 days,
tx asked
Augmenton
Benzyl penicillin
Flucloxacillin

13Mother comes with 10 months infant with height 45 head size 25th at birth then now
increase to 75 percentile now .. On examination child is mild hypotonic with open anterior
fontanelle ,other examination normal,feeding well,what to do??
Ct head
b. Check cmv
C.tsh
D. Usg head

14.a female had a normal delivary of a normal weight baby , there was perianal tear that was
small and didn't needed suturing , after the delivary she noticed that her locia didn't flow well
in the following days , and at Day 4 she presented with heavy bright red bleed with fever of
37.5 what is the reason for fever
A)endometritis
B)cervical tear
C)infection with perianal tear
D)DVT
15.Male  60  0r  65  ?withho  backache  had  sudden    vertebral  #  at  T8/T10  …after  lifting  of  heavy  
things..(.no history of ca)
ESR 104, HB 10.4.,low plt.144 wbc normal., ca .normal 2.4..What will do for to rule out
diagnosis?
A)bone marrow
B)PSA
C)testeron

16.A man with rheumatoid arthritis and HTN is on thiazide ,chloroquine ,aspirin ,enalapril,
feeling lethargic and fatigued.
C/F pale, HR ~80bpm BP normal
Lab inv: hb-low, leukocyte-low, platelets-low
What is the cause:
A. Chloroquine
B. Thiazide
C. Aspirin
D. Enalapril

17.bamboo  spine.  Pic  pt  on  paracetamol+  physiotherapy  for  ….long  time  but  not  getting    
better .what will next step Mx?
A.physio
B.mtx
C.sulfasalazine
D.steroids
E.infilixamb

18.female ,pg 18 wks , sudden frontal headache, bp 80/60, pulse 90, on exam: confused and
drowsy, heart normal what inv:
A.Mri
B.Us
C. Echo
D.CTG

19.ECG bradycardia with hyperacuteTwave only  one  strip….Middle  age  pt  c/o  light  headach  
and syncopal attack 3 episode in one month, bp 100/60 0r 90/60 ??he is on multiple
drugs.which drug comination will be tha cause of his problem? Actually confusing scenario
not  that  simple….
A.Amlodipin +Aspirin
B.frusimide+Aspirin
C.Amlodipin+Amiodaron
D.Aspirin+ Nitrtriglycerite

20.youngpt 2weaks pain in shoulder with limited abduction and flexion of shoulder after
lifting  of  heavy  things  …what  is  the  initial  Mx?
a- paracetmol b-MRI c- intrarticular Cortisone injection
21.parents brought 3 yrs old son with c/o room spinning around with no loss of
consciousnesses and lasting for 1-2minutes every month for last 3 months and subside
spontaneously.No h/o headache or vomiting. Ear examination normal .What will you do ?
A.EEG and CT scan head
B.Audiology and ENT referral
C.Reassure due to BPPV that it will resovle later.

22.pyloric stenosis pic from anthology. Pt uop 50ml/hr asking what to give with iv fluid in 24
hours?
a.20mmolkcl
b. 50 mmmolkcl
c. more than 50 mmolkcl
d. no kcl required!

23. 60 yr male pt with 2 days ho intense photophobia with blurring of vision. .vision also
reduced on left side asking diagnosis?
A.conjuctivits
B.gluacoma
C.keratitis

24..Pic of BCC asking diagnosis


.
25. ECG AF, Brady , complete HB, VF,
With different senario on multiple drugs .

26.old recall old man found collapsed in garden with temp 33.4 + hypothyroidism + heart
block asking cause of collapse? ((((cardiac arrest))

27.Pt on chemotherapy receiving treatment from cvs line , then she developed neck and
facial swelling gradually . What is the next imp mx?
A.Ctchest b. CT head & neck C. Limb angiography

28.ECG.  VF….  Pt  brought  in  ER  unconsiuos  ,CPR on going….what  will  do  immediately?
A. Defib
B.Intubation

29. 60 yr male pt with 2 days ho intense photophobia with blurring of vision. Pain behind
left  eye….vision  also  reduced  on  left  side  asking  for  next  investigation  ?  
A.CT B.MRIC.Carotid Doppler

These are some recalls may be it will help for  coming  exam….sorry  if  any  mistake  I  am  not  very  
good recaller….i  will  upload  more  recalls  once  I  memorized  all  hopefully

Please  remember  in  prayers…  best  of  luck  to  all  ..
41 years old man has headache and is accusing neighbors as they use
insecticide excessively. He mentioned he had frequent trouble with
them as they are very noisy.he changed his living place two times
before as he was unlucky with his neighbors

Thinking that they hate him and want to harm him .other persecution
thinking was there.

a delusion

b.depression

c. schizophrenia

d. Paranoid personality disorder

old man coming from travel presents with increasing chest pain and
sweating with hypotension and ECG done it showed unspecific ST
elevation in lead 2 3 and avf with rbbb (rsR pattern) inverted T in lll
and avf what is the diagnosis ?

a- Acute inferior MI

b- PE

C lbbb

D pericarditis

41 years old man has headache and is accusing neighbors as they use
insecticide excessively. He mentioned he had frequent trouble with
them as they are very noisy.he changed his living place two times
before as he was unlucky with his neighbors

Thinking that they hate him and want to harm him .other persecution
thinking was there.

a delusion

b.depression

c. schizophrenia

d. Paranoid personality disorder

Hemochromatisis pt cofused , abd. Distenton ,afebril , mild


tenderness with alt and ast too high

A- albumin to treat hepatorenal syndrome in liver failure due to


hemochromatosis

B-lactulose .as a laxative to help empty the intestine which is supposed to improve mental
status, here the patient is confused which reflects Hep encephalopathy

C-spirolactone This is given to treat ascites resulting from liver failire


In hemochromatosis, we generally treat the disease by phlebotomy to get rid of excess iron and
chelation. We also treat the complications of it as a result of organ failure.

Pt wit pruitius ,ithing,afebrile with jaundice ,small mass in upper lef


quadrent ejection 20% what to do

A-cholycwystectomy
B-ercp

C- precutounous transhepatic pancangiography

Pt with hemoptysis and cough afebrile for 3 years with infilterate


lower zone and x ray wierd

A-lobar pneumoia

B-bronchiactesis

C-chronic bronchitis

D-lung abcess

C or B according to x ray
Bronchiectasis

chronic bronchitis

Pt with chest tightness dyspnea recurrent syncope what from this ecg
( give 5 ecg )

A-complete heart block

B-v.f
C- a.f

D-v.tac not sure

E- hyperkalemia

Pic of biceps tendon rupure

Pic of barret oesphages with metapasia no dysplasia

A-endoscopy after 24monthe

B- endoscopy after 12 m

Pt after hip operation by 5 days come with pulse 120


,agitation,afebril

A-duples of calf ms

B-ctpa

C-vq scan

Pt with alcohol bing and liver impairment

A- diazepam

B-lorazepam

Pt alcohol bing filterate with is huspand ten with you after a while
become agitated,rritable
A-diazepam

B-thiamn- this is answer but confirm it again

Pt not comfortable in sex wth husband, ttt by clotrimazole but now


recurent white discharge ,on exam redness on genital area with small
ulcer

A-herpes pcr

B- vdrl

C-vaginal swap

Pt with candida ttt by clotimazole ,burpt recurrent several times ,


bl.sugar 5 , invest needed

A-vaginal swap

B-vaginal culture

C-blood culture

D-glucose tolerance test

E-hiv antibody test—not sure E or B

Pic of dypturin contracture bl.sugar 9 , best to do before specific ttt

A-glucose tolerance test

B-us
Pt with ankle swelling, afebril, stifness for an hour relieved by walk
,xray show periarticular osteopenia and narriw space, other foot joint
is normal

A-osteomylitis

B-rheumatoid arthritis

C-sle

D-septic arthrutis

Pt with mouth ukcer, arthlagia, stiffeness of hand

A-sle

B-sarcoidosis

C-rh.arthritis

Osteoarthritis

Pt leave her job with depression symptom and say i dont want to live
,with alot of suicide attempt

A-dilectal therapy

B-drama therapy

C-aversive behavior modification


Pt on clozapine and ok now except some idea of reference, complain
of sleep 15h per day

A-add respiredone

B-addmethylphenidate

C-increase dose of clozaoin

D-decrease dose of clozapin

Genital warts in preg. Lady after cryotherapy increased

A-aldara cream

B-surgical excision of labia

C- surgical excision ofbwart by diathermy

No optionfor leave it until delivery or again cryotherapy

48 male all normal ask of screening of dm

A-fbs/3y

B-fbs/1y

Fobt positive in 52 y male

A-fri ( something ) colonscopy


B-fixed sigmoidescoy

Teacher complain of studnt not regulral in his lessons , leave school ,


mother say he go ipto school in time but he became agressive against
his young brother

A-epseparation anxity

B-tuprency

C-conduct disorder

17year with 2ry sexual character with no lower abd pain

A-us

B-fsh

Pt with dm ,b.p 135/90 drink 14 standard cup alcohol in saturday,


what to do for control htn

A-24 h monitor b.p

B-home b.p monitor

C- advise decrease alcohol intake


20 y pt come for invest for insurance company urine +1 no heamaturia

A24h urine protien excretion

B-repeat urine dipstik on waking

Pt withirregular pulse and abd pain , take digoxin for another reason,
cause

A-digoxin toxicity

B-mesentric embolism

Pt with sudden headache but improved in 2 hours with neurological


deficit , no histoY of trauma, afebril

A-subarchnoid hge

B-cerebral abcess

C-exrradural hge

D-subdudal hge

Ct of ishemic stroke asking diagnosis

Pt after surgery now well, start drink some fluids but his lap Na
110(low) k 2.5( normal 3.5_5)

A-hypertonic saline

B-k replacement
No option for normal saline

65y with painless haematuria, occasionl loin pain ,what is the invest
of great value

A-cystoscopy

B-urineanalysis

C-ct abdomen

Pt with left iliac pain, rebound tenderness temp37.5, intial invest

A-us

B- ct abdomen

Pt with otoache on exwm ear full of wax and there is irregular


swelling below mandiple in rt

A-repeat otoscope after irregation of the ear

B- ct head

C-exam the tongue

Chest stab wound b.p 90/60 dullness on percusion , decrease breath


sound bilateral, trachea shifted to same side of wound intial
managment
A-x ray

B-ct

C-needle thorocotomy

No option fot i.v fluid or o2 or chest tube


MONA ,Newdelhi ,24 june

1- paient with neck stiffness, csf findings: lymphocyte 2000,monocyte, rbc 200,
temperature 37''c . wt next?

a. analgesia and observation.

b. iv acyclovir

c.cefotaxime

2-
patient with hypertension c/o recent painless loss of vision for 3hrs.visual acuity
reduced in one eye.what next??

a-Digital ocular pressure

b-Tropical pilocarpine

c-Acetazolamide

3-Old lady with painless hematuria and right loin pain ..most appropriate next
step ??

A ct abdomen and pelvis

B cystoscopy

C urine culture

4- Melanoma skin care screening. 24 years old Computer technician come for
screening. What would be the risk for him?

A)recent diagnosis of melanoma in cousin

B) outdoor working in early twenties

C)history of sunburn in childhood

D) family history of basal cell carcinoma

5- female patient 60 years old ,menopause at 45 years, came for clinck for
assesemnt of osteoporosis, her mother has history of hip fracture .what is the
most risk factor for her ?
A. EARLY MENUPAUSE AT AT AGE 35YEARS

B. SMOKING HISTORY BEFORE AGE 25 YEARS

C. WAS ETHELTES AT AGE 18-20 YEARS AGE

D. FAMILY HISTORY OF OSTEOPOROSIS

6- 4 YEAR old ,IRRITABLE, LETHERGIC, NOT FEEDING WELL,COUGH, FEVER


38,DYSPNOEC ,RR 40-60 ,SPO2 88, ON CHEST REDUCED AIR ENTERY TO LEFT SIDE
OF BASE, TRACHEA SHIFTED SLIGHTLY TO RIGHT SIDE.WHAT IS THE INITIAL
MANAGEMENT?

A. oral steroid

B. blood culture

C. oral amoxyclave

D. ivpenzylepenicllin

e. iv fluoxacillin

7- female had a normal delivary of a normal weight baby , there was perianal tear
that was small and didn't needed suturing , after the delivary she noticed that her
locia didn't flow well in the following days , and at Day 4 she presented with
heavy bright red bleed with fever of 37.5 what is the reason for fever

A)endometritis

B)cervical tear

C)infection with perianal tear


D)DVT

8- parents brought 5 years old son with spinning around with no loss of
consciousness and lasting for 1-2minutes every month for last 3 months and
subside spontaneously. No h/o headache or vomiting. Ear examination normal
.What will you do ?

A.EEG and CT scan head

B.Audiology and ENT referral

C.Reassure due to BPPV that it will resovle later.

9- student is brought to you for assessment as his school performance is


deteriorating. noprob for the last 2 yrs in school although he was just an average
student. parents claim he doesnt listen and has some problem in written chores,
what will u do first?

A-ophthalmology assessment

B-assess dynamics

C-auditory assessment

d-family problems

10- man with faintness for twice a week, lethargic. On exam on right side diastolic
murmur, on left systolic murmur.Whats the cause?

Mixed vulvular heart disease

A-Asd
B-Vsd

C-viral cardiomyopathy

D-pulmonary hypertension

11- A man with peptic ulcer who is taking triple therapy (PPI, amoxicillin,
metronidazole) for 6 weeks but urease breath test was positive for H.pylori

what is the reason?

a. resistance to metronidazole

b. resistance to amoxicillin

c.unreliable urea breath test

12- Releation between navie and melanoma??

A- COHORT
B-CASE CONTROL
C-CROSS SECTION
E-PREVELANCE

13- 3 days ago pt drove his brother from airport to home, yesterday his brother
had rash and was diagnosed with measles , what is your management

A) reassure he is immune

B) Do MMR Serology

C) give him MMR vaccine now


D)GIVE him human immunoglobulin

14- patient will travel to endemic area of typhoid,, what is ur advice??

A- boilling water

B -avoid vegtables and fruits

C- wash hands

15- Obstructive sleep apnea,, obese with waist n neck circumstances 110 .asking
for long term treatment?

A.wt loss

B.metformin

C.Cpap

D.surgery

16- Patient with hypothyroidism . Ischemic heart disease and hypertension on


levothyroxine 150 mcg daily she developed signs of hyperthyroidism and labs
confirmed it so they decreased her dose to 100 mcg today , when should you
follow up TSH level ?

a- twice daily

b- after 1 day

c- after 7 days

d- after 2 weeks

e- after 4 weeks

17- patient with headness and palpitation and gives 5 ecg and wants the right
answer from them

18- schizo p.t on antipscychtc have weight gain, most is the next investigation?

a. TSH

b. FBS

c. Lipid profile

19- man with nasal blockage especially at night & can't sleep well.he has
periorbitaldarkness.allergy test (+).what is ur initial step??

a-Normal saline wash

b-Oralcetrizine at night

c-Oral antihistamine in the morning

d-Oralsteriod at night

e-Nasalephedrine spray at night

20- Woman with pap smear showing LSIL.Previous pap smear 3 yrs back was
negative.

A. Repeat pap smear in 6 months

B. Repeat pap smear in 12 months

C. Colposcopy

21- History of sigmoid colectomy after volvulus presenting again with signs of
obstruction, xraygiven.next appropriate managment ?

A. Colectomy

B. Oil enema
C. Barium enema

22-picture of parotid tumor old man ask about dx same as in hand book

23- pctue of femorla hernia ask about dx

24-

Picture of cleft palate baby born for 41 years old mother using carbamazepine
and alcoholic and ask what is the cause in this case

a. Genetic

b. Alcohol

c. Carbamazepine

d. Maternal age

25- male attack of angina and hypertensive, 170/? , s.cholesterol 5.5, most risk
factor for IHD?
A-Angina

B-BP

C-Cholesterol

26- A boy came to you with history of ear discharge 2 weeks ago, his tympanic
membrane  looks  dull  and  retracted  and  there’s  yellow  discharge  from  his  ear.  
What’s  the  nest  step  in  management?

a- Amoxicillin

b- Ear toilet
c-amoxyclave

27- young man who is presented with infertility. He had mumps when he was 6
years old. He was on methotrexate before some months or years and he is taking
sulfasalazine now his semen analysis showed motility of 5% , sperm count 1
million /ml and abnormal sperms 90%

what is the cause of his infertility

a. methotrexate

b. sulfasalazine

c. mumps

28- Cervical spine injury bp 80/55, HR 50,what is the NEXT most appropriate
step??

a-Atropine
b-IVcolloids

29- Most teratogenic in last trimester?

a. anticonvulsant

b. antipsycotic

30- aboriginal man came to dm medication ..obese..alcohol takes 14 sd at


weekend..advise

A-regular bmi checking

B-wastcirSud<102

C-reduction. Of fat in diet will reduce wt

D-environmental & social issue

31- Pregnant female with genital herpes how to differentiate 1ry From recurrent
herpes simplex:

A- HSV Igm now

B-HSV specific serology now

C- PCR

32- old man has headache and is accusing neighbors as they use insecticide
excessively. He mentioned he had frequent trouble with them as they are very
noisy.he changed his living place two times before as he was unlucky with his
neighbors

Thinking that they hate him and want to harm him .other persecution thinking
was there.

a delusion

b.depression

c. schizophrenia

d. Paranoid personality disorder

33- patIENT with renal failure got utitx

A-prednisolone

B-Nitrofurantion

C-Trimethop/sulpha

D-amoxicillin

34- pregnant lady(G1)come to u with family h/o of mother with DVT at 50age, her
sister has h/o abortion.during antenatal period what inx will be most benefit for
her current pregnancy??

A-Thrombophilia screen

B-Antiphospholipid Ab

C-Coagulation profile
35- Builder with TIA, carotid duplex done normal, ASA given and discharged what
is your advice?

a- Add anticoagulant??

b- No drive for 6 months

c- Not return work except after his doctor

d- Avoid heavy work

36- Baby in first day jaundice, ask about the cause ?

A. Haemolytic Anemia

B. Biliary Atresia

C. Breast Milk Jaundice

37- pregnant at 37 wks..sprained ankle badly..u applied crepe bandage and and
advised to keep foot elevated..what is the best advice u can give regarding pain
management?..

a. avoid codein

b. avoid all pain medicines unless it is intolerable

c.Avoid NSAIDS

d. use topical medicine


That what I remember now, plz pray for me and thanks to all of you
25th June 2016, (Dr.Asish)

i had a few recalls,mostly new stem but not so difficult ,among them two were very big stem with lab
value,four  were  picture,I  had  much  from  psychiatry,here  some  of  them……….not  much  recall  so  
remembering  is  little  difficult……

1)a 25 yrs old male had an accident with multiple ribs fractures ,difficulty in breathing,with diminished
breath sound in rt side (not mentioned whether there is pain or not),bp 90/60 mmhg,pulse 120 ,after
initiating iv fluid and resuscitation ,what will be mx???
a)watet seal drainage
b)usg
c)neddlethoracocentesis
d)oxygen

2)a 80 yrs old man taking anti parkinsonian drugs ,had a surgery 30 yrsback,complains nausea
,vomiting,and abdominal distension, (picture was twisting) not so much clear supporting SBO and
LBO,dx??
a)pseudoobstruction (I choose it)
b)SBO
c)LBO

3)picture of CRAO ,(absolutely clear),Rx


a)Iv acetazolamide
b)oral acetazolamide
c)pilocarpine
d)atropine
e)oxygen

4)(pelvic picture given),10 yrs old boy ,limping,esr 10mm in 1sthour,usg 6 mm joint fluid, (no URTI
history)
a)irritabe hip
b)parthesdz
c)juvenile osteoarthritis
d)septic hip

5)( a clear chest xray given)history of firm smooth swelling (complain of this) in the posterior triangle of
the neck,nopain,nofever,what  is  the  next??  Very  weired……….
a)excision
b)ct chest
c)xray (of which site not mentioned)
othrs r not relevent

6)a 28 yrs old female complaining of tiredness present after delivery of 3rd child having previous 2 alive
child,with a very long lab value but out of them only low Hb and Low MCV,and rest long lab value are
quite  normal……….
a)Iron deficiency anaemia
b)thalassaemia
c)pernicious anaemia

7)a  middle  aged  agitated  people  brought  by  police  by  trolley  in  ER  ,pt  saying  “I  know  you  taken  me  in  
prison,I  know  u  will  kill  me”,(no  history  mentioned  he  is  alcoholic/binge  drinker  already),what  is  the  
present state of him?
a)delirium
b)delusional disorder
c)dementia
D)drug abuse

8)27 yrscontinuely ecstasy taker reuesting to take sartaline,what would be that effect?
a)synergistic effect
b)drug interaction
c)potntiate the effect
d)enhance the effect

9)a 8 year old boy present with persistant (++) proteinuria,nohaematuria,noHTN,what will be the next
investigation?(no drug histry like prednisolone taking histry is mentioned)
a)renal biopsy
b)C3 C4 level estimation
C)12 hrs urinary protein estimation
d)urine examination

10)an ECG ,cleat st elevation on lead 2,3,aVL,present in 2 hrs, what will be done now?
a)angioplasty
b)aspirin
c)clopidogrel
d)dipyrimadolan

11)an ECG of Complete HB,what will you do do now??


a)temporary pacemaker
B)permanent pacemaker
c)cease digoxin

12)an ECG given (wide QRS complex ,regular,tachycardia,no p wave,)history of HTN of 40 yrs ,what is the
Dx?? (no atrial flutter is given in option)
a)ventricular ectopics
b)atrial fibrillation
c)SVT

13)2 Ct scan of head given (in a single slide),clear Ct


a)cerebral tumer (I choose it,seems to Hand book question)
b)cerebral haemmorage
c)cerebral infarction
d)cerebral abscess

14)picture of anus showing little something prolapsed with blood stained ,no anal tug for sure,dx asked
?
a)prolapsed internal haemorrhoid
b)anal fissure
c)anal abscess

15)18 yrs old painful reddish scrotum (pic given),what is the probable cause?
a)clamydial
b)gonorrhoea
c)staphylococcal
d)streptococcus

16)a child with clear ingunal hernia picture

17)a 36 yrs old male alcoholic ,complining pain ,wt loss 3 kg,given a very very long lab value ,among
them HB low,LFTabnormal,bilirubinhigh,ALP was high,what will be the investigation ??
a)ct abdomen
B)xray abdomen
c)USG

18)a scenario of 29 yrs old female there was a choledocolithiasis ,what will be the next Mx ??
a)ERCP
b)MRCP
c)CT
D)USG

19)there was a another very very long lab interpretation given,among them PO2 is 45%,PCO2
65%,hyperinflated lung,dysnoeic,labouredbrething,and rest of the lab value is normal to me, ( in this
case Ca level was also normal)??
a)bronchoscopy with alveolar lavage
b)ct chest
c)CXR

20)4 days after cholecystectomy operation male pt present with dysponea,with no cough,nofever,hat
will be next?
a)chest physiotherapy
b)CXR
d)oxygen
D)salbutamol inhalation

21)25 yrs old lady something drug taking now recently switched to lithium and resperidon,develops acne
and wtgained,what is the effect of this conditions??
a)combined effect of both drug
b)Lithium
c)resperidon

22)picture of Toxic megacolon

23)12 months old boy presents with his mother with the complaining of general check up,which one of
the following indicates serious developmental delay?
a)don’t  able  to understand NO
b)don’t  able  to  walk  unsupported
c)don’t  able  to  put  shoes  wire
d)don’t  able  to  draw  a  human  face

24)19 years university student ,complaining by his mother that, now he is sleep mostly in day time
rather than night,playing much more computer game,his academic status are good,lack of social
behave,what is the drug treatment?
a)mirtazapine
B)risperidone
c)some others very weired drug I cant remember them

25)one child 6 yrs ,lack of soial interaction ,prefer to play with her doll in the same manner ,any
alternation ,she crys, (clearly said that her language development are normal),what is the diagnosis??
a)autism
b)ADHD
c)Asperger

26)woman brought her child for vaccination ,does not look into doctors eye,what would be best advice?
a)tell her to bring family member in next visit
b)decline to visit
c)call for assistance

27)16 yrs old girl comes with complaining of recent start of sexual activities and seeking contraception
advice ,what would be best advice beside contraception?
a)chlamydia PCR
b)gonorrhoea
c)syphilis
d)AIDS

28)16 yrs old girl comes and said that she started her sexual relationship with her partner 2 yrs back ,and
she told doctor for chlamydia testing,and doctor advicedher,a few days later her mom ring you and tells
doctor that her daughter told her to follow up the test result,what will you tell that lady?
a)decline to talk with that lady
b)tell that you cant share any information of test result rather than her daughter
c)tells comes with her daughter in next visit

29)a senior told a new doctor to perform (something invasive surgical procedure),and the new doctor
didn’t  saw  that  procedure  before  and  don’t  knw  about  that  procedure,what  to  do  now??
a)tell him first one should be done for him first
b)tell the doctor that you cant do that  as  he  don’t  know  the  procedure
c)decline the procedure

30)26 years lady at term presents with the complaining with the pain,cervix 2cm,head 3 cm above the
ischial spine ,fetal heart rate 140/min, about 5 hrs later what is the indication of C/S?
a)cx 3cm ,head still 2 cm above spine
b)Cx 9 cm ,head still 2 cm above the spine
c)remains same after 5 hrs
d)cx 9 cm ,head stil 2 cm above spine,membrane ruptured

31)nurse in a nursing home got needle prick during removal of DJ stent from renal system of known case
of HIV pt,what should be done now??
a)give antiretroviral now
b)give antiretroviral and Immunoglobulin
c)perform IgM and IgG

32)21 yrs medical student got needle prick got needle injury during some procedure (cant remember) of
a known HcV positive patient,what should be done now?
a)do immunological test in 3,6,12 months
b)do antibody test right now
c)perform IgM and IgG
d)give Immunoglobulin

33)30 yrs old present with melaena,to find out the cause there is endoscopy done but nothing is
conclusive,what should be done now??
a)colonoscopy
b)sigmoidoscopy
c)capsule endoscopy (pillcam)

34)have  got  two  like  this  question  t  answer  “capsule  endoscopy”

35)patient having problem with BP 165/90 mmhg, (+++)proteinuria,


(++)haematuria,ankleoedema,abdominalswelling,what is the dx?
a)NS
b)interstitial nephritis
c)CCF
d)cirrhosis of liver
e)others are irrelevant
exactly these are options were there

36)a sudanese7 months baby has got (first attack may be) seizure (<1 min),and everything are normal in
physical examination,what to be done now?
a)to estimate vitA,vit D (I choose it one)
b)EEG
c)CT head
d)FBC

37)  a  traveller  from  sudan  (may  be,  people  don’t  miss  a  single  question  related  Sudan,dot  forget  AMC  
loves Sudan very much) with a very very long lab interpretation,with RUQ pain ,jaundice,norash,return
after 5 days,all were normal except LFT,among them bilirubin is high,but GGT is in normal,so what is the
Dx?
a)Hepatitis C infestion
b)dengue
c)Malaria
d)others are irrevelent

38)a 8 years old boy got acute attack of asthma admitted in hospital,after giving a 6 puff of inhaled
salbutamol,he got little improvement,but he cant speak a full sentence,what to do now?
a)repeat this dose in 20 minutes
b)give fluticasone
c)give SCG
no options for repeat 12 puff more

39)65 years old man present with 38.7 degree temperature,cough ,dyspnoeic ( given a clear chest xray
of middle lobe pneumonia),what to give now?
a)Amoxicillin+clavulunate
b)ceftriaxone
c)flucloxacillin
d)paracetamol

40)A man taking sertraline and selegeine,after few days develop


dyasthesis,sweating,hyperreflexia,diarrhoea,etc (serotonine syndrome),what to do now?
a)stop selegeline
b)stop sertraline
c)stop both sertraline and seegeline

41)a 44 yrs old female has done her mammography that is normal,and also done her usg showing 1.5 cm
hypoechoic shadow in her left outer quardrant of left breast,what to do now next?
a)core biopsy
b)FNAC
c)excision biopsy
others cant remember

42)there was a female 34 yrs having amenorrhoea since 2 yrs ,long lab scenario but everything was
normal,FSH,LH,prolacine  ,all  are  normal,what  is  the  Dx??……………………………………………………..
a)hypothalamic dysfuction
b)pituitary tumer
c)adrenal tumer

43)others I cant remember right now,but wish to upload soon more.


FEW WORDS:
*one thing want to add that is ,there is no worry about time management at all,there is enough time to
finish your exam .

*read ECG very well (specially VT,VF,AF,MI,PE,BBB,LVH,HB,SVT,)


*read anthology picture A-Z
*Read Hand book well
*previous 3 months recall very well with their topics ,plus addition 3 months recall as well
*read Kaplan for ethics very well ( case related only)
*time  in  exam  is  retrograde  (100>99>98……)
*bring passport +one any ID card having your signature

And    after  all  keep  belief  in  your  “God”,    keep me  in  your  prayer  that’s  enough,only  by  this  we  can  go  a  
long way in our respective way.Good luck.

Most of my questions were recalls, some of them controversial ones.


The exam is extremely tiring so word of advice, get plenty of rest and yes, get yourself a
tranquillizer if need be for day before exam because my night before the exam made me
respect that relaxation therapy senario to the next level!!!! I barely got any sleep.

Study advice. Do recalls and recall topics, read JM for diagnosis but for treatments please
keep checking websites like emedicine, uptodate, mercks manual, and Australian
therapeutic guidelines.
Wishing all of you the very best. Please keep me in your prayers.

1.what to check in a patient on clozapine who develops palpitations etc


troponin
echo
holterecg

2. A young man brought to you.he is distressed,teary and angry on a someone who has
killed his wife by passing a vehicle over her when she was crossing road 2 weeks ago.he says
he can see his wife or listen her voice,also he says his wife is not really dead.
1.temazepam
2.olanzapine

3.Child with URTI and after two days presents with BP 140/90, edematous face and bilateral
ankle adema, urinalysis show rbc++,protein++, asotiter raised, what to give him:
a.Iv hydralazine
b.furosmide+iv fluid
c.steroid

4.old man with increasing constipation since one month , now comes with abdominal pain n
distension for 24hours. History of appendectomy 20years ago n now he has reducible
inguinal hernia.Dx?
1. ca sigmoid colon
2, adhesions
3. sigmoid volvulus
4 strangulated hernia

5.middle aged lady with uterine prolapsed..weakness in which of the following structure is
responsible for this?
a. uterosacral ligament
b. transverse cervical ligament
c. pubocervical ligament

6.An immigrant from Afghanistan complaining of fever and prolonged productive cough. He
has no history of vaccination against TB. His mantoux test is positive with induration of 12
mm. his sputum showed acid fast bacilli. Next step:
A- Isolate in negative pressure room
B- Do x ray
C- Do sputum culture
D- Do sputum PCR7

7. 6 years old boy presented to the ER due to asthma attack. He had 6 puffs of salbutamol
and he improved but still has wheezing and speaks in words
a. add salmeterol
b. more 12 puffs of salbutamol
c. ipratropium nebulization
d. oral prednisolone
e. hydrocortisone iv
8. A truck driver was fired from his job because of always being late. He makes sure that he
wash all the keys one by one so that nothing bad will happen on the road.
What does his behavior denote?
A. Obsessive
B. Compulsive rituals
C. mannerisms

9. 40 yr female had tonsilitis, took amoxicillin, after 3 weeks she devloped purpuric rash, not
blanching over her ankles. the joints are not painful
A. Hypersensitivity vasculitis
B. HSP
C. I.Mononucleosis

10. colle's fracture xray with scenario of a lady who falls on her outstretched arm and you do
a below elbow plaster. when to follow up
2 weeks
4 weeks
6 weeks

11. scenario of a patient on levothyroxine 75 mg who had some surgery done. his labs
revealed Tsh low and t4 raised. what to prescribe to him on discharge

25 mg levothyroxine
100 mg
150 mg

12. scenario of a lady with splenomegaly and rash on the legs thrombocytopenia, borderline
anemia and leucocytes decreased also
hb 11 wbc 4000 plat 50000
complaining of fatigue past few months
diagnosis
sle
itp
hsp
aplasticanemia
AML

13. Pt with candida (all the features of soreness brick red vagina white discharge) treated
with clotrimazole ,but recurrent several times , she always responded to topical clotrimazole.
What will be the most important step in her management?
-vaginal swab
-fluconazole
-glucose tolerance test
-hiv antibody test
No option for culture

14. Ctg showed reduced baseline variability and variable deceleration (ctg not given) the
mother had 4 cm cervical dilation within 14 hours , pethidine was given to her 3 times I
guess the fetal head is in the left occiput posterior. What is the cause of ctg abnormality
prolonged labour
elevated resting uterine pressure
increased pressures due to incoordinate uterine action

15. a 45 year old woman, a case of 2yr infertility , her children are 14, 15 and 20 years old,
h/o endometriosis in uterine lig , previous h/o pelvic operation, mid cycle sex. Which of the
following is the cause of her infertility , partner never fathered a child.

increased maternal age


sperm abnormality
ovulation problems
endometriosis
tubal pathology

this question I got thrice with a bit of change in stem


in one no partner history was mentioned

16. in the other there was mention of dyspareunia and it was a young couple with all normal
investigations and one sided tubal block  endometriosis

17. 6 year old girl with urti and + rbc in urine , all other investigations normal but 1 week
later the urine exam was same
Post strep
IgA nephropathy
Transient hematuria

18. XRay of cecal volvulus, in a patient with a history of sigmoid colectomy, after initial
management with iv fluids, what is the most appropriate initial step
Low pressure NG decompression
Hemicolectomy
Colonoscopy

19. 2 year old child, had episodes of vomiting associated with abdominal pain, mother also
noticed a mass in Rthypochondrium
Diagnosis asked
Wilm’s tumour
Neuroblastoma
Pelviureteric junction obstruction

20. Old patient taking many painkillers for cancer


He is unresponsive and received naloxone and high flow oxygen after which he opened his
eyes and then he became drowsy again.
ABG Po2 120, PCo2 55, Hco3 26
What to do.
Decrease oxygen
Give naloxone
Intubation
Give morphine

21. 1 CT of glioblastoma with r/l disorientation, apraxia, dyscalculia, dysphasia in a 60 year


old man since 3 months

22. The same scenario without a CT given asking most likely cause

23. XRay of ankylosing spondylitis, asking what will be the best initial management

No nsaids in options
Physiotherapy and hydrotherapy
Methotrexate
Infliximab

24. Xray showing bilateral hilar lymph nodes, asking most appropriate initial investigation
Ca levels
Urine microbiology
Ct abdomen

25. A boy ate a pizza before few days when he developed diarrhea and now presented with
subconjunctival hemorrhage and epistaxis and rash
what is the test that will lead you to diagnosis
a. blood culture
b. stool culture

26. a man who has cough since 6 months and a rash on his legs, he has loin pain and
hematuria, what is the most appropriate investigation to be done

Skin biopsy
Ct abdomen

27. picture of molluscum


No exclusion

27. picture of herpes


Spot diagnosis

28. ecg of 1st degree heart block in a man who lost consciousness but is now stable his heart
rate is 35 bpm, asking whats the most important thing to do
Isoprenaline
Atropine
Digoxin
Permanent pacemaker

29.progressive swelling of eyelids with serous discharge and pain in one eye, temp 37.8 hay
fever hx

oral ceftriaxone
ivflucloxacillin
prednisolone
topical chloramphenicol

30. Another growth curve for 18 bmi boy with weight more than 98% percentile for his age
Normal
Obese
Overweight

31. 47 yrs female came concerned with weight loss her bmi is 29 what else you have to do
for her
Bp
Waist circumferance
Tsh
32. Patient on venlafaxine before increasing the dose what will you check
Lfts
Bp
Tsh

33. Man came with 3 cm painful tendermass over inguinal ligament , appeared after he
returned from a hiking trip

inguinal hernia
femoral hernia
Iymph node

34.An old lady lives by herself mmse 28 had fracture of femur, Family wants everything to be
done she also has copd and had to be admitted in the icu many times . This is her 6th time
.Doesnt want further treatment . Lady is competent to make her decisions
A . Psychoneurologicalassessemnet
B. Arrange meeting with the family
C . Inform the hospital ethics group

35. Farmer wid hemicolectomy due to colonic ca, liver usg given which were clearly mets
Triphasicct
Hydatid serology

36. Woman on trifluphenazine well controlled, left drug when she started developing
tremors one yr back
Now thinks has intestinal worms over her whole body, initial treatment asked
Haloperidol
Trifluphenazine
Quietapine
Depot risperidone consta
Clozapine

37. Accident many ribs fractured patient is severely dyspneic severely cyanosedand in a lot
of pain. What will be your best management at the accident site
Cover the wound
Needle thoracostomy
Morphine
38. CT chest of rt sided haemothorax after penetrating trauma, no tracheal deviation
maintaining spo2 96% how will you manage this patient?
Needle thoracostomy
Morphine
Underwater seal drainage

39. Picture of Pilonidal sinus .. Recurrent


Fistulectomy
Marsupialisation
Antibiotics
Radical excision

40. 55 year old man with hematuria,mass on the lower pole of kidney 3 cm on CT
left nephrectomy
Biopsy
Surveillance

41. a known case of polycystic kidney disease with lower urinary tract obstruction, what
investigation will you do to evaluate
CT abdomen
US kub
Retrograde pyelogram
Cystoscopy

42.A man with peptic ulcer who is taking triple therapy. PPI,Amoxicillin, metronidazole for 6
weeks but urease breath test was positive for H.pylori what is the reason ?
A) resistance to metronidazole
B) resistance to amoxicillin
C) unreliable urea breath test
D) restested too early

43. Case of stone 9 mm at ureteric orifice in bladder, prepared for surgery, which
investigation must be done before going to surgical table
a. abdominal xray
b. u/s
c.CT

44.Pt diagnosed with AIDS 6 months before, on indinavir developed severe loin pain with
hematuria..his urine showed +++RBC asking for investigations
triple phase ct scan
non contrast ct scan
abdominal x ray
Us

42. 9years old child, his BMI is 20 came with a sore throat. All other family members are
overweight. What will you tell his parents?
A- He will grow and become normal
B- He should reduce hours of watching TV
CHe should start on weight reduction regimen
D- replace sodas with fruit juices
E increase salt intake

43. a scenario of a woman who has repeated hallucinations when she tries to sleep, she also
had few episodes of amnesia in the past few months followed by hallucinations, what is the
cause
Temporal lobe epilepsy
Meningitis
Migraine
Cannabis intoxication

44.Pregnant female with genital herpes how to differentiate primary from recurrent herpes
simplex:
HSV Igm now
HSV type specific serology now
PCR
Check antenatal bloods for serology
Culture

45. 50 year old man who is working in real estate. He used to dress well but recently he
looks in bad shape, shouted at customers and got fired from his job. His wife brought him,
how will you evaluate
A. verbal fluency test
B. minus 7 test
Others irrelevant

46.patient with renal failure got utitx


Gentamicin
Nitrofurantoin
Trimethop/sulpha
Amoxicillin
46.An old age pt brought to you with complaint of weakness of lower half of face and right
upper limb,she has flaccid paralysis for 1 hour.she is on aspirin for some cardiac issue. CT
scan is normal. What is most appropriate next step?
a.aspirin and dipyridamole
b.rtpa

47. dementia patient with hypertension and IHD, his bp is 130/90 pulse regular 80
What will you prescribe to him

Rivastigmine
Memantine
Donepezil

48. Taxi driver who had symptoms of right sided weakness for 10 mins, resolved, CT normal,
carotid duplex also normal, he is insisting on discharge to work as a driver again .
A. Discharge but sign consent
B. Refer to neurologist to decide about driving clearance
C. Tell him you can drive in 6 month
D. We need to assess driving after more imaging

49. COPD with severe dyspnea, tachypnea , on 8L O2 , spo2 86%, bilateral good air entry,
whats the most imp Inv to be done
spirometry
ABG
Cxr
Ct

50. an old woman admitted for orthopaedic surgery due to fall, she was alright but the night
before surgery she got agitated and started blaming the staff for sexually abusing her,
which of the following do we expect to find on examination
Fluctuating levels of consciousness
Paranoid delusions

51.A 26 year old woman who is not sexually active since 2 years came for her regular pap
test, question asking why to screen for chlamydia when you want to do opportunistic
screening

Chlamydia causes infertility


Chlamydia is asymptomatic
52. pregnant lady came for antenatal checkup, told her husband abused herinpast month
when he was intoxicated with alcohol and makes sure not to leave any marks what is your
next action:
urgent admission
urge her to call police
reassure
make a safety escape plan

53.16 year old female, separated from her boyfriend since 6 weeks, now lost weight. lazy at
work, has insomnia, anxiety attacks &is stressed
Fluoxetine
Female adolescence group
Start CBT
refer to psychiatrist

54. Patient is an outdoor worker, came with low BP (90/60) lab: High platelets, others
normal, dx:
essential thrombocytosis
post splenectomy
myelofibrosis
CML

55. A patient has nausea vomiting, epigastric pain or tenderness (with no signs of
peritonitis). From the history, you knew that the patient had gastric ligation surgery before,
what is you initial management?

a. barium swallow

b. urgent surgery

c. CT scan

d.reduce fluid from band

56. years old female with body mass index 23, complaining of tiredness and laziness at work.
She had menstruation at age of 12 and menses are regular. She has dark pigmentation on
back of her neck and axilla and abdominal striae. Her grandmother has type 2 DM.
diagnosis?

A- Cushing disease

B- Addisson disease
C- Hypothyroidism

D- PCO

E- Metabolic syndrome

57.Pregnant lady at 39 weeks of gestation presents with bleeding of 200ml, now stopped ,
clinical examination reveals 3 cm dilation and visible membrane, rest is normal vitally, whats
next:

admission and observe

AROM

C-section

58. pregnant at 37 weeks sprained her ankle badly, you applied crepe bandage and advised
her to keep foot elevated. what is the best advice you can give regarding pain
management?

a. avoid codeine

b. avoid all pain medicines unless it is intolerable

c.Avoid NSAIDS

d. use topical analgesics

59.Old female with breast cancer her son doesn't want her to know as she will get
depressed

A Insist to meet the patient alone

B Tell her son to bring all family for a meeting except her

C Give an appointment for the son to discuss this issue

D encourage the patient to bring a family member in her next appointment


60.scenario of secondary amenorrhea all her investigations was normal (FSH AND LH are
within normal range and no increase in LH/FSH ratio ).. her weight is 45kg.…

Us shows normal ovaries with 3- 4 cysts.Whats the most likely cause

-‐PCOS

-‐POF

-‐idiopathic hypothalamic dysfunction

61. aboriginal female who doesn’t have access to medical facilities with 3 month dyspnea
brought by her friend, on examination b/l basal crepitation and a diastolic murmur, not a
smoker but a binge drinker, next step?

A. ECG

B. Echo

C. alcohol counsel about intake

D. establish an ongoing therapeutic relationship with her

62. 37yr old woman with secondary amenorrhea normal BMI ,FHS- high OESTRADIOL low
prolactin normal .USG-3 to 4 cyst in ovary.Doesn’rwant to conceive and is not sexually
active, What is the best treatment

A)POP

B)Menopause hormone therapy

C)OCP

D)Metformin

63. A young woman with persistent pain on the lower right side of the face with redness
asking about the most probable diagnosis

a.varicella

b. trigeminal neuralgia

C herpes simplex
64. 6 weeks old baby well and thriving, 3 days ago he has developed increasing vomiting
but he is well hydrated, no increase in weight since 1 week, all examinations are normal

A-GERD

B-PS

C-uti

65. Pic of an old female in nursing home with NG tubes and sick looking and has a left sided
parotid swelling and redness asking about cause

a. Parotid gland tumor

b. Parotid stone

c. Parotid infection

d. Poor oral hygiene

66.old man feels pain when he walks only two blocks and stops to rest after that. He has a
recent history of left buttock and back of thigh pain, but he has three month history of
ischaemia in left leg and foot. A bruit is present over femoral artery and his limb is
pulseless.Next appropriate step?

A- Thrombolytic

B- Angioplasty

C- Bypass surgery

D- Walking program

E- Nefidepine

67. picture of pyogenic granuloma after getting finger trapped in doorway

A excision with 1 cm safety margin

B diathermy

C cryotherapy
D local excision

E incisional biopsy

68. 3 year old child came with complain of cough and mother had hay fever and father is a
smoker, what drug is used for the prevention of asthma

a.SCG

b.Fluticasone

c.budesonide

d.salbutamol

69. a 2 year old child presented with stridor, harsh cough, temp 38 degrees and intercostal
recessions, he is alert and active, how will you manage him

Oral prednisolone

Nebulized adrenaline

Iv steroid

Nebulized salbutamol

70. Mass in anterior leg, firm and regular, attached to gastrocnemius muscle, most
appropriate inv?

A. MRI

B. U/S

C. CT

D. Biopsy

71. a25 yr old female complained of Raynaud's phenomenon in cold which started after
pneumonia 2 years ago,ana 1/60, ENA neg. cause
A-primary Raynaud's
C-sle
D-limited sclerosis

72. 3 yr child got pertussis, family got ab prophylaxis but not his 5 month old sister who has
completed her vaccination
a. advise mmr vaccine at 12 months
b. advise dtpa boosters for parents & grandparents
c. give dtpa vaccine to family adults who are of unknown immunization status
d. avoid child care till age of 6years

73. mother comes with her 2 month old child for immunization, the mother is living now
with her parents, the mother has egg allergy. what is your advice for the family
a. advise mmr vaccine at 12 months
b. advise dtpa boosters for parents & grandparents
c. treat with ciprofloxacin if develop symptoms

No option of unknown immunization dtpa

74. 60 year old man with iron deficiency anemia, he complains of bleed on toilet paper and
mass on straining, on DRE everything is normal, what is the next appropriate investigation

FOBT

Colonoscopy

75. lady feeling faint twice a week, lethargic. On exam on the right side there is a diastolic
murmur in the 2nd intercostal space, on the left side a systolic murmur at apex. Whats the
cause?
Mixed valvular heart disease
Asd
Vsd
pulmonary hypertension
pda
76.during a family gathering your 17 yr old nephew tells you that he is very anxious about
his upcoming exam next week and asked you to prescribe him temazepam. what will you
do?
a-advise relaxation and exercise
b-tell him to go and consult a doctor properly

c.prescribe melatonin

In the name ofAllah

1. 2 scenario of 48 year old lady who afraid from breast


cancer what to do?
2. 6 hour neonatal jaundice?
3. 3 patient on 6 puffs salbutamol, not improving?
4. Patient on carbamazepine, we want to add
lamotrigine and increased it when we can stop?
5. 21 year old man afraid from multiple things occurred
to his father such as dm and htn and ischemic heart
disease, what screen to do?
6. 89 year old woman pap smear normal last one and
the current one, what next?
7. Old woman after purulent discharge and after her
menopause purulent green discharge for 8 days
sexually active?
8. 18 girls came to the hospital with abdominal pain
and her mom called you at night to know
what’swrong?
9. Prostatescreencancer, no signs or symptom of any
problem in his prostate?
10. Chest wound laceration 2 scenario, what to do
even he received blood in the ambulance and they
are stable?
11. Young woman her mom have breast cancer with
brca mutation what to discuss with her?
12. Employee complains always she want some
certificate and keep not going and she is on alcohol
.what is the reason?
13. A woman is depressed and have two child 8 and
10 what to do with here immediately?
14. Girl with multiple partner what to do when she
came to you, what advice?
15. Small bowel obstruction. What to do?
16. Large bowel obstruction what to do?
17. 3  electrocardiogram  I  don’t  know  any  of  them?
18. 2 Cases of renal failure low creatinine and high
urea and abnormal complete blood count, dyspnea
what medication to give?
19. Huge lump lacerated in the abdomen in type 2
diabetes after antibiotic and debridement what to
do?
20. Patient on prednisolone for 15 years and now
anemia, what to do?
21. Baby with rectal prolapse 2 year old age, what
questions you are to ask to help you in the reason?
22. Baby with history of eczema and hay fever,also
in his parents while they are young what medication
to give to improve his status?
23. A woman came to a hospital for angioplasty,
now agitated what risk is significant to here during
these days and she is old?
24. A patient schizophrenic there is possible harm
on his friend what to do?
25. Now in Australia what psychiatric illness is
increasing?
26. A patient old wondering in the street brought
two times to the emergency department?
27. A woman pregnant came from travel overseas
to an African city very ill symptoms what is the risk in
her baby?
28. A woman pregnant in her first trimester used a
lot of illicit drugs what the most teratogenic?
29. A patient old one 79 year old keep forgetting
occasions  he  attended.  What’s  going  with  him?
30. Asperger child on sertraline and now is agitated.
Why?
That’s  all  I  remember  right  now  please  let  anyone  
upload these questions again and discs everything in it
for  helping  each  other’s  and  I  will  upload  another  files  
if I remember any further questions , happy Ramadan
friends and keep us on your prayer . Hope you all going
to pass
1. Man 40 yr always thinking that police is after him history of many oad accidents after one of which he
was in hospital for 10 days (intensive care) and that was 6 years ago. what will help you in diagnosis?
A. Henrick Anxiety Test
B. Beck Depression Test
C. Wechsler Adult Intellegence scale
D. Post traumatic scale
E. Neurological assesement

2. 7 days old with jaundice started on fourth day of delivery with total bilirubin 240 & conjugated 120,
asking cause?
A. Hemolytic
B. Biliary atresia
C. Breast Milk Jaundice

3. History of sigmoid colectomy after volvulus presenting again with signs of obstruction, xraygiven(to
me it looks more like fecal impaction than cecalvolvulous) next appropriate managment ?
A. Colectomy
B. Oil enema
C. Barium enema

4. Pregnant Recieved 3 times pethidine with total of 300mg, cause of fetaldistress ?


A. Sedative drug
B. Inc. Intrauterine Resting Pressure --> I opted for this

5. 2 ECGS:
Q.Vent. Fib. clear, started cprwhatsnext ? -->Defebrillation
Q. ECG Not clear one with many ectopics, the old one with biventricular failure & many drugs asking
next ?
A. Stop all and reassess in 12 hours
B. Increase digoxin dose
C. Give furosemide

6. Pic of old lady with parotitis --> Poor oral hygeine

7. Pic of middle lobe pneumonia not very clear , it was mild by history, asking managment ?
A. Penicillin
B. Augmentin

8. Pic of Scaphoid # --> Review after 2 weeks

9. Cleft lip --> Genetic

10. Scenario of alcoholic cardiomyopathy asking most appropriate inv ? --> Echo transthoracic

11. Pt with palpable painful skin rash on the knee, with hemoptysis& very elevated serum levels of ACE
asking to reach Dx ?
A. Skin biopsy
B. ANA

12. Pt on spironolactone, enalapril, no dyspnea on exam. high jvp and lower limb edema and bilateral
basal crepitations& ejection fraction 40% what to add ?
A. Digoxin
B. Metoprolol
C. Warfarin

13. Pt with painful leg on 100 meterwalk, well perfused and warm with absent pedal pulse -->
Graduated walking preg

14. Pt with scenario of acute limb ischemia with no similar incidents before asking hx --> Iv Heparin

15. Pt with scenario of chronic limb ischemia with an acute attack on top, best Hx ?
A. Angioplasty
B. Bypass
C. Intensive Walking Program

16. Pt with chronic limb ischemia, what will be useful to be useful for to be offered for this pt beside
cessation of smoking ?
A. Angioplasty
B. Bypass

17. Driver with scenario of TIA with a similar incident last week, best regarding his return back to job ?
A. After more inv.
B. Discharge on aspirin
18. Clear CT pic of hemothorax with pain on respiration asking appropriate next ?
A. Intercostal Drain
B. Morphine
C. Needle Aspiration

19. Another one with flial chest and painful respiration due to MVA, asking initial at the site of accident -
-> Morphine

20. On the phone son telling you that family is against you telling his mom about her malignant disease
discovered ?
A. Insist to see mother alone
B. Tell him to bring his mother next time
C. See him alone
D. Hold a family meeting without mother

FELICIA HOR------ RECALLS – 24th June Singapore

female has dvt last week started on warfarin.


presents with perforated duodenal ulcer.
wat to do
-stop warfarin n give vit k
-proceed with surgery
-give ffp n do surgery
-give iv heparin n do surgery

woman after excision of a hyperpigmented area in skin with LA feels tingling in her mouth.
likely cause?
-lignocaine toxicity
-hypocalcemia
i din now so i put A

female with AF comes with left weakness, pic ct of ischemic stroke.


wats the investigation?
-chest xray
-carotid doppler
-ct brain
89 year old male with constipation for months. on examination, ab is soft.no mass. rectal
examination : empty rectum.
what is the appropriate management?
-lactulose
-fleet enema
-colonoscopy

36 weeks pregnant primi, labor progressed normally.cervix 4cm n effaced after 1 hour into labor.
membrane intact, uterine contraction was 5/15 mins. heart rate 140/min. fetal skull 1cm above ischial
spines.
4hour later same finding but fetal heart rate not heard. ctg not detected.
management?
-cesarean
-oxytoxin
-amniotomy

woman has a 1.5cm on right upper outer quadrant. usg shows hypoechoic.
next management
-fnac
-excisional biopsy
-ct scan

lady came with pain in right upper quadrant. stone in vesicoureter junction. usg shows mild
hydronephrosis in the right side.
management?
-cystoscopy n put stent
-eswl
-refer for open surgery
-urethroscopy with fragmentation or stone removal

pic of groin swelling. reddish skin.


no cough impulse non reducible.
askwats the next investigation
-usg
-fnac
-ct pelvis
parents concerned 2 years old child not toilet trained. able to walk at 12 months. everthing normal
but wont interact n cries when u try to examine.
whats d dx
-asperger
-autism
-normal

description of right homonymous hemianopia.


askingwhr is the lesion.
-right visual cortex
-left visual cortex
-right optic nerve
-lt optic tract
-optic tract

needle prick in a nurse.


seeology of pt given hcvabneg, hbeab positive, hiv positive, hbcab positive.
asking next step.
-immunoglobulin
-start lamivudine
-start zidovudine

mother came for followup of child. child normal however mother is silent and withdrawn.
wat to ask mother?
-mood
-financial strain
-maritial status

56 years male general checkup, is well no history of dm, waist circumferance 101, clinical exam
normal.
how often to screen him?

pic of a hyperpigmented leg given.


ask what is it u will ask in history?
-severe melacosis
-smoking 40 pack years
-history fracture lower tibia/ fibula
MASOOMA ZAIDI

My recalls 24 June Sydney !


#  Infections  
1) clear scenario of herpes simplex with ulcers on lips gums ..Dx asked
2) molluscum pic ... No exclusion
3) man from Ethiopia with hx of hCG vaccination now mtx 12 mm no s/s asked next ....Xray
4) pain in the face in maxillary division constant for 24 hours ... Varicella
#  cns  
Meningitis scenario .. Headache n malaise with mild fever from 3 weeks .. Ct normal ..Lp shows
mono nuclear cells .. Glue oar n protein normal ... Ans:Echo virus
Another scenario with same findings n RBCs ... Mx.. Start antiviral
#  Ecg  
1) complete heart block with heart rate of 36 .. Pt on digoxin enalpril n metoprolol ..Mx: pace maker
2) Ecg given of a pt with MI 3 years ago .. Coping well bp in control with a murmur on left sternal
border .. Patient is taking Ace n metoprolol .Ecg had a 1st degree block ... Options 1) cease
metoprolol .. 2) commence warfarin
#  CTG  
1) same old recall of meconium staining .. Rest all was normal ..
2) women was given pethidine and CTg showed reduced variability and decelerations .. Cause :
effect of analgesics
#  Psych  
3 to 4 clear scenarios to dx schizophrenia .. Depression
A man facing severe depression admired after coming suicide is now discharged .. He's not
interested in work .. Eat n sleep normal
A woman with severe depression 10 kg wt loss.. Early morning awakening not willing to take
treatment .. How will u tell her prognosis if she's denying treatment : no other suitable option other
than she's going to commit suicide
A man is being taken off the anti depressanttherapy : reduce the dose over 2 to 3 weeks
#  Trauma  / fractures
Colles # when to review: 2 weeks
Chest trauma with bilateral # ribs ..Hemothorax is there too .. Patient is vitally stable n maintaing
oxygen saturation .. Not breathing well because of pain .. Next step : morphine
Patient with knife in chest .Bp is fine . Oxygen given already ..Next : remove knife under GA
Patient with knife in chest .Haemodynamicallyunstable . After giving fluids and oxygen whtsnext
..Cross match
Old question of man after hip surgery : SC enoxaparin for 6 weeks
60 year old man developed acute onset knee joint swelling . Aspiration revealed no organisms . He
has history of CKD with very low GFR ... Prednisolone
#  surgery  
Clear pic of SBO ..Hx of appendectomy ..Adhesion obstruction
#  Gynae  
Risk factor for osteoporosis in a 60 year old : early menopause at 35
A young couple in 20's trying to conceive for 2 years . Semen analysis was normal .. Labs were
norma. On hysterosalpingogram only one tube was patent ..regular intercourse with mild pain and
exam Uterus is retroverted . Cause : tube. Defect 2) endometriosis
Another. Question with slight change of details
#  renal  
Patient on treatment for AIDS . Developed Hematuria and dysuria with flank pain : non contrast CT
70 year old with macroscopic Hematuria . Most app inv
:Cystoscopy
2 Clear scenarios of IgA nephripathy... Dx was asked
Child scenario after sore throat developed high bp and oliguria ...: IV hdralazine
One question of PE...... Women obese and smoker 14 weeks pregnant developed SOB after
international travel ...
Patient with low GFR which drug to stop : metformin
New question #
A man 25 years took some Drug for gastritis 2 days ago since then he's been experiencing upward
rolling of eyes he went to party took MDMA n had a fit. Now he's conscious and alright . Only
complains of that upward rolling .Cause : 2) Oculogyric crises
Guys this is what I can recall up till now . Will keep adding more.Remember in your prayers....

1.Old lady comes with swelling in front of tragus, pain present before meals. Inv ?

A. Intra oral x Ray

B. Usg

C. Ct

D. MRI

2.Same scenario n management asked


A: gentle massage on it

B. Warm compressions

C. Antibiotics

3.Old lady with swelling in front of tragus. Cause ?

A. Poor oral hygiene

B. Ca parotid

C. Duct stenosis

D. Parotid duct stone

4.A foreign worker does not speak English brought by his work boss by foreign body in his eye. His boss can speak
English. What to do?

a-Take consent from the patient to make his boss as translator

b-Arrange translator by phone

c-Call his girl and ask her to come and translate

d-Tell the patient to bring an accredited translator and come back later

5.Girl experiences anxiety when goes out in meetings with strange people . Has stress at work cuz of work load. Next
?

A. Propranolol before going to meetings

B. Relaxation therapy

C. Psychotherapy

6.Maleptnt likes to dress up in female clothes. He says even his family treats him like if he is a girl. Dx ?

A. Transvestism

B. Transsexualism

7.Middle age 47 or 48 yr non to have IHD, have RUQ pain, put stent from 2 months, on clopidogrel, pain resolved, but
after 9 week the pain comes back. on U/S stone was in neck of gallbladder, there was fever n chills n tenderness, no
jaundice ?

A. Transcutaenous cholecystectomy
B. Antibiotics

C. Stop Clopi n Make operation in 5 day

D. Give hip platlets& Do operation

E. Endoscopic cholecystectomy

8.A school teacher comes with difficulty to initiate sleep every night. He says whenever he goes to sleep he feels he
might have done something wrong so he starts recalling each n every moment of his day. And after he has recalled
his day completely m realised that he has not done any wrong, then only he gets happy n satisfied n sleep . It takes
almost 1 -2 hours each day. T/t asked.

A. Antidepressants

B. Antipsychotics

C. Ssri

D. Antidepressants+ something

E. Bzd

9.Wheneever man enters your room, he always salutes thrice. Dx?

A. Ocd

B. Schizophrenia

C. Major depression

10.39 weeks vid contractions every 3 mins , cervix posterior n 2cm dilated n head 3 cm above IS. After 4 hours cervix
mid position, fully effaced, 4 cm dilated, intact membranes, head 1 cm above IS. Dx ?

A. Spurious labour

B. Incordinate labour

C. Normal labour

D. Obstructed labour

11.Clear scenario of imperforate hymen


12.Female delivered a baby 3800gm at 37 weeks without any complications. Placenta out after 5 mins, syntocinon
given. After 30 mins of deliver, 1000ml of bleeding vidin10 mins. Dx ?

A. Birth canal lacerations

B. Atony

13.55 yr female with breast lump in outer quad. Mammodone , found nothing. Usg done, found hypoechoeic mass.
What next ?

A. Core biopsy

B. Reassure

C. Excision biopsy

14.Male with fever , headache, papilloedema weakness on right side of body , on examination muscle strength
diminished on right side . Dx ?

A. Cerebral abscess

B. Left side infarct

Dun remember other options..

15.Which of the following has worse prognosis?

A. Hodgkins

B. Non hodgkins

C. Metastatic ca breast

D. Ca testis

16.Most common complication after pancreatitis . A. Pseudo cyst

B. Renal failure

17.1.5 cm left lower renal mass . What to do? A. Biopsy b. Partial nephrectomy c. Total nephrectomy

18.One direct q bout hydrocoele, asked inv to differentiate it from others.. Ans: transluminable
19.Pitutary necrosis post deliver Vich was complicated by pph

20.55 yr old female, had radio I t/t for goitre now presents with (features of hypothyroid) ..Tsh t3 t4 were given.
Management asked:

A. Thyroxine 25

B. Thyroxine 100

C. Thyroxine 150

D. Carbimazole

21.Hyperthyroidptnt has to start Ocp.. What to do ?

A. Cease carba n start Ocp

B. Increase carba dose n start Ocp

C. Decrease carba dose n start Ocp

D. Dun change carba dose n start Ocp

22.Lady brought her husband who is on Parkinson treatment , n haloperidol for psychosis, now vid cogwheel rigidity .
What to do ?

No option for benztropine

I chose decrease haloperidol

Nitika

23. X Ray of tibia fracture. Deep lacerated wound on fracture site. What is the most imp next step ?

A. Wound debridement

B. External fixation

C. Internal fixation

D. Antibiotics

24. X Ray scaphoid given, dx asked.

25. Pic of dacrocystotis. Treatment asked:


A. Oral antibiotics

B. I/v antibiotics

C. Massage

D. Incision and drainage

26. Lady fell down while horse riding. Now she has come with pain abd, BP 104/60 . Ct given, dx asked:> rupture
spleen

27. Skin tag on eye lid pic given dx asked.

28. Old recall of copdptntans> decrease oxygen

29. Post partum patient complaining of easy fatiguabilitylathergy breast feeding. Blood picture given, hb low Mcv low
.Next inv?

A. Iron deficiency anaemia

B. Folate deficiency

C. Pernicious anaemia

30. 44 yr old man comes vid painless macroscopic haematuria .Cause ?

A. Ca bladder

B. Rcc

C. Renal stone

D. Pyelonephritis

31. Old lady has plans to go to Europe for a bush walking trip n comes to u for advice . She has been diagonosed vid
Alzheimer's . MMSE 22/30. What will u do ?

A. Advice her not to go as she's not fit for such trips

B. Report to immigration dept

32. 55 yr female with breast lump in outer quad. Mammodone , found nothing. Usg done, found hypoechoeic mass.
What next ?
A. Core biopsy

B. Reassure

C. Excision biopsy

33.Body dysmorphic concern is present in all except :

A. Hypochondriasis

B. Amputated leg

C. BDD

D. Anorexia nervosa

E. Non dominant parietal lobe lesion

SYEDA MAHI RECALLS 24th JUNE

Alhamdulillah just finished with exam..full of recalls with some option changed
1.20 yr boy with sore throat. What will lead you to think its viral? A)previous tonsilitis,b)exudation in
tonsil c)laryngitisd) temp 37
2.ctg hr 115, variability 10, acelaration ten no deceleration- normal ctg
3.aspergus girl recall
4smoker with tonsiler firm non tender swelling- squamous cell ca
5.dvt scenerio asking rx asked
6.ckd pt with pe asking inv
7)famous scenerio of somalian boy with seizure
8.old scenerio of homeless man with police mafia fear-ask colateral info
9) confused ill man asks to be witness of his will-refuse
10))alchoholic agitated difficult to control- imhalop, oral benz, oral thymin iv bnz
11)) old man smoker with 1 year breathless, hazy lower lung, couging blood-x ray was
oofbtonchietasi
12) man with lacerated wound 3 years ago last tt booster- nothing now
13)same scenerio with last tt 8 years ago- give tt now
14))digitalis toxicity ecg
15)pancreatic pseudocystscenerio asking inv
16)ascending cholengitis asking next inv
17)baby dev delay 12 months-cant stand unsupport,cant understand no...
18) scenerio of hyalinmem disease new born giving high o2  stiĺlhyoxic- pneumothorax
19) old lady with fecal incontinence.. next dre
20) lady 40 years comes for breast screan mother got ca at 57, nothing abnormal in mammo- adv to
do bianualmammo from 50
21) boy 23 yr comes for colon cascrean, father dx at 35 and paternal uncle at 40
22)lady with irregular mens now no mens for 1 year obese fsh low lh normal what test to dx-
testosteron
23)obese lady irregular mens last lmp 7 weeks back. Confirmed preg from home check..now
bleeding.. no other pelvic finding- serial beta hcg, usg pelvis, lap
24) woman with severe abd pain in 6th week lmp normal cycl of 4 to 6 weeks. no other pelvic finding,
no tenderness, no buldging dx-corpus luteal cyst complication??? Ectopic???
25) obstructed labour
26)indication of cs
27) old lady sigmoid volvulus
28) catatonic schizo
29) ca colon rx after surgery
30)nurse with son who had a head injury few days back was wearing helmet no abnormality except
intermittent headache.. reassure or ct??
31) bamboo spine initial rx
32) lady 45 with 4 weeks malaise, fever, wt loss arthralgia, flushing temp 37 after 4 hours 38.8 what
invana,crp,blòodcs, etc
33) old dm lady with painless ulcer yellow exudate redness spreading to whole dorsum of foot after
hosp admit debribementwhats next- mri, xray, oral antibiotic, iv antib
34) 4 or 5 from uti
35) depression
36) gad with insomnia scenerio all benzo in option except mitrazipine
37) boy no interest in study now become veg as he thinks meat is poisonous- tell him to change his
study subj, get help from local mental health group, drug screaning, its normal to have this prob
38) boy 20 yrsdont do any thing watch tx whole nyt sleeps all day- mitra???
39) 40 years painful breast lump mobile round firm dx???
40) copd confused o2 40 co2 60 what mx- intubate
Cant remember any more now... i will post as soon as i can recall, there was no prob in diagnosis
but options were sometimes confusing. . No prob with time mx..dont know how my exam went. But
thanks to all for your support..keep me in your prayers .

My recalls 24 June Bangkok !


Neonate with skin lesion (described only) with pus .. Organism asked: - Staph - HSV
2) molluscum pic ... No exclusion
3) man from Ethiopia with hx of hCG vaccination now mtx 12 mm no s/s asked next .... Chest Xray
4) swollen inflammed scrotum with black tissue in DMII patient after Antibiotics and IVF what to do:
- blood culture - wound debridement
4) CT IMAGE subdural hge
5) CT IMAGE ICH white rounded lesion next Inv. - CT angio - MRI
6) complete heart block with heart rate of 36 .. Pt on digoxin enalpril n metoprolol ..Mx: pace maker
7) ECG Rapid AF asking Dx
8) ECG mobitz type II what to stop? - Digoxin - ACEI - Apirin
9) 5 sentences about CTG to select which one is the right one !
10) Chest trauma with bilateral # ribs ..Hemothorax is there too .. Patient is vitally stable n maintaing oxygen
saturation .. Not breathing well because of pain .. Next step : morphine
11) Patient with knife in chest . Haemodynamically unstable . After giving fluids and oxygen whts next .. -
cross matching - chest tube
12) Old man with Fx one leg he will use the walking cane (crutches) where he will put it?!!
same side of affected leg and move it with the healthy leg
The side of the healthy leg and move it with the affected leg
13) Risk factor for osteoporosis in a 60 year old : early menopause at 35
14) Patient on treatment for AIDS . Developed Hematuria and dysuria with flank pain : non contrast CT
15) least 5 yrs survival rate Cx : - testis - lung - pancreas - stomach
16) Q about staging of colon Cx ?????
17) MI from 8 months , known AF on warfarin . New X Ray of Cx colon, patient was stable, no distressing
symptoms what to do: - urgent operation - postpone until warfarin effect reversed
18) Another similar Q but patient with prosthetic valve
19) Mother brought her daughter claiming that she is behaving differently and some troubles with father ,
sitting alone , lack of interest asking which Q (interviewing the daughter alone) will help you know if she has
depression ? - ideas about suicide - does she feel satisfied about herself - lack of sleep
20) recurrent candidal vaginitis treated every time with clotrimazole this time when you swab the surface bleed
asking what to do ? - cervical swab for culture - vaginal swab for culture - HIV test
21) 3 or 4 milestones Q one of them 12 months and another 18 months
22) Irregular period , dark skin areas , normal hormones Dx? - POF - PCO - Metabolic S
Unfortunately, I had a lot of new questions (Pilot Q :)) which I can't remember because of the lack of sleep
Please avoid reading any questions before the exam or even a day before to avoid what happened with me
This is what I can recall up till now . If I remembered any other question or topic I will update you. Remember
me in your prayers.... Thanks god for everything

1. Scenerio of Hyperkalaemia in, ECG shows severe


hyperkalaemia.Initial management,
a.Insulin
b. Isotonic solition
c. 5% dextrose
d.Infusion of Calcium Carbonate. Ans: d

2. Post op.3rd day Na. 110 mmol/L, k 2.6, Cl 93, Hco3 16,
pt is allert & take some sips of water& not
confused . Which electrolyte to give initially?
a. Na
b. K+
c.CL
d Hco3
Ans: b

3. A pt took 3 tab of 75mg of Dothepin . Which


complication mostly occure of this pt?
a. Hepatotoxicity
b. Gastric bleeding
c. MI
d.Cardiac Arrythmia Ans:d
4.A 64 yr pt in his 4rth POD of partial colectomy, c/o mild
fever37.6 C ,associated with tachycardia,
tachypnoea,mild
chest pain with scenerio of resp.acidosis.Appropriate
initial management?
a. Urine R/E
b.Blood C/S
c. ABG
d CXR
e. CTPA Ans : CTPA

5.34 age pt on 5th POD of perforated appendicitis with


mild fever & tachycardia ,no chest sign,
Which one is the common complication in this situation?
a. Atelectasis
b. Pneumonia
c. Wound infection
d.Pelvic abscess Ans ;C

6.Bipolar pt getting olanzapin & lithium with lithium


toxicity scenerio, BP 80/60.pulse 120. Initial
Management?
a. Haemodialysis
b. activated charcole
c. IV infusion of NS Ans: C

7. Pt taking risperidone & Lithium developed Acne & wt


gain.Which drug is responsible?

8.Women of 35 yrs came at 16 wk pregnancy.Her brother


has down syndrom. Pt worried as her baby could have
this problem.
Appropriatr investigation?
a. Beta HCG,Alpha fetoprotien, PApp
b.Amniocentesis
c.Chordocentesis

9. Same HBQ 2.150 but ask for investigations.

10. HBQ 3.111

11. Mentaly retarted girL came with mother, asking for


appropriate contraception.Ans:Mirena

12. A 16 yrs girL with no 2ndary charecteristics, ht & wt r


below the level of one & half of her peers, which one is
initial investigation?
a. FSH& LH
b. usg pelvis
c. Caryotyping
d. S.oestrogen
e. CT pituitary Ans: a

13. A man 64 yr age ,came with cough & yellow


productive sputum with fever 38.8 degree C, O2 92%.
which one is the most appropriate Rx?
a. I/V Ceftriaxone
b. I/V Penicillin
c. Amoxyclav
d. Doxycyclin Ans: C

14. A man 64 yr age ,came with cough & sputum & mild
ocassional haemoptysis with childhood H/O pneumonia.
CXR given looks like bronchiectasisasking Dx.

15. 21 yr oLdboy came to GP which screening is


appropriate?
a. Lipid profile
b. HTN
16. 25 yr old women with the result of LSIL , what is ur
next action?
a. Do PAP smear after 6 months
b.Do PAP after 1 yr
c. PAP after 2 yr Ans: b

17.HBQ 3.287

18. HBQ 3.282

19. Which muscle injury occure in long lithotomy


position?
a. E. HaLusis Longus
b. F. HaLusis brevis
c F. Halusis Longus Ans: a
20. HBQ 3.303

21 34 wks pregnancy with clear vaginal fluid . Usg


showsh Normal foetus , amniotic fluid index
normaL,membrane intact. Which one is appropriate Mx?
a. Vaginal swab gm stain & C/s
b. Urine C/s
c. Blood C/s Ans: a
Test Date: 25/06/2016 Dr.TahiraSaeed Location: Sydney

1.Ureteric colic scenerio. Pt was little dizzy but otherwise all normal except pain. After adequate analgesia,
what will b next management
1.send him home and ask ti drink plenty of water
2.send him home and ask ti sieve urine for stone
3. Refer to urologist
4. Plain abdominal xray

2.Vegan pregnant lady. In addition to folic acid she needs


1 iron and thiamine
2 iron and zinc
3iron and vit c
4vit c and vitamin d
5 vit k and iron

3.Pt with urinary incontinence. Most appropriate next investigation


1 urine dipstick
2urine culture
3 urodynamic studies

4.Best data to collect for study of prevalence of hepatitis c in a community


1 number of pts with positive hep c antibody
2 pts e positive hep c PCR
3 pts e hep c antibody and deranged LFTs
4 pts receiving treatment for hep c

5.Pt with increasing forgetfulness. Visual hallucinations. Repeated falls. Which feature will b there to help u
diagnose
1. Disorientation
2 repeated episodes of confusion
3 difficulty in face recognition
4 psychomotor slowing

5.Child with vericella treated a few days ago.idont remember name of drug for infected pustules. Now having
ataxia and nystagmus. Most appropriate next investigation will be
1 vericellaIgM
2 LP
3 CT

6.Mass in anterior leg, firm regular, attached to gastrocnemius muscle, most appropriate inv ?
A. MRI
B. U/S
C. CT
D. Biopsy

7.24 yr female had 2 babies before, amenorrhea 2 years, all normal hormones (prolactin, testosterone,FSH,LH)
, estrogen 70, US 3-4 multiple cyst, Dx ?
A. PCOS
B. Hypothalamic dysfn.
C. Pituitary Adenoma
D. POF

8.Same  above  Scenario  but  ↑↑↑  FSH  and  ↓↓↓  estrogen,  Dx?
A. PCOS
B. Hypothalamic dysfn.
C. Pituitary Adenoma
D. POF

9.Molluscum C. pic exclusion ?

10. 24 yr female cough, bloody  sputum,  hematuria,  rash  on  ant.tibia,  ACE  ↑,  Inv  ?
A. Skin Biopsy
B. ANA
C. ANCA

11.Xray hilar L.N &


chest symptom &hematuria , Next inv. ?
A. Ca level
B. CT Abd.
C. XrayAbd.

12.Old lady in nursing home. Swelling and redness in parotid region. Cause
A.poor oral hygiene

13.24  yr  female  cough,  bloody  sputum,  hematuria,  rash  on  ant.tibia,  ACE  ↑,  Inv  ?
A. Skin Biopsy
B. ANA
C. ANCA

14. Xrayhilar L.N &


chest symptom &hematuria , Next inv. ?
A. Ca level
B. CT Abd.
C. XrayAbd.
15.38 week preg., cervical dialt. 3cm, 200ml bleed, U/S normal baby, Post. placent ?
A. ARM
B. Admission and Obervation

16.old POF scenario: she dosen't want to be preg. & Don't have sex ?
A. OCP
B. HRT
C. POP
D. Estrogen

17.Old male present with constipation, have ca with bone metastasis, took 20mg morphine for pain last week,
Inv ?
A. Morphine level
B. AbdXray
C. Serum Ca.

18.9 month boy chest Xray normal, grunting & 39' fever, given O2 n IV fluid, ?? (same old recall)
A. RSV
B. Bacterial pneumonia
C. Asthma

19.23 yr female epistaxis, headache, malaise, dizziness, LABS: Hb-8.5 Ca-1.9 INR-1.5 Ferritin  ↓  , Rx?
A- Vit. K
B. Iv Ca
C. Plasma
D. BT

20.47 year female afraid from osteoporosis, what is risk factor of Osteopo. in history?
A. Family Hx at 80 years
B. Early menopause at 35

21. Open flial chest, pain, distressed, cyanosed ? (most appropriate next)
A. Morphine
B. Needle thoracotomy
C. Cover wound

22. ECG wide complex , slow af biventricular failure, basal crepitation, E.F=40% on digoxin,lisnopril,verapamil
?
A. increase digoxin
B. stop all drug and check in 12 hours
C. decrease verapamil
D. Add lasix 20ml/day

23. aborignal female 3 month dyspnea, on examination b/l basal crepitation and diastolic murmur, not smoker
but binge drinker, next step ?
A. ECG
B. Echo
C. alcohol counsel about intake

24.Recall of chronic limb ischemia _ gradual walking program

25. 80 yrfemale , her daughter will have baby in 2 days, what to take?
A. DTP vacc.

26. Baby in 7th day of age jaundice, Total Bil. 240, Direct 120, palpable spleen 2cm below c.m ?
A. Haemolytic Anemia
B. Biliary Atresia
C. Breast Milk Jaundice

27.23 yr female, palpable spleen, pic of pancytopenia, rash on her legs Dx ?


A. SLE
B. ITP
C. HSP

29.Old age , fever sign of pyelonephritis, BP 90/60, confused and dysuris, pus in urine in urinanalysis, u send
for culture, GFR<20 *managment ?
A. Amox
B. Trimeth. &Sulfa
C. Gentamycin

30. CT given (Ischemic stroke), Old female, Rt side hemiplegia, dysarthia came after 1 hour from symptom,
best Mx ?
A. rtPA
B. Clopidogrel& Aspirin
C. Clopidogrel
D. Warfarin

31. Old male on antidepressant for 8 year, MMSE score- 23 , now irritable and insomnia, not socially active,
lazy all the time, dont talk to anyone ?
A. increase antidepressant
B. change antidepressant
C. Add Antipsych with antidepressant

32. You r in family party, your cousin have exam next week, want BZD ?
A. Give BZD
B. Give melatonin
C. Physiotherapy & exercise
D. tell his parents
E. Tell him to see formal doctor for advice

33. Male, MMSE 18, HTN &IHD ?


A. Memantine
B. Rivastigmine

34. Male, pancreatitis two months have tenderness and fullness in Abd. , On CT 10cm Pseudocyst ?
A. Endoscopic Gastrectomy
B. Laprotomy

35.A woman on prosthetic valve, she is taking warfarin and you are checking her INR every week.
Date INR WARFARIN dose in mg
2nd April 1.5 4
9th April 1.75 5
16th April 2 6
23 April 2.1 ?
what is the dose of warfarin will you give for this week?
a. 5 mg
b. 6 mg
c. 7 mg
d. 8 mg
36. Male immigrant, Chest symp. , Xraydone .its apical tb, sputum +ve bacilli
A. INH 9 months
B. Isolation

37 Periorbital cellulitis, eyelid swelling, orbital redness, fever, he can move eyeball, what is Rx ?
A. IV fluxacill.
B. Oral AMox
C. IV Ceftriaxone

38. 24 yr female, no GI symptoms, anemia, IDA pic, Upper GI endo free, next ?
A. HB Electrophoresis
B. Colonoscopy
C. Small bowel Biopsy

39. Capsule endoscopy recall

40. Male 70 yr want to do check up for insurance , Plasma Protien Increase, s.albumin 3 (n: 3.5-5.5) , No
symptoms ? (what to do next)
A. Serum immunoglobulin
B. Plasma Protein Electrophoresis
C. Bone Marrow Biopsy

41. Spine X-ray bamboo sign, back pain, Now Can't bend, took PCM but not relieved?
A- Physiotherapy &Hyfrotherapy
B - Rest
C - Methotrexate

42. Old male, backpain, Xray done, Little osteoarthritic changes in spine without compression on spinal cord,
he want opoids ?
A. GIve
B. Rest
C. Exercise
D. Tell him morphine is taken only in malignant

43.BMI 25 in boy, came for tonsillitis, you find that parents r obese too (old scenario)
A. Decrease watching tv
B.Wt. Reduction Schedule
C. Reduce Caloric intake

44.male attack of angina and hypertensive, 170/? , s.cholesterol 5.5, most risk factor for IHD?
A-Angina
B-BP

45. Irreducible mass pain , no impulse cough, lateral to pubic tubercle?


A- Surgical
B- AbdXray
C- US

46. 6 wk boy was ok and increase weight, 1 wk H/O vomiting but he is well hydrated but not increase weight
since vomit started ?
A-GERD
B-PS

47.  Middle  age  man  ,  ↑↑  ca,  his  father  75  yr  have  ca  ↑↑  but  didnt  take  Rx,  PTH  ↑↑,  what  is  your  Inv  ?
A. Urinary ca
B Parath. scan
48. Pic of GOUT but renal impairment
A. Stop Allopurinol and give colchine
B.  ↑  allopurinol  and  give  colchine
C. Give allopurinol and give stenoroid
D. Stop allopurinol and give steroid

49. egg allergy Q

5o. Dumping recall :Pt had ca stomach, did partial gastrectomy, after meal by 1 hour have tachycardia,
sweating and hypotension, Dx?
A- Early dumping
B- Late Dumping

51. Bariatric surgery history 6 weeks, now abdominal pain and vomiting
A barium swallow

52. Pilonidal Sinus pic , asked Rx?


A- Fistulectomy
B- Marsupialization
C- Antibiotic

53. Female on Clozapine ,devloped tachycardia and flattened T wave, Inv ?


A- Echo
B- Troponin
C- Holter ECG

54. Female on venlafaxine ?


A-BP
B-LFT

55. BMI 14 in female, excessive exercise, you will do all inv, except ?
A- ECG
B- Electrolyte
C- Gonadotropin
D- LFT
E- Bone Density

56. Initial adverse effec after taking resperidone ?


A. Wt. gain
B. Hyperglycemia

57 Menopause on HRT, Initial beneficial effect ?


A- Insomnia
B- Depression
C- Cognitive function & Dementia

58.pic of pyogenic granuloma after fingure injury. Treatment asked


A. Local excision
B. Incisional biopsy

59. pregnant 24 weeks ,cervical dilated 3 cm & uterine contractions a-IV salbutamol
b-Oral erythromycin
c-IV oxytocine
d-IV methylprednisolone
60.HIV positive patient on antiviral drugs comes with hematuria, what investigation needed:
a. non-contrast CT
b. triphasic CT
c. U/S

61. Features of moderate croup(stridor and accessary muscle usage on rest but alert), what to give:
a. Im methyl predinisone
b. Oral prednisone

62. 3 cases of salmonella, which study


a.case control
b.cohort study

63. case of trauma and fracture of obital floor -numbness on cheek

64. Young age football player suddenly collapsed without any event and later regain consciousness and returns
to game with 30mins, what is dx:
a.vasovagal
b.arrhthymia

ct scans with long scenerios one smal bowel obstruction


one was brain metastasis
Achlasia
Fecal impaction
One i dnt know what was that
Pic of ingrowing nail mx(handbook question)
1...picture of pyogenic granuloma
A excision with
B diathermy
C cryotherapy
D local excision
E incisional biopsy
2..Dumping recall : just dumping not early of late
3..Hx of a man with GERD , obese , smokes 15 cigaretes, drinks 2 cans ofbeer a
day and eats spicy foods. Best long term treatment?
A. Reduce weight
B. Reduce spicy food
C. Reduce alcohol
D. Stop smoking
4... 24yrs. old woman with secondary amenorrhea, normal BMI, FSH: 55, LH: 54,
oestradiol: 77, Prolactin: normal, in USG of ovaries: 3-4 cyst, all hormone
elevated.if she is not interested in her sexual life and not want to conceive what
is the most appropriate treatment?
a. POP
b. menopause hormone therapy
c. OCP
d.. Metformin
e.clomiphene
5..Ecg of torsa de ...what is the most previous disease before it
A. Myocardial inf
B.heart block
C.cardiomyopathy
D. Chf
6..Lady with hx of GDM, otherwise normal and non diabetic, what to advise?
a. check FBS three yearly
b. check during next preg
c. OGTT 2 yearly
7..A diabetic 65 year old woman noticed an ulcer on her foot for 7 days she came
to hospital and u admitted her, surgical debridment was done and the ulcer is 1
cm and oosing a clear fluid , next best step ?
a. Oral amox. Clax + metronidazole
b. IV ticracillin + metronidazole
c. MRI
d. Dressing and checking the wound everyday
e. Wound toitel with povidone iodine
Another scenerio of diabetic foot management was asking about after dressing
next
8..An obese woman with hirsutism and occasional migraine headaches came to
you for OCP prescription what will you give her ?
A) barrier method
B) next 3 options were just different combinations of oestrogen and
progesterone in pill
C) Give depot provera injection 3 monthly
9..patient travelling to endemic area of TYPHOID,, what is ur advice
A Bottled water
B avoid vegtables and fruits
C wash hands
10.. a scenario of a woman who has repeated hallucinations when she tries to
sleep, she also had few episodes of amnesia in the past few months followed by
hallucinations, what is the cause
A Temporal lobe epilepsy
B Meningitis
C Migraine
D Cannabis intoxication
Another scenario that a women with temporary memory loss something like that
sorry i cant remember options were basal ganglia occipital lobe temporal
11..patient with renal failure got uti tx
A Gentamicin
B Nitrofurantoin
C Trimethop/sulpha
D Amoxicillin
12.. A 2 year old child presented with stridor, harsh cough, temp 38 degrees and
intercostal recessions, he is alert and active, how will you manage him
Oral prednisolone
Nebulized adrenaline
Iv steroid
Nebulized salbutamol
13,,You r in family party, your cousin have exam next week, want BZD ?
A. Give BZD
B. Give melatonin
C. Physiotherapy & exercise
D. tell his parents
E. Tell him to see formal doctor
14..treatment of delusion and bipolar mania during 3rd trimester ??
A. Carbamazepine
B. Lithium
C. Clozapine
15..A man with schizophrenia comes to you and shares with you
persecutorydelusions he has of a friend. He tells you to not tell anyone about his
illness. What will you do?
A) Tell the friend anyway
B) Maintain patient confidentiality
C) Tell him to bring this friend next time
D) Send written note to a friend
16..A patient with MI history 2 months ago, presented with acute cholecystitis.
What’s  the  most  appropriate  management?
•  Percutaneous  cholecystectomy
•  Analgesics
•  Antibiotics
17..1.patient feels there are funny games arround going on and there is someone
who will harm him cause he win a promotion and he mentioned to police there is
terrible think he expect is coming in coming days :
1. idea of reference
2.delusional
3.derailment
4.schizotypal personality
2 scenarios of pulmonary embolism
18..21. Pt submitted for hemicolectomy but didn't so the operation. (Didn't
mentioned why) she is very well and her labs
Na 110 (135-145)
K 2.5 (3.5 - 5)
Chloride is low
Osmolality slightly high 291 (upper limit 290)
Glucose is 60 (4.5- 7) i'm sure
A.urine culture
B. Urine osmolality
C. Repeat labs
D. Give LMWH
19..Young female, primigravida came at 38 weeks pregnancy. LOP fetus
position. Head is not engaged. What is the cause?
A. Contracted pelvis
B. LOP position
C. Placenta previa
D. Cephalo-pelvic disproportion
20..Lithotomy position which muscle affected
Ext hallucis longus
Ext hallucis brevis
Flexor hallucis longus
21..ureteric  stone  high  in  position  making  pain..  it  is  2  cm  but  doesn’t  block  the  
passage of urine.what is the mx
A- lithotomy
B- some substance that dissolve the stone
C- ESWEL
D- Ureteroscopy with basket removal of stone
22..A  man  came  because  he  can’t  see  the  temporal  side  in  his  right  eye  and  
nasal side of his left eye field, where is the lesion ?
a. Optic chiasma
b. Right optic nerve
c. Left optic tract
d. Right visual cortex
e. Right Optic radiation
23..16 yrs old girl comes with complaining of recent start of sexual activities and
seeking contraception advice ,what would be best advice beside contraception?
a)chlamydia PCR
b)gonorrhoea
c)syphilis
d)AIDS
24..dm pt with foot ulcer, after debridement wt is next?
1. i/v ticarcillin+clavulinic acid
2. oral amoxycillin+clavulinic acid
3. mri
4. xray
25. A lady with h/o MVA came with feature of shock and & stridor.Neck x ray
showing # 2nd cervical spine. Asking initial Mx.
a.Cricothyroidectomy b. Tracheostomy c. Intubation
26..Child with URTI and after two days presents with BP 140/90, edematous face
and bilateral ankle adema, urinalysis show rbc++,protein++, aso titer raised, what
to give him:
a.Iv hydralazine
b.furosmide+iv fluid
c.steroid
27.. Pt with candida (all the features of soreness brick red vagina white
discharge) treated with clotrimazole ,but recurrent several times , she always
responded to topical clotrimazole. What will be the most important step in her
management?
-vaginal swab
-fluconazole
-glucose tolerance test
-hiv antibody test
28..a 45 year old woman, a case of 2yr infertility , her children are 14, 15 and 20
years old, h/o endometriosis in uterine lig , previous h/o pelvic operation, mid
cycle sex. Which of the following is the cause of her infertility , partner never
fathered a child.
increased maternal age
sperm abnormality
ovulation problems
endometriosis
tubal pathology
29.. Old patient taking many painkillers for cancer
He is unresponsive and received naloxone and high flow oxygen after which he
opened his eyes and then he became drowsy again.
ABG Po2 120, PCo2 55, Hco3 26
What to do.
Decrease oxygen
Give naloxone
Intubation
Give morphine
30. 50 year old man who is working in real estate. He used to dress well but
recently he looks in bad shape, shouted at customers and got fired from his job.
His wife brought him, how will you evaluate
A. verbal fluency test
B. minus 7 test
C Others irrelevant
31.  aboriginal  female  who  doesn’t  have  access to medical facilities with 3 month
dyspnea brought by her friend, on examination b/l basal crepitation and a
diastolic murmur, not a smoker but a binge drinker, next step ?
A. ECG
B. Echo
C. alcohol counsel about intake
D. establish an ongoing therapeutic relationship with her
32. Obstructive sleep apnea scenario obese with waist n neck circumstances
given.asking for long term treatment?
A.wt loss
B.metformin
C.Cpap
D.surgery
33.Pancreatic pseudocyst scenario post Pancreatitis episode fullness and stuff
Asking Ix most app
CT
EUS
34.6 wk boy was ok and increase weight, 1 wk H/O vomiting but he is well
hydrated but not increase weight since vomit started ?
A-GERD
B-PS
35. a 25 yrs old male had an accident with multiple ribs fractures ,difficulty in
breathing,with diminished breath sound in rt side (not mentioned whether there is
pain or not),bp90/60 mmhg,pulse 120 ,after initiating iv fluid and resuscitation
,what will be mx???
a)watet seal drainage
b)usg
c)neddle thoracocentesis
d)oxygen
36.18 yrs old painful reddish scrotum (pic given),what is the probable cause?
a)clamydial
b)gonorrhoea
c)staphylococcal
d)streptococcus
37. Pic of GOUT but renal impairment
A. Stop Allopurinol and give colchine
B.  ↑  allopurinol  and  give  colchine
C. Give allopurinol and give steroid
D. Stop allopurinol and steroid
others I cant remember right now,but i ll upload soon more. Thank you friends..
some imp docz
– 25TH JUNE, SINGAPORE

1) scenario of viral meningitis headache neck stiffness flue for 2


days Ct scan normal RBC in csf 200 but no s/o encephalitis
1.acyclovir
2.ceftriaxone
3.analgesia and observe

2).47 year female afraid from osteoporosis, what is risk factor of


Osteopo. in history?
A. Early menopause at 35

3) A patient on peritoneal dialysis who was going well suddenly


declined to have dialysis. On examination she was having slight
temperature and mild tenderness in upper abdomen. What
finding you will most likely find in this patient?
A Disorientation
B Depressed mood
C Blunted affect

4) A girl afraid to stay at  friends  home  for  night  stay,  don’t  want  
to  get  out  of  car.  What’s  the  most  important  in  history?
•  Absence  of  symptoms  in  father’s  absence
•  Night  terrors  and  sleep  walking
•  Hallucinations
•  Irritable  bowel

5) 3yrs  old  asthmatic  child…night  symptoms  also..best


preventer
Fluticasone

6) Ca prostate with gleason score 4.. what is the most


appropriate next management
A radical prostatectomy
B radiotherapy
C chemotherapy
D- External beam radiation
7) Patient wd 5days history of melena admitted from last 3 days
in  hospital…his  endoscopy  and  colonoscopy  eas  normal  but  he  
transfused 4 pints of blood what is the next most approptiate
investigation?
1.sigmoidoscopy
2.capsule endoscopy
3.ercp
4.angiogram

8) old male with anemia, FOBT +ve , upper endoscopy –ve,


colonoscopy  didn’t  pass  after  the  hepatic  flexure  with  normal  
findings, wt next:
a-force colonoscopy to pass
b-capsule endoscopy
c-ct
d- Tagged Red Blood Cell Scan

9) 2 year male complains of cannot climb reflexes ...muscle bulk


is normal ..loss of ankle reflex ,his mother also had loss of
anklle reflex ...
a... ck
b...vit b12
c... muscle biopsy
d…nerve  conduction

10) 8yr old boy, previously was doing well but last 2yrs
academically not normal, teachers worried that he is not doing
well this year, poor in writing and home works. How to reach
diagnosis?
a.audiometry
b.family dynamics
c.ent refer
d.visual test
e. school performance

11) pt with history of travel just returned from thailand, now


shortness of breath, left chest pain, temp normal, and pain
increases  with  cough  and  inspiration  …almost  similar  above  
question but here no history of fever.what is possible diagnosis?
a- peumonia
b- pulmonary embolism
c- acute pericarditis
d- myocardial infarction
12) patient type 2 DM with history DVT present by red leg .temp
38.5 ,pain in calf ms ,venous pigmentation . what to do next?
- blood culture
- Doppler

13) Pt with claudication at 100m distance on rt foot..on


examination, dorsalis pedis is very weak palpable..femoral
artery pulse volume reduced..leg and foot is well
perfused..whats most appropriate initial.
a. reconstructive surgery
b.gradual walking programme
c.aspirin

14) DVT scenario, doppler showed Thrombus extending from


popliteal to femoral
management.
IV Heparin
LMWH
warfarin

15) young male came for insurance check up..on usg there is a 3
cm heterogenous mass on upper pole of rt kidney confirmed by
ct..mx-
a. radical nephrectomy
b. review after 12 months
c. biopsy

16) female has dvt last week started on warfarin.presents with


perforated duodenal ulcer.wat to do-
stop warfarin n give vit k
-proceed with surgery-
give ffp n do surgery-
give iv heparin n do surgery

17) The relationship between number of neavi and development


of melanoma
A. cohort
b.case control
c.randomization

18) Which study to do to find causality between HTN and MI?


a. Case control
b. Cohort
c. Cross sectional
d. Randomised control trial
e. Case series

19) Melanoma skin care screening. 4o years old IT guys come


for screening. What would be the risk for him? That’s  all  given  
in exam. Nothing mentioned.
A)recent diagnosis of melanoma in cousin
B) outdoor working in early twenties
C)history of sunburn in childhood
D) family history of BCC

20) young male who felll suddenly in the field of a match ,


without being touched,after a few MINUTES he regained
consciousness without any intervention and continued playing
what is the case ?
a)- vasovagal attack
B)- Jacksonian seizures
C)- arrhythmia

21) A case of premature ovarian failure,dnt want to conceive,


management??
1)COCs
2)HRT
3)POPs

22) which antibiotic combination is nephrotoxic:


a.gentamicin plus chloramphenicol
b.gentamicin plus cephalothin
c.gentamicin plus cloxacillin
d.tetracycline plus erythromycin.

23) Hyperkalemia Ecg combination asked --


A.Amiodarone +Aspirin
B.frusimide+Aspirin
C.Amlodipin+Amiodaron
D.Aspirin+ Nitrtriglycerite
E. aspirin pls isosorbide mono nitrate

24) ECG of inferior MI within 2 hours..whats most appropriate


next?..
a.GTN infusion
b. TPa
c. angioplasty
D.morphine
E.observation
25) Ecg of heart block 2nd degree mobitz type 2 HR 35 but pt
comes to ur clinic not emergency BP stable no complaints ,he is
on polypharmacy
1. stop meto
2. Pacing

26) lbbb ecg..unstable...pacemaker

27)  Colles’  Fracture  --- review in 2 weeks

28) Pleural fluid xray- Aspiration

29). COPD pt come to ED, o2 88% they give him 6L/o2 and
now his o2 is 96%. what is yr next Mx?
A. V/Q
B. IV Amox
C. Salbutamol
D. Reduce o2 to 2L

30) hyperkalemia 6.5 inc confusion plus vomiting next


a.Dialysis
b. insulin dextrose

31) . Man came with 3 cm painful tender mass over inguinal


ligament , appeared after he returned from a strenous trecking
A inguinal hernia
B femoral hernia
C Iymph node

32) cervical spine frature attempt to suicide not able to breath


what to do in emergency department..
A..crico
B.ett
C.tracheostomy

33) 8yr old boy, previously was doing well but last 2yrs
academically not normal, teachers worried that he is not doing
well this year, poor in writing and home works. How to reach
diagnosis?
a.audiometry
b.family dynamics
c.ent refer
d.visual test
e. school performance
34) .8 month old child present with fever 39.2'C,
cough,dyspnoea,tracheal tug present but chest was clear.after
admission he was given oxygen and iv access had done ,whats
ur dx?
RSV bronchiolitis
sterp pneumonia
Coxsackie

35) boy has asthma attack..6 puff given...baby cant speak in


sentences...reduced air entry in lungs...wat next
salbutamol at 20 mins interval
or inhaled ipratropium bromide

36) A woman with a left side blurry vision of two days.


Fundoscopy showed nothing due to bilateral cataract what is the
most appropriate next investigation
a. ESR
b. carotid angiography
c.MRI
d.CT head & neck

37) women on treatment of schizo having clozapine for her


condition now presented with chest pain, palpitation and
tachycardia
a.Clozapine level
b. Echo
c.Troponin level

38)  Clear  Scenario  of  IgA  nephropathy  …  renal  issues  with  


URTI question

39) Pancreatic pseudocyst scenario post Pancreatitis episode


fullness and stuff
Asking Ix most app
CT (ans)
EUS (endo US)

40) Obstructive sleep apnea scenario obese with waist n neck


circumstances given.asking for long term treatment?
A.wt loss (ans)
B.metformin
C.Cpap
D.surgery
41) a boy at er said he do sex with anyperson he wants..lives on
street...after explaing him risk and hazards wats next
a)...tell that life style is dangerous
b)..report of child pretoetion authority as high risk child

42) Famous recall of young couple with infertity, regular


menses, mid cycle sex. Cause of infertility?
A- Tubal abnormalities
B- luteal phase defects
C- Sperm antibodies
D- Bad timing

43) 45 yr male with malaise 3weks, microscopic


hematuria,dysuria, o/e all abdomen normal,on microscopy pus
cells positive,no organism on culture, Dx
1.t.b nephritis(ans)
2. C.a bladed
3. RCC
4.bladder stones
44) farmer U/S liver multiple hypoechoic lesion, had
hemicolectomy for ca. colon before 2 year, most app inv ?
A. Hydatid cyst serology
B. Triphasic CT

45) 38 week preg., cervical dialt. 3cm, 200ml bleed, U/S normal
baby, Post. placent ?
A. ARM
B. Obervation

46) Travelling to endemic typhoid, took vaccine already, what


to advice ?
A. Wash hands
B. Avoid all fruits n veg.
C. Bottled water

47) Old male present with constipation, have ca with bone


metastasis, took 20mg morphine for pain last week, Inv ?
A. Morphine level
B. Abd Xray
C. Serum Ca.
48) 24 yr female, no GI symptoms, anemia, IDA pic, Upper GI
endo free, next ?
A. HB Electrophoresis
B. Colonoscopy
C. Small bowel Biopsy

49) 6 wk boy was ok and increase weight, 1 wk H/O vomiting


but he is well hydrated but not increase weight since vomit
started ?
A-GERD
B-PS

50) Young male with ingestion of taking 20 tab of 500 mg


comes to ED after 4 hrs no symptoms what next
1. NAC
2. charcoal
3. check PCM levels

51) algerian Vegetarain lady with labs showing dec, HB, dec
ferritin, dec mcv, dec mchc , complains of malaise and fatigue,
what is the cause of this :
a. Thalasemia
b.Celiac disease
c.Dec Iron in diet

52) Pregnant female with genital herpes how to differentiate 1ry


From recurrent herpes simplex:
•  HSV  Igm  now
•  HSV  specific  serology  now  

53) pregnant lady who her husband abuse her and make sure not
to leave any marks what is your next action:
-urgent admission
-urge her to call police
-reassure
54) A girl brought an old man who has
dysphasia,dyscalculia,Right left disorientation,he is heavy
smoker(30 pack year) and drinker.what is diagnosis
a.temporal metastasis
b.vermis carcinoma
c.frontal meningioma
d.left parietal glioblastoma multiforme

55) A patient has nausea vomiting, epigastric pain or tenderness


(with no signs of peritonitis). From the history, you knew that
the patient had gastric ligation surgery before, what is you initial
management?
a. barium
b. urgent surgery
c. CT scan

56) pregnant at 37 wks..sprained ankle badly..u applied crepe


bandage and and advised to keep foot elevated..what is the best
advice u can give regarding pain management?..
a. avoid codein
b. avoid all pain medicines unless it is intolerable
c.Avoid NSAIDS
d. use topical medicine

Вам также может понравиться